e2

¡Supera tus tareas y exámenes ahora con Quizwiz!

Penicillin

Amoxicillin (Amoxil, Trimox), Ampicillin (Principen)

Chlamydia

Venereal disease, endometriosis

Tetracycline

Doxycycline (Vibramycin)

Negative feedback loop ensures endocrine homeostasis by:

Inhibiting secretion from the first hormone.

Which of the following is a symptom of Cushing's syndrome?

Moon face

Antitubercular Drug

Rifampin (Rifadin, Rimactane)

Rickettsia

Rocky mountain spotted fever

Type 2 diabetics never require insulin since their pancreas still secretes insulin. True False

false

NPH

intermediate action insulin

lantus

long acting insulin

apridra (insulin glulisine)

rapid action insulin

solu-cortef (hydrocortisone)

short acting corticosteroid

The client has diabetes mellitus type 2. The nurse has taught the client about the illness and evaluates learning has occurred when the client makes which statement?

"My cells cannot use the insulin my pancreas makes."

The client has been diagnosed with diabetes mellitus type 1. He asks the nurse what this means. What is the best response by the nurse? Select all that apply.

1. "Without insulin you will develop ketoacidosis (DKA)." 2. "It means your pancreas cannot secrete insulin." 3. "The endocrine function of your pancreas is to secrete insulin."

The client has been diagnosed with diabetes insipidus. What will the best plan by the nurse include? 1. Assess for increased urine production. 2. Assess for hyperglycemia. 3. Assess for hyponatremia. 4. Assess for fluid retention.

1. Assess for increased urine production.

A client will be prescribed an oral corticosteroid for treatment of a rash. The prescriber says, "In order to avoid adverse effects, I am prescribing the smallest amount possible." What other prescribing regimens could the nurse suggest to achieve this same effect? Select all that apply. 1. Give the oral medication every other day. 2. Administer a larger dose for a few days and then gradually decrease the dose until the medication is discontinued. 3. Use a topical application instead of an oral product. 4. Start with a small dose and work up to the desired dose. 5. Administer the doses intramuscularly for the first few days of treatment.

1. Give the oral medication every other day. 2. Administer a larger dose for a few days and then gradually decrease the dose until the medication is discontinued. 3. Use a topical application instead of an oral product.

The client receives metformin (Glucophage). What will the best plan of the nurse include with regard to patient education with this drug? Include all that apply.

1. It reduces insulin resistance. 2. It must be taken with meals. 3. It decreases sugar production in the liver.

The client has hyperthyroidism. The nurse teaches the client to avoid which food selections? 1. Soy sauce 2. Milk products 3. High-calorie foods 4. Caffeine-free soda

1. Soy sauce

A client diagnosed with an inoperable tumor of the adrenal gland is prescribed mitotane (Lysodren). The nurse would attribute which assessment finding as indicating an adverse reaction to the drug? Select all that apply. 1. The client reports that food is not appealing. 2. The client reports joint pain. 3. The client reports ringing of the ears. 4. The client reports increasing depression. 5. The client reports dizziness.

1. The client reports that food is not appealing. 4. The client reports increasing depression. 5. The client reports dizziness.

A client with diabetes mellitus type 1 is found unresponsive in the clinical setting. Which nursing action is a priority? 1. Call a code. 2. Treat the client for hypoglycemia. 3. Call the physician STAT 4. Assess the client's vital signs

1. Treat the client for hypoglycemia. When a client with diabetes mellitus type 1 is found unresponsive, the nurse should focus on and treat for hypoglycemia, as this is more likely than hyperglycemia.

Vasopressin is used in the treatment of 1. diabetes insipidus. 2. dehydration. 3. electrolyte imbalances. 4. diabetes mellitus.

1. diabetes insipidus.

The nurse understands that which of the following drugs falls under the classification of biguanides? 1. Metformin HCl 2. Repaglinide (Prandin) 3. Tolbutamide (Orinase) 4. Acarbose (Precose)

1.Metformin HCI is the only drug that falls within the classification of biguanides.

Which of the following is a sign or symptom of hypothyroidism? 1. Anxiety 2. Bradycardia 3. Tachycardia 4. Weight loss

2. Bradycardia

A nurse is reviewing the blood work of a client who has recently begun treatment for type 2 diabetes. Which results would indicate that the client is on target with therapy? Select all that apply: 1. HBA1C level is 8.4% 2. HBA1C level is 6.3% 3. Fasting blood glucose is130 g/dL 4. Fasting blood glucose is 100 g/dL 5. Fasting blood glucose is 68 g/dL

2.HBA1C level is 6.3%. 4.Fasting blood glucose is 100 g/dL.

Insulin is released when 1. Blood glucose stays the same 2. Blood glucose increases 3. Blood glucose decreases 4. Glucagon increases

2. blood glucose increases.

The client injects his insulin as prescribed, but then gets busy and forgets to eat. What is the nurses most likely assessment finding? 1. The client will be very thirsty. 2. The client will need to urinate. 3. The client will have moist skin. 4. The client will complain of nausea.

3 Rationale 1: Thirst is a sign of hyperglycemia; the client would experience hypoglycemia if he did not eat. Rationale 2: Increased urination is a sign of hyperglycemia; the client would experience hypoglycemia if he did not eat. Rationale 3: Moist skin is a sign of hypoglycemia, which the client would experience if he injected insulin and did not eat. Rationale 4: Nausea is a sign of hyperglycemia; the client would experience hypoglycemia if he did not eat.

A client has been prescribed a topical antifungal medication for treatment of ringworm. What teaching should the nurse provide? Select all that apply. 1. Avoid alcohol while using this medication. 2. Scrub the area vigorously prior to applying the medication. 3. After applying the medication let the area air-dry. 4.Apply a think layer of the medication 5. Cover the area with an occlusive dressing after applying the medication.

3 and 4

A client is being discharged from the hospital with a nebulizer for self administration of inhalation medication. Which statement made by the client indicates to the nurse that the client education has been successful? 1. "Inhaled medications should only be taken in the morning." 2. "Doses for inhaled medications are larger than those taken orally" 3. "Medicines taken by inhalation produce a very rapid response." 4. "Inhaled drugs are often rendered inactive by hepatic metabolism reactions"

3. "medicines taken by inhalation produce a very rapid response" rationale: Inhaled drugs produce an immediate therapeutic response. Options 1,2,4 are incorrect. Inhaled medications can be used at any time during the day. Drugs for inhaled drugs are small compared to orally ingested medications, and because these drugs go directly to the lung surface area and are readily absorbed, very little of the substance is lost due to metabolism

38. Which of the following laboratory tests best evaluates HIV disease? A. CD4 count B. Complete blood count C. Platelets D. Liver function studies

38. Answer: A. CD4 count Rationale: A. CD4 count best determines the progress of the disease. B. Complete blood count can be used, but does not assess the progress. C. Platelets are not indicated. D. Liver function studies can be used, but do not assess the progress.

The client receives glucocorticoid therapy. The nurse would prioritize assessment for which finding? 1. Hypothermia 2. Hypotension 3. Hypertension 4. Weight loss

3. Hypertension

The client has type 1 diabetes and receives insulin. He asks the nurse why he can't just take pills instead. What is the best response by the nurse? 1. I know it is tough, but you will get used to the shots 2. Have you talked to your doctor about taking pills instead? 3. Insulin can't be in a pill form because it is destroyed in stomach acid 4. Insulin must be injected because it needs to work quickly

3. Insulin can't be in a pill because it is destroyed in stomach acid."

The client has acquired immune deficiency syndrome (AIDS) and has just learned she is pregnant. She tearfully asks the nurse if her baby will die of acquired immune deficiency syndrome (AIDS). What is the nurse's best response? 1. There are special tests that we can do to see if your baby is positive before birth 2. I know that is a great worry for all of us 3. There are drugs that can be given after birth to reverse AIDS 4. Drug therapy can help prevent your child from contracting the virus

4

Which laboratory test best evaluates HIV disease? 1. Complete Blood Count 2. Liver function studies 3. Platelets 4. CD4 Count

4

What is the primary function of the islets of Langerhans in the pancreas? 1. Secretion of enzymes 2. Acting as exocrine 3. Secretion of glucagon and insulin 4. Absorption of insulin

3. Secretion of glucagon and insulin

The client receives treatment with radioactive iodine (Iodine-131) therapy. What will the best evaluation by the nurse reveal? 1. The client will only temporarily accomplish the euthyroid state. 2. The client could safely become pregnant while receiving this treatment. 3. The client will most likely require thyroid replacement therapy. 4. The client does not have to distance herself from others.

3. The client will most likely require thyroid replacement therapy.

The client has been diagnosed with Cushing's syndrome. What will the nurse's assessment likely reveal? 1. Low blood pressure and hypoglycemia 2. Well-healed scars on the upper body 3. Upper body obesity 4. Thin, gaunt appearance of the face

3. Upper body obesity

Which sign or symptom is most typical of an untreated client with type 1 diabetes? 1. Increased energy 2. Weight gain 3. Fatigue 4. Decreased hunger

3. fatigue Fatigue is a typical sign/symptom of type 1 DM due to sustained hyperglycemia.

The client has hypothyroidism and is treated with levothyroxine (Synthroid). The nurse plans to do medication education. What will the best plan by the nurse include? 1. Monitor daily weights. 2. Assess for decreased appetite. 3. Assess weekly serum blood levels. 4. Assess for altered sleep patterns.

4. Assess for altered sleep patterns.

4. During posttest counseling for a patient who has positive testing for HIV, the patient is anxious and does not appear to hear what the nurse is saying. At this time, it is most important that the nurse: A. Inform the patient how to protect sexual and needle-sharing partners. B. Teach the patient about the medications available for treatment. C. Ask the patient to notify individuals who have had risky contact with the patient. D. Remind the patient about the need to return for retesting to verify the results.

4. D. Rationale: After an initial positive antibody test, the next step is retesting to confirm the results. A patient who is anxious is not likely to be able to take in new information or be willing to disclose information about HIV status of other individuals.

An elderly client has hypothyroidism. Which assessment finding would the nurse report to the physician immediately? 1. Dry skin 2. Generalized weakness 3. Muscle cramps 4. Slurred speech

4. Slurred speech

8. The occupational health nurse will teach the nursing staff that the highest risk of acquiring HIV from an HIV-infected patient is: A. A needlestick with a suture needle during a surgical procedure. B. Contamination of open skin lesions with vaginal secretions. C. A needlestick with a needle and syringe used to draw blood. D. Splashing the eyes when emptying a bedpan containing stool.

8. C. Rationale: Puncture wounds are the most common means for workplace transmission of blood-borne diseases, and a needle with a hollow bore that had been contaminated with the patient's blood would be a high-risk situation. The other situations described would be much less likely to result in transmission of the virus.

The function of the thyroid gland is to 1. stimulate growth. 2. control pituitary gland secretion. 3. conserve water in the body. 4. control basal metabolism.

4. control basal metabolism.

Negative feedback loop ensures endocrine homeostasis by 1. stimulating secretion from the first hormone. 2. inhibiting release of the second hormone. 3. preventing under the responses of the endocrine system. 4. inhibiting secretion from the first hormone.

4. inhibiting secretion from the first hormone.

A clinical feature that distinguishes a hypoglycemic reaction from a ketoacidosis reaction is: A. Blurred vision B. Diaphoresis C. Nausea D. Weakness

B. Diaphoresis

Which class of antibiotics is associated with cartilage toxicity and rupture of the Achilles tendon? A. Penicillins B. Aminoglycosides C. Fluroquinolones D. Sulfonamides

C. Fluroquinolones

The nurse is providing care for a patient with a diagnosis of Parkinson's disease who has preexisting liver failure. What class of medications, if prescribed, would the nurse question? Anticholinergics Dopamine modulator Catechol ortho-methyltransferase (COMT) inhibitors Ergot derivative

Catechol ortho-methyltransferase (COMT) inhibitors COMT inhibitors are contraindicated in patients with liver failure or liver disease. Potential liver failure is also a known adverse effect.

Cephalosporins have a different mechanism of action then penicillins?

False

Aminoglycoside

Gentamycin (Garamycin), Neomycin

Bacteriocidal drug kill infectious organisms, whereas bacteriostatic drugs slow down their growth?

True

Clinical manifestations associated with a diagnosis of type 1 DM include all of the following except: A. Hypoglycemia B. Hyponatremia C. Ketonuria D. Polyphagia

A. Hypoglycemia

the nurse understands that which following drug falls under the classification of biguanides? Select one: A. Tolbutamide (Orinase) B. Metformin HCI (Glucophage) C. Repaglinide (Prandin) D. Acarbose (Precose)

B. Metformin HCI (Glucophage)

The client has type 1 diabetes and receives insulin. He asks the nurse why he can't just take pills instead. What is the best response by the nurse?

Insulin can't be in a pill because it is destroyed in stomach acid."

The mechanism of action of antithyroid drugs is to:

Interfere with synthesis of thyroid hormones.

Escherichia

Traveler's diarrhea, UTI, endometriosis

A client is admitted to the intensive care unit with a diagnosis of diabetes insipidus. The nurse caring for the client recognizes that the patient should have the following medication on the medication list: a. Insulin b. Metformin c. Desmopressin d. Hydrocortisone

c

The nurse has finished teaching a client with diabetes mellitus how to administer insulin. The nurse evaluates that learning has occurred when the client makes which statement?

"I should only use a calibrated insulin syringe for the injections." To ensure the correct insulin dose, a calibrated insulin syringe must be used.

"I don't run the risk of blindness and kidney disease like type 1 diabetics."

/

The nursing instructor teaches the student nurses about negative feedback as a feature of calcium homeostasis in the endocrine system. Identify the steps, in order, of this mechanism.

1.The parathyroid glands shut off PTH (parathyroid hormone) secretion. 2.Serum calcium levels fall. 3.PTH (parathyroid hormone) increases serum calcium. 4.PTH (parathyroid hormone) is released.

The client receives nystatin (Nilstat) for a fungal infection in the month. The nurse plans to do medication education prior to discharge. What will the best plan by the nurse include? 1. Swallow the oral tablet whole without chewing or crushing it 2. Dissolve the oral tablet in your mouth and then swallow it 3. Take the oral suspension with a straw to prevent tooth decay 4. Crush the oral tablet mix it with orange juice and then swallow it

2

THe nurse is monitoring the therapeutic drug level for a client on vancomycin (Vancocin) and notes that the level is within the acceptable range. What does this indicate to the nurse? Select all that apply. 1. The drug should cause no toxicities or adverse affects 2. The drug level is appropriate to exert therapeutic effects 3. The dose will not need to be changed for the duration of treatment 4. the nurse will need to continue monitoring because each client response to a drug is unique

2 & 4 Rationale: a therapeutic drug level that is in the accepted range indicates that the drug is at a minimally effective concentration, but not at a toxic level. Because each client response to a drug is unique, the nurse should continue monitoring the client throughout the drugs use. 1,3,5 are incorrect because individualized client responses to drugs can be highly variable, adverse effects, toxicities, or even no effect may occur at levels within therapeutic range. Therapeutic effectiveness of a drug depends on many factors and the therapeutic range of a drug is the level between minimally effective and toxic levels. It is not an indicator of how effective a drug will be in treating an individual condition

A client is recieving amphotericin B. The nurse will reinforce teaching by telling the client that he should report which symptoms? Select all that apply. 1. Constipation 2. Hearing loss 3. Dizziness 4. Feeling hot 5. Heartburn

2, 3, and 4

A nurse is providing community education after and outbreak of amebiasis at a summer camp. What information should the nurse provide? Select all that apply. 1. Brain abscesses are the typical fatal event 2. Amebiasis is contracted through contact with contaminated water 3. The primary symptom is diarrhea 4. Amebiasis may damage the liver 5. Sever muscle cramping is a common symptom

2, 3, and 4

The physician orders insulin lispro (Humalog), 10 units for the client. When will the nurse administer this medication? 1. Thirty minutes before meals 2. Five minutes before a meal 3. When meal trays arrive on the floor 4. Fifteen minutes after meals

2. Five minutes before a meal The onset of action for insulin lispro Humalog is 10 to 15 minutes so it must be given when the client is eating

Superficial fungal infections differ from systemic fungal infections in that superficial fungal infections 1. Can be fatal 2. Are less common 3. Affect hair and skin 4. Affect internal organs

3

The nurse accidentally sticks herself with a needle after starting an intravenous (IV) line on a client with acquired immune deficiency syndrome (AIDS). The nurse tells the supervisor about the accident. What is the best initial response by the supervisor to decrease anxiety in the nurse? Select one: A."Fortunately, the chances of you contracting human immunodeficiency virus (HIV) after the stick are very small." B. "If you are started on medications soon, it will decrease the severity of the disease." C. "Did you use the hospital protocols for starting intravenous (IV) lines on a client with acquired immune deficiency syndrome (AIDS)?" D. "Workers compensation will cover the cost of your illness and medications."

A."Fortunately, the chances of you contracting human immunodeficiency virus (HIV) after the stick are very small."

While obtaining a patient history, the nurse notes that the patient has been prescribed ethosuximide (Zarontin). The nurse suspects that the patient has which type of seizure? Partial Generalized Absence Tonic-clonic

Absence Ethosuximide is used to treat uncomplicated absence seizures.

The nurse is obtaining a medication history from a client diagnosed with genital herpes. Which drug would the nurse expect this client to be prescribed? Acyclovir (Zovirax) Ribavirin (Virazole) Amantadine (Symmetrel) Zidovudine (Retrovir)

Acyclovir (Zovirax) Acyclovir is the drug of choice to treat herpes simplex infections, which includes the virus type 2 that causes genital herpes. Ribavirin is effective against respiratory syncytial virus, zidovudine against human immunodeficiency virus (HIV), and amantadine against Haemophilus influenzae type A.

The nurse needs to know that major adverse effects are MOST common by which drug? Griseofulvin (Fulvicin P/G) Ketoconazole (Nizoral) Amphotericin B (Amphocin) Fluconazole (Diflucan)

Amphotericin B (Amphocin) The major adverse effects caused by antifungal drugs are encountered most commonly in conjunction with amphotericin B treatment. Drug interactions and hepatotoxicity are the primary concerns in clients receiving other antifungal drugs, but the IV administration of amphotericin B is associated with a multitude of adverse effects.

A patient has been diagnosed with genital herpes and has been started on oral acyclovir (Zovirax). What should be included in the teaching instructions for this patient? (Select all that apply.) 1. Increase fluid intake up to 2 L per day. 2. Report any dizziness, tremors, or confusion. 3. Decrease the amount of fluids taken so that the drug can be more concentrated. 4. Take the drug only when having the most itching or pain from the outbreak. 5. Use barrier methods such as condoms for sexual activity.

Ans: 1,2,5 Rationale: Acyclovir can be renal toxic and fluids should be increased throughout therapy. Neuro- toxicity may occur and increasing dizziness, tremors, or any confusion should be reported immediately. Acyclovir does not prevent transmission of the disease and transmission may occur even if the host is asymptomatic. Barrier methods for sexual activity should be used. Options 3 and 4 are incorrect. Fluid intake should be increased, not decreased, and the drug must be taken consistently throughout the entire course of therapy. Suppressive therapy may also be ordered.

What is the action of bactericidal drugs? 1. They disrupt normal cell function. 2. They will slow the growth of the bacteria. 3. They have a high potency. 4. They will kill the bacteria.

Ans: 4 Rationale 1: They do not disrupt normal cell function. Rationale 2: Bacteriostatic drugs slow the growth of bacteria. Rationale 3: Potency is related to the properties of resistance. Rationale 4: Bactericidal drugs kill the bacteria.

49.9 The nurse has taught the client about glaucoma. The nurse evaluates learning has occurred when the client makes which statement? 1. "I must use my eye drops as prescribed for the rest of my life." 2. "I can stop the eye drops when the glaucoma has resolved." 3. "The eye drops only need to be used when my eyes hurt." 4. "I will need to continually increase the dose of my eye drops."

Answer: 1 "I must use my eye drops as prescribed for the rest of my life." Rationale: Glaucoma can be controlled, but not cured; the client must use eye drops for the rest of his life. Glaucoma can be controlled, but it will not resolve, the client cannot stop using the eye drops. Eye drops must be used continuously; most clients with glaucoma do not experience eye pain. Increasing the dosage of eye drops is only necessary if the ocular pressure is not controlled; this must be determined by the physician, not the client.

36.6 The nurse plans to teach a client with human immunodeficiency virus (HIV) infection about zidovudine (Retrovir). What will the best plan of the nurse include? Select all that apply. 1. Zidovudine (Retrovir) will need to be stopped if bone marrow depression occurs. 2. Zidovudine (Retrovir) therapy frequently results in the development of anemia. 3. Zidovudine (Retrovir) prevents spread of the virus through sexual contact. 4. Zidovudine (Retrovir) will slow the disease, but not cure it. 5. Zidovudine (Retrovir) was the first drug developed to treat acquired immune deficiency syndrome (AIDS)

Answer: 1, 2, 4, 5 Rationale: Stopping zidovudine (Retrovir) if bone marrow depression occurs allows the bone marrow time to recover. Treatment with zidovudine (Retrovir) frequently results in anemia. The drug only slows the disease; it will not cure it. Zidovudine (Retrovir) was the first drug developed to treat AIDS. Zidovudine (Retrovir) will slow the progression of the disease; it will not prevent its transmission.

35.8 The client receives amphotericin B (Fungizone) for histoplasmosis. What does the best assessment of the nurse include? Select all that apply. 1. Serum creatinine 2. Serum glucose 3. Serum sodium 4. Blood urea nitrogen 5. Serum amylase

Answer: 1, 4 Rationale: Amphotericin B (Fungizone) is nephrotoxic, so serum creatinine and blood urea nitrogen should be monitored. Amphotericin B (Fungizone) does not affect serum glucose. Amphotericin B (Fungizone) does not affect serum sodium. Amphotericin B (Fungizone) does not affect serum amylase.

35.10 The client is being treated for pinworms with mebendazole (Vermox). The nurse has completed medication education and evaluates that learning has occurred when the client makes which statement? 1. "I need to have three negative stool cultures before I am cured." 2. "I may expel worms for three days after I finish the medication." 3. "I must avoid aspirin while I am on this medication." 4. "I need high-fiber foods to help with passage of the worms."

Answer: 2 "I may expel worms for three days after I finish the medication." Rationale: It can take up to 3 days for all the worms to be eliminated from the GI system. Stool cultures are not necessary. There is no interaction between mebendazole (Vermox) and aspirin. Diet will not impact the passage of worms.

49.12 The client is to receive ear drops. What is the correct method of administration that the nurse plans to teach the client? 1. Sit up straight and turn your head to the side. 2. Lie on the unaffected side to instill your drops. 3. Hold your head back while you instill the drops. 4. Prop your head on a pillow after administering the drops.

Answer: 2 Lie on the unaffected side to instill your drops. Rationale: Lying on the unaffected side will facilitate the flow of drops into the ear. The client should lie on the unaffected side, not hold the head back. The client should lie on the unaffected side, not prop the head on a pillow after administration. The client should lie on the unaffected side, not sit up straight.

35.4 The nurse works in infection control at a large hospital. Which client does the nurse recognize as being at greatest risk of acquiring a fungal infection? 1. The client with anemia who is pregnant with triplets 2. The client with malignant melanoma who is receiving chemotherapy 3. The client with severe burns over 20% of the body 4. The adolescent client with a fractured femur from an auto accident

Answer: 2 The client with malignant melanoma who is receiving chemotherapy Rationale: Chemotherapy suppresses the immune system; clients with a suppressed immune system are at highest risk. The pregnant client with anemia is not at risk. The client with burns is at risk, but will not acquire the infection if infection control procedures are followed. The adolescent with a fractured femur is not at risk.

34.15 The physician orders cefepime (Maxipime) for a client. What is a priority question for the nurse to ask the client prior to administration of this drug? 1. "Are you pregnant?" 2. "Are you breast-feeding?" 3. "Are you allergic to penicillin?" 4. "Are you allergic to tetracycline?"

Answer: 3 "Are you allergic to penicillin?" Rationale: Cephalosporins are contraindicated in clients who have experienced a severe allergic reaction to penicillin. Cefepime (Maxipime) is a Pregnancy Category B drug, and is safe for use during pregnancy. Cefepime (Maxipime) is a Pregnancy Category B drug, and is safe to use while breast-feeding. Cephalosporins are contraindicated in clients who have experienced a severe allergic reaction to penicillin, not tetracycline.

35.14 The client receives oral nystatin (Mycostatin) suspension for an oral candidiasis infection. She tells the nurse she cannot continue to "swish and swallow" because her nausea is too great. What is the best response by the nurse? 1. "You can take a phenergan suppository before the nystatin (Mycostatin)." 2. "Try drinking a 7-Up after you swallow the medication." 3. "It is all right to swish the medication and then spit it out." 4. "I will ask your doctor if a pill form can be substituted."

Answer: 3 "It is all right to swish the medication and then spit it out." Rationale: If GI side effects are disturbing, the client may swish the medication and then spit it out. A phenergan suppository is not necessary. Drinking a 7-Up is not necessary. Substituting a tablet form is not necessary.

35.11 The female client has a fungal infection and will receive griseofulvin (Fulvicin). What assessment data is critical for the nurse to collect prior to administering this medication? 1. The client's height and weight 2. The type of diet the client is eating 3. The type of birth control the client is using 4. The amount of fat in the client's diet

Answer: 3 The type of birth control the client is using Rationale: Griseofulvin (Fulvicin) is a Pregnancy Category C drug, which means there isn't any data to indicate it is safe to take during pregnancy. The client's height and weight are not significant. The type of diet the client consumes is not significant.

36.15 The client receives delavirdine (Rescriptor) as treatment for acquired immune deficiency syndrome (AIDS). The nurse assesses the client for which serious adverse effect? 1. Bone marrow suppression 2. Cardiac arrest 3. Seizures 4. Rash

Answer: 4 Rash Rationale: A serious adverse effect of delavirdine (Rescriptor) is Stevens-Johnson rash, which can be life threatening. Stevens-Johnson rash, not bone marrow suppression, is an adverse effect of delavirdine (Rescriptor). Stevens-Johnson rash, not cardiac arrest, is an adverse effect of delavirdine (Rescriptor). Stevens-Johnson rash, not seizures, is an adverse effect of delavirdine (Rescriptor).

35.22 A client is receiving amphotericin B. The nurse will reinforce teaching by telling the client that he should watch for: a. Fever and chills. b. Headache. c. Constipation. d. Heartburn.

Answer: A Fever and chills. Rationale: A. Fever and chills are the important adverse effect to teach to clients taking amphotericin B. This could be related to an allergic response. B. Headache is not common. C. Constipation is not an adverse effect. D. Heartburn is not an adverse effect.

35.21 The mechanism of action of systemic antifungal agents is: a. Interfere with the synthesis of ergosterol in the fungal cell membrane. b. Bind to steroids in the fungal cell membrane. c. Bind to toxic levels of parasite in the red blood cells. d. Kill fungal growth.

Answer: A Interfere with the synthesis of ergosterol in the fungal cell membrane. Rationale: A. Systemic antifungal agents interfere with the synthesis of ergosterol in the fungal cell membrane, causing them to become permeable. B. Superficial fungal agents bind to sterols in the fungal cell membrane, allowing leakage of contents. C. Antiprotozoal drugs bind to toxic levels of parasite in the red blood cells. D. None of the these agents will kill the fungal growth.

35.20 The drug that most likely will be used for treatment of Trichomonas vaginalis is: a. Metronidazole (Flagyl). b. Praziquantel (Biltricide). c. Suramin (Germanin). d. Pyrimethamine (Daraprim).

Answer: A Metronidazole (Flagyl). Rationale: A. Metronidazole is the drug of choice for both giardiasis and trichomoniasis due to Trichomonas vaginalis. B. Praziquantel (Biltricide) is an anthelmintic effective against flatworms. C. Suramin (Germanin) is used for African sleeping sickness. D. Pyrimethamine (Daraprim) is an antimalarial drug.

36.16 The nurse is aware that the major structural components of viruses are: a. Intracellular bacteria. b. Extracellular bacteria. c. Intracellular parasites. d. Extracellular parasites.

Answer: C Intracellular parasites. Rationale: A. Viruses are not bacteria. B. Viruses are not bacteria. C. Viruses are intracellular parasites. They must be inside a host cell to cause infection. D. Viruses are not extracellular parasites.

The client has hypothyroidism and is treated with levothyroxine (Synthroid). The nurse plans to do medication education. What will the best plan of the nurse include?

Assess for altered sleep patterns

The client has been diagnosed with diabetes insipidus. What will the best plan of the nurse include?

Assess for increased urine production.

A child is prescribed methylphenidate (Ritalin) to treat attention deficit hyperactivity disorder (ADHD). The parent expresses concern about using a controlled substance to treat ADHD and asks the nurse about using a noncontrolled substance. The nurse knows ADHD can be treated with which noncontrolled substance? Methylphenidate (Concerta) Atomoxetine (Strattera) Amphetamine aspartate (Adderall) Dextroamphetamine sulfate (Dexedrine)

Atomoxetine (Strattera) Atomoxetine (Strattera) is not a controlled substance because it lacks drug addictive (psychological dependence) properties, unlike amphetamines and phenidates.

A patient has a viral sinus infection, and the provider tells the patient that antibiotics will not be prescribed. The patient wants to take an antibiotic and asks the nurse what possible harm could occur by taking an antibiotic. Which response by the nurse is correct? a. "Antibiotics are mutagenic and can produce changes that cause resistance." b. "Even normal flora can develop resistance and transfer this to pathogens." c. "Host cells become resistant to antibiotics with repeated use." d. "Patients who overuse antibiotics are more likely to have nosocomial infections."

B

The physician orders IV insulin for the client with a blood sugar of 563. The nurse administers lispro (humalog) intravenously (IV). What does the best evaluation by the nurse reveal? a. the nurse used the correct dose of insulin b. the nurse should have contacted the physician c. the nurse should have used regular insulin (Humilin R) d. the nurse could have given the insulin subcutaneously

B and C are correct. She originally had B as the correct answer but gave credit for C too

The nurse is admitting a client with hypoglycemia. Identify the signs and symptoms the nurse should expect. Select all that apply. A. Thirst B. Palpitations C. Diaphoresis D. Slurred speech E. Hyperventilation

B, C, D Palpitations, an adrenergic symptom, occur as the glucose levels fall; the sympathetic nervous system is activated and epinephrine and norepinephrine are secreted causing this response. Diaphoresis is a sympathetic nervous system response that occurs as epinephrine and norepinephrine are released. Slurred speech is a neuroglycopenic symptom; as the brain receives insufficient glucose, the activity of the CNS becomes depressed

Jansen receives metformin (Glucophage). What will the best plan of the nurse include with regard to patient education with this drug? Select all that apply. A It stimulates the pancreas to produce more insulin. B It must be taken with meals. C It decreases sugar production in the liver. D It inhibits absorption of carbohydrates. E It reduces insulin resistance

B, C, E Question 6 Explanation: Metformin (Glucophage) reduces insulin resistance, decreases sugar production in the liver, and should be taken with meals for the best absorption and effect. It does not stimulate the pancreas to produce more insulin and does not inhibit the absorption of carbohydrates.

Which adaptations should the nurse caring for a client with diabetic ketoacidosis expect the client to exhibit? Select all that apply: A. Sweating B. Low PCO2 C. Retinopathy D. Acetone breath E. Elevated serum bicarbonate

B, D Metabolic acidosis initiates respiratory compensation in the form of Kussmaul respirations to counteract the effects of ketone buildup, resulting in a lowered PCO2. A fruity odor to the breath (acetone breath) occurs when the ketone level is elevated in ketoacidosis.

A client diagnosed with human immune deficiency virus is prescribed zidovudine (Retrovir), efavirenz (Sustiva), lamivudine (Epivir), and enfuvirtide (Fuzeon). The client asks the nurse what will happen if the prescriptions are not refilled on time, or if a few doses of one of the medications are missed. What is the nurse's best response? A. "This will not make any difference in the viral load." B. "Blood concentrations will be decreased, which will lead to increased viral replication." C. "If only one dose of medication is missed, this will not make a difference." D. "This will cause an increase in opportunistic infections."

B. "Blood concentrations will be decreased, which will lead to increased viral replication." Rationale: When doses are missed, blood concentrations become lower than what is needed for inhibition of viral replication (often called the inhibitory concentration). When this concentration is too low, the organism can replicate and produce new organisms that are resistant to the drugs being used. Therefore, it is critical to ensure that highly active antiretroviral therapy (HAART) doses are not missed, delayed, or administered in lower-than-prescribed dosages in the inpatient setting. Teach clients the importance of taking their drugs exactly as prescribed to maintain the effectiveness of HAART.

A nurse is providing teaching to a client who has type 1 diabetes mellitus about exercise. Which of the following statements should the nurse include in the teaching? A. "You should exercise during a peak insulin time" B. "Wear a medical alert identification tag when you exercise" C. "Exercise can decrease the effects of insulin and cause the blood glucose levels to increase" D. "You will get the most benefit from exercise when your glucose levels are higher than normal"

B. "Wear a medical alert identification tag when you exercise" in case of a hypoglycemic event, exercise can potentiate the effects of insulin & cause the BG levels to decrease

A nurse is planning a community health screening for a group of clients who are at risk for type 2 diabetes mellitus. Which of the following clients should the nurse include in the screening? A. Men who smoke B. Men and women who are obese C. Women who have hepatitis D. Men and women who consume high-protein and low-carbohydrate foods

B. Men and women who are obese

The white blood cell count with differential of an undergoing preadmission testing before surgery indicates a total count of 10,000 cells per cubic millimeter (mm3) of blood. Which of the follow differential counts or percentages does the nurse report to the physician? A. Eosinophils 200/mm3 B. Monocytes 2000/mm3 C. Segmented neutrophils 5700/mm3 D. Lymphocytes 2100/mm3

B. Monocytes 2000/mm3 Rationale: The total WBC count is in the normal range. However, the normal monocyte population in peripheral blood should not be greater than 5%. A monocyte count of 2000 in 10,000 WBCs represents 20% of the total and indicates a significant increase. A common cause of this abnormality is mononucleosis.

Nurse Oliver should expect a client with hypothyroidism to report which health concerns? A. Increased appetite and weight loss B. Puffiness of the face and hands C. Nervousness and tremors D. Thyroid gland swelling

B. Puffiness of the face and hands Hypothyroidism (myxedema) causes facial puffiness, extremity edema, and weight gain. Signs and symptoms of hyperthyroidism (Graves' disease) include an increased appetite, weight loss, nervousness, tremors, and thyroid gland enlargement (goiter).

Albert refuses his bedtime snack. This should alert the nurse to assess for: A. Elevated serum bicarbonate and a decreased blood pH. B. Signs of hypoglycemia earlier than expected. C. Symptoms of hyperglycemia during the peak time of NPH insulin. D. Sugar in the urine

B. Signs of hypoglycemia earlier than expected.

A client is admitted to the hospital with suspected Goodpasture's syndrome. Which findings does the nurse expect to observe? A. Bradycardia B.Hemoptysis C. Increased urine output D. Weight loss

B.Hemoptysis Rationale: Hemoptysis (bloody sputum) is a manifestation of Goodpasture's syndrome. Goodpasture's syndrome usually is not diagnosed until serious lung and kidney problems are present. Tachycardia, decreased urine output, and weight gain are manifestations of Goodpasture's syndrome.

Insulin is released when:

Blood glucose increases.

Which of the following is a sign or symptom of hypothyroidism?

Bradycardia

A child has received amoxicillin [Amoxil] for three previous ear infections, but a current otitis media episode is not responding to treatment. The nurse caring for this child suspects that resistance to the bacterial agent has occurred by which microbial mechanism? a. Alteration of drug target molecules b. Antagonist production c. Drug inactivation d. Reduction of drug concentration at the site of action

C

A nursing student asks a nurse to clarify the differences between the mechanisms of spontaneous mutation and conjugation in acquired resistance of microbes. What will the nurse say? a. Conjugation results in a gradual increase in resistance. b. Conjugation results in random changes in the microbe's DNA. c. Spontaneous mutation leads to resistance to only one antimicrobial agent. d. Spontaneous mutation can transfer DNA from one organism to another.

C

A diabetic patient has the following presentation: unresponsive to voice or touch, tachycardia, diaphoresis, and pallor. Which of the following actions by the healthcare provider is the priority? Please choose from one of the following options. Send blood to the laboratory for analysis Administer oxygen per nasal cannula Administer 50% dextrose IV per protocol Administer the prescribed insulin

C Administer 50% dextrose IV per protocol Rationale: 11 / 3 Medical emergencies that occur with diabetic patients are often due to either hyperglycemia or hypoglycemia. Hint #22 / 3 Recall the body's response to an increased or decreased blood glucose level. Hint #33 / 3 The body responds to hypoglycemia by mounting a sympathetic response, causing tachycardia, diaphoresis, and pallor. The patient may become unresponsive as the glucose supply to the brain decreases.

A clinical instructor teaches a class for the public about diabetes mellitus. Which individual does the nurse assess as being at highest risk for developing diabetes? A The 50-year-old client who does not get any physical exercise B The 56-year-old client who drinks three glasses of wine each evening C The 42-year-old client who is 50 pounds overweight D The 38-year-old client who smokes one pack of cigarettes per day

C. Question 4 Explanation: Obesity increases the likelihood of developing diabetes mellitus due to over stimulation of the endocrine system. Exercise is important, but lack of exercise is not as big a risk factor as obesity. Smoking is a serious health concern but is not a specific risk factor for diabetes. Consuming alcohol is associated with liver disease but is not as high a risk factor for diabetes as obesity. Question 5 WRONG

The nurse is teaching a client about cyclosporine (Sandimmune) therapy after liver transplantation. Which client statement indicates the need for further teaching? A. "I will be on this medicine for the rest of my life." B. "I must undergo regular kidney function tests." C. "I must regularly monitor my blood sugar." D. "My gums may become swollen because of this drug."

C. "I must regularly monitor my blood sugar." Rationale: Blood sugar is not affected by taking cyclosporine, so the client has no need to monitor blood sugar. The client must take cyclosporine for the rest of his or her life. Kidney dysfunction is a side effect of cyclosporine, so regular monitoring is required. Swollen gums are a side effect of taking cyclosporine.

The lowest fasting plasma glucose level suggestive of a diagnosis of DM is: A. 90mg/dl B. 115mg/dl C. 126mg/dl D. 180mg/dl

C. 126mg/dl

The client has hypothyroidism and is treated with levothyroxine (Synthroid). The nurse plans to do medication education. What will the best plan by the nurse include? Select one: A. Assess for decreased appetite. B. Assess weekly serum blood levels. C. Assess for altered sleep patterns. D. Monitor daily temperature

C. Assess for altered sleep patterns.

Nurse Kristine is trying to communicate with a client with brain attack (stroke) and aphasia. Which of the following actions by the nurse would be least helpful to the client? A. Speaking to the client at a slower rate B. Allowing plenty of time for the client to respond C. Completing the sentences that the client cannot finish D. Looking directly at the client during attempts at speech

C. Completing the sentences that the client cannot finish Clients with aphasia after brain attack (stroke) often fatigue easily and have a short attention span. General guidelines when trying to communicate with the aphasic client include speaking more slowly and allowing adequate response time, listening to and watching attempts to communicate, and trying to put the client at ease with a caring and understanding manner. The nurse would avoid shouting (because the client is not deaf), appearing rushed for a response, and letting family members provide all the responses for the client.

A nurse is assessing a client who has Graves' disease. Which of the following findings should the nurse expect the client to display? A. Constipation B. Cold intolerance C. Difficulty sleeping D. Anorexia

C. Difficulty sleeping A client who has Graves' disease can have difficulty sleeping and anxiety due to the overproduction of thyroid hormone.

Which antibiotic is sometimes considered a "last chance" drug for those with resistant infections? A. Clarithromycin B. Dicloxacillin C. Vancomycin (Vancocin) D. Trimethoprin-sulfamethoxazole (Bactrim, Septra)

C. Vancomycin (Vancocin)

What body system will the nurse assess for known common adverse effects of traditional antihistamines? a. respiratory b. gastroinintestinal c. central nervous d. cardiovascular

C. central nervous The most common adverse effect of antihistamines is drowsiness, but the sedative effects vary among antihistamine drug classes.

A patient diagnosed with narcolepsy is prescribed a central nervous system (CNS) stimulant. Which statement best describes the action of CNS stimulants? CNS stimulants activate cyclic adenosine monophosphate. CNS stimulants block or reduce the activity of inhibitory neurons. CNS stimulants increase release of and block reuptake of neurotransmitters. CNS stimulants decrease the production of excitatory neurotransmitters.

CNS stimulants increase release of and block reuptake of neurotransmitters. CNS stimulation occurs when the amount of neurotransmitters being released and the duration of action of excitatory neurotransmitters are increased.

The nurse explains to a patient that using caffeine may exacerbate which health condition? Cardiac dysrhythmias Constipation Heart block Myelin degeneration

Cardiac dysrhythmias Caffeine is a CNS stimulant. It should be used with caution in patients with a history of peptic ulcers, cardiac dysrhythmia, or recent myocardial infarction.

A client who is allergic to penicillin is at increased risk for an allergy to which drug? a. Erythromycin (E-mycin) b. Gentamicin (Garamycin) c. Cefazolin sodium (Ancef) d. Demeclocycline (Declomycin)

Cefazolin sodium (Ancef) Clients who are allergic to penicillins have an increased risk of allergy to other beta-lactam antibiotics. The incidence of cross-reactivity between cephalosporins and penicillins is reported to be between 1% and 4%.

What is the MOST important action for the nurse to complete before administration of intravenous (IV) amphotericin B? Monitor for cardiac dysrhythmias. Assess for nausea and vomiting. Monitor IV site for signs of phlebitis. Check for premedication prescriptions. Submit

Check for premedication prescriptions. Almost all clients given IV amphotericin B experience fever, chills, hypotension, tachycardia, malaise, muscle and joint pain, anorexia, nausea and vomiting, and headache. Pretreatment with an antipyretic, antihistamine, and antiemetic can minimize or prevent these adverse reactions. The other choices are appropriate nursing actions after the IV infusion has begun.

Vibro

Cholera

To assess for the dose-limiting toxicity of ganciclovir (Cytovene), the nurse will monitor which laboratory test result? Creatine phosphokinase Liver function tests Blood urea nitrogen Complete blood count (CBC)

Complete blood count (CBC) Bone marrow suppression is a dose-limiting toxicity of ganciclovir; therefore, the CBC should be monitored.

The function of the thyroid gland is to:

Control basal metabolism.

The client has been diagnosed with diabetes insipidus. What will the best plan by the nurse include? 1. Assess for increased urine production. 2. Assess for hyperglycemia. 3. Assess for hyponatremia. 4. Assess for fluid retention.

Correct Answer: 1 Rationale 1: Diabetes insipidus results from decreased ADH (antidiuretic hormone) production, so the client will have increased urine output. Rationale 2: Hyperglycemia is not an effect of diabetes insipidus. Rationale 3: Hypernatremia results from the volume of fluid that is lost; the client will not have hyponatremia. Rationale 4: The client will have increased urine output and fluid volume depletion, not retention.

The client has hyperthyroidism. The nurse teaches the client to avoid which food selections? 1. Soy sauce 2. Milk products 3. High-calorie foods 4. Caffeine-free soda

Correct Answer: 1 Rationale 1: Foods high in iodine, such as soy sauce, can affect the effectiveness of medication therapy for clients who are diagnosed with hyperthyroidism. Rationale 2: Milk products should be included in the diet for the client with hyperthyroidism because they are high in protein and calcium. Rationale 3: High-calorie foods are important for clients with hyperthyroidism in order to meet metabolic demands. Rationale 4: There is no reason to restrict caffeine-free soda.

Which laboratory tests will be performed to determine whether a specific bacterium is resistant to a specific drug? 1. Culture and sensitivity test 2. Complete blood count 3. Blood urea nitrogen 4. Urinalysis

Correct Answer: 1 Rationale 1: Culture and sensitivity is the examination for a specific organism and can determine the correct medication. Rationale 2: Complete blood count would not determine the specific drug for the specific organism. Rationale 3: Blood urea nitrogen would not determine the specific drug for the specific organism. Rationale 4: Urinalysis would not determine the specific drug for the specific organism.

Following surgery, a client is placed on cefotaxime (Claforan). The assessment for possible adverse effects should include observing for 1. diarrhea. 2. headache. 3. constipation. 4. tachycardia.

Correct Answer: 1 Rationale 1: Diarrhea is a frequent adverse effect of cephalosporins. Rationale 2: Headache is not an adverse effect. Rationale 3: Diarrhea, not constipation, is a common problem. Rationale 4: Tachycardia is not an adverse effect.

The client tells the nurse that the doctor told him his antibiotic did not kill his infection but just slowed its growth. The client is anxious. What is the best response by the nurse to decrease the client's anxiety? 1. "This is okay because your body will help kill the infection too." 2. "This is okay because your doctor is an infectious disease specialist." 3. "This is okay because your blood work is being monitored daily." 4. "This is okay because your infection is not really that serious."

Correct Answer: 1 Rationale 1: Some drugs do not kill the bacteria but instead slow their growth and depend on the body's natural defenses to dispose of the microorganisms. These drugs, which slow the growth of bacteria, are called bacteriostatic. Rationale 2: Telling the client that the doctor is a specialist does not answer the question and will increase anxiety. Rationale 3: Telling the client that his blood work is being monitored does not answer the question and will increase anxiety. Rationale 4: Telling a client with an infection that the infection is not serious will increase anxiety because, to the client, all infections are serious.

A client will be prescribed an oral corticosteroid for treatment of a rash. The prescriber says, In order to avoid adverse effects, I am prescribing the smallest amount possible. What other prescribing regimens could the nurse suggest to achieve this same effect? Select all that apply. 1. Give the oral medication every other day. 2. Administer a larger dose for a few days and then gradually decrease the dose until the medication is discontinued. 3. Use a topical application instead of an oral product. 4. Start with a small dose and work up to the desired dose. 5. Administer the doses intramuscularly for the first few days of treatment.

Correct Answer: 1,2,3 Rationale 1: Alternate day administration can limit adrenal atrophy. Rationale 2: There are specific formulations of corticosteroids that are packaged for use of a large dose initially which is then tapered off until all the medication is gone. Rationale 3: Topical application does not cause as many systemic effects as oral dosage. Rationale 4: Corticosteroid doses are generally started high and worked down to discontinuance. Rationale 5: Using a less systemic approach would lessen effects.

A client diagnosed with an inoperable tumor of the adrenal gland is prescribed mitotane (Lysodren). The nurse would attribute which assessment finding as indicating an adverse reaction to the drug? Select all that apply. 1. The client reports that food is not appealing. 2. The client reports joint pain. 3. The client reports ringing of the ears. 4. The client reports increasing depression. 5. The client reports dizziness.

Correct Answer: 1,4,5 Rationale 1: About 80% of clients started on this drug will report anorexia. Rationale 2: Joint pain is not an expected adverse effect of this drug. Rationale 3: This is not an expected effect of this drug. Rationale 4: About 40% of people taking this drug will experience depression. Rationale 5: About 40% of the people taking this drug report dizziness.

Insulin is released when 1. blood glucose stays the same. 2. blood glucose increases. 3. blood glucose decreases. 4. glucagon increases

Correct Answer: 2 Rationale 1: Insulin would not be released. Rationale 2: Insulin is released when blood glucose increases. Rationale 3: Glucagon is released when glucose decreases. Rationale 4: Glucagon increases when insulin is not needed.

The nurse provides care for clients with acquired immune deficiency syndrome (AIDS). Which laboratory test is the best indicator of effective treatment with antiviral medications? 1. CD4 count 2. Viral load 3. T4 lymphocyte count 4. Absolute neutrophil count

Correct Answer: 2 Rationale 1: The viral load, not a CD4 count, is the best test. Rationale 2: The viral load is an actual count of viral presence and is the best of these tests. Rationale 3: The viral load, not a T4 lymphocyte count, is the best test. Rationale 4: The viral load, not an absolute neutrophil count, is the best test.

A patient has recurrent skin infections. The nurse anticipates administering an aminopenicillin such as which drug? Select all that apply. 1. Oxacillin 2. Ampicillin 3. Piperacillin 4. Amoxicillin 5. Dicloxacillin

Correct Answer: 2,4 Rationale 1: Oxacillin is a penicillinase-resistant penicillin. Rationale 2: Ampicillin is a broad-spectrum or aminopenicillin. Rationale 3: Piperacillin is an extended-spectrum penicillin. Rationale 4: Amoxicillin is a broad-spectrum or aminopenicillin. Rationale 5: Dicloxacillin is a penicillinase-resistant penicillin.

The primary goal of pharmacotherapy in hyperthyroidism (Graves disease) is to: 1. increase metabolism. 2. increase synthesis of corticosteroid. 3. lower the activity of the thyroid hormone. 4. increase the activity of the thyroid hormone.

Correct Answer: 3 Rationale 1: Increased metabolism is a sign of hyperthyroidism. Rationale 2: Adrenal drugs increase synthesis of corticosteroids. Rationale 3: The goal is to lower the activity of the thyroid hormone. Rationale 4: Increased activity of the thyroid hormone is a physiological mechanism of hyperthyroidism.

The physician orders intravenous (IV) insulin for the client with a blood sugar of 563. The nurse administers insulin lispro (Humalog) intravenously (IV). What does the best evaluation by the nurse reveal? 1. The nurse used the correct insulin. 2. The nurse should have contacted the physician. 3. The nurse should have used regular insulin (Humulin R). 4. The nurse could have given the insulin subcutaneously

Correct Answer: 3 Rationale 1: The nurse did not use the correct insulin as it was not regular insulin. Rationale 2: There was no need to contact the physician; regular insulin is the only insulin that can be given intravenously (IV). The physician should be contacted now. Rationale 3: Regular insulin is the only insulin that can be given intravenously (IV). Rationale 4: The nurse cannot give the insulin subcutaneously when it is ordered to be given intravenously (IV).

The function of the thyroid gland is to 1. stimulate growth. 2. control pituitary gland secretion. 3. conserve water in the body. 4. control basal metabolism.

Correct Answer: 4 Rationale 1: Growth hormone stimulates growth. Rationale 2: The thyroid gland does not control the pituitary. Rationale 3: Antidiuretic hormone conserves water in the body. Rationale 4: The function of the thyroid gland is to control basal metabolism and affect every cell in the body.

The client has hypothyroidism and is treated with levothyroxine (Synthroid). The nurse plans to do medication education. What will the best plan by the nurse include? 1. Monitor daily weights. 2. Assess for decreased appetite. 3. Assess weekly serum blood levels. 4. Assess for altered sleep patterns.

Correct Answer: 4 Rationale 1: Weights can be monitored on a weekly, not daily, basis with clients receiving levothyroxine (Synthroid).. Rationale 2: The appetite tends to increase, not decrease, with clients receiving levothyroxine (Synthroid). Rationale 3: Serum blood levels are not required on a weekly basis with clients receiving levothyroxine (Synthroid). Rationale 4: Insomnia is an adverse effect of levothyroxine (Synthroid); altered sleep patterns must be assessed.

The nurse is caring for a client receiving gentamicin IV. The nurse would observe for adverse effects of 1. diarrhea. 2. bleeding 3. increased urinary output. 4. ototoxicity.

Correct Answer: 4 Rationale 1: Diarrhea is not a common adverse effect of gentamicin. Rationale 2: Bleeding is not a common adverse effect. Rationale 3: Increased urinary output is not an adverse effect. Rationale 4: Ototoxicity is an adverse effect that could occur while receiving gentamicin. This could become permanent with continued use.

The client has diabetes mellitus type 2. The nurse has taught the client about the illness and evaluates that learning has occurred when the client makes which statement? 1. My beta cells just cannot produce enough insulin for my cells. 2. My peripheral cells have increased sensitivity to insulin. 3. My cells have increased their receptors, but there is not enough insulin. 4. My cells cannot use the insulin my pancreas makes.

Correct Answer: 4 Rationale 1: The beta cells continue to produce insulin with type 2 diabetes. Rationale 2: Peripheral cells have a decreased, not an increased, sensitivity to insulin. Rationale 3: There is a decrease, not an increase, in receptor sites with type 2 diabetes. Rationale 4: With type 2 diabetes mellitus, the pancreas produces insulin, but the cells cannot use it.

A parent asks a nurse if the provider will prescribe an antibiotic for a child who attends school with several children who have strep throat. The child is complaining of a sore throat and has a fever. What will be the nurse's response? a. "Because strep throat is likely, your child should be treated empirically." b. "With good hand washing, your child should not get strep throat." c. "Your child probably has strep throat, so your provider will order an antibiotic." d. "Your child should come to the clinic to have a throat culture done today."

D

A patient has a localized skin infection, which is most likely caused by a gram-positive cocci. Until the culture and sensitivity results are available, the nurse will expect the provider to order a ____-spectrum ____ agent. a. broad; systemic b. broad; topical c. narrow; systemic d. narrow; topical

D

The nurse has been caring for a patient who has been taking antibiotics for 3 weeks. Upon assessing the patient, the nurse notices the individual has developed oral thrush. What describes the etiology of the thrush? a. Antibiotic resistance b. Community-acquired infection c. Nosocomial infection d. Superinfection

D

During the morning rounds, Nurse AJ accompanied the physician in every patient's room. The physician writes orders for the client with diabetes mellitus. Which order would the nurse validate with the physician? A Use Humalog insulin for sliding scale coverage. B Metformin (Glucophage) 1000 mg per day in divided doses. C Administer regular insulin 30 minutes prior to meals. D Lantus insulin 20U BID

D Question 14 Explanation: Lantus insulin is usually prescribed once-a-day so an order for BID dosing should be validated with the physician. Humalog insulin can be prescribed for sliding scale coverage. Regular insulin is administered 30 minutes before meals. Metformin (Glucophage) is often prescribed in divided doses of 1000 mg per day.

Daniel is diagnosed of having hyperthyroidism (Graves' disease). Which of the following is a drug of choice for his condition? A Furosemide (Lasix) B Digoxin (Lanoxin) C Propranolol (Inderal) D Propylthiouracil (PTU)

D Question 27 Explanation: Propylthiouracil (PTU) initially is given in divided doses, and functions to block thyroid hormone synthesis.

Which home health nurse should the nurse manager assign to care for an 18-year-old client with a kidney transplant who has many questions about the prescribed cyclosporine (Sandimmune)? A. RN who has worked for the home health agency for 5 years in maternal-child health B. RN who has extensive critical care nursing experience and has worked in home health for a year C. RN who transferred to the home health agency after working for 10 years in an outpatient dialysis unit D RN who worked for 5 years in an organ transplant unit and has recently been hired by the home health agency

D RN who worked for 5 years in an organ transplant unit and has recently been hired by the home health agency Rationale: The RN who worked for 5 years in an organ transplant unit and has recently been hired by the home health agency has the experience and understanding of the needs of a post-transplantation client, as well as knowledge of cyclosporine, and would be the best choice. An RN who has worked for the home health agency for 5 years in maternal-child health, an RN who has extensive critical care nursing experience and has worked in home health for a year, and an RN who transferred to the home health agency after working for 10 years in an outpatient dialysis unit would not have knowledge and information on the care provided and medications used in post-transplantation clients.

Ben injects his insulin as prescribed, but then gets busy and forgets to eat. What will the best assessment of the nurse reveal? A The client will be very thirsty. B The client will complain of nausea. C The client will need to urinate. D The client will have moist skin.

D. Question 3 Explanation: Moist skin is the sign of hypoglycemia, which the client would experience if he injected himself with insulin and did not eat. Thirst, nausea, and increased urination are signs of hyperglycemia.

During preoperative teaching for a female client who will undergo subtotal thyroidectomy, the nurse should include which statement? A. "The head of your bed must remain flat for 24 hours after surgery." B. "You should avoid deep breathing and coughing after surgery." C. "You won't be able to swallow for the first day or two." D. "You must avoid hyperextending your neck after surgery."

D. "You must avoid hyperextending your neck after surgery." To prevent undue pressure on the surgical incision after subtotal thyroidectomy, the nurse should advise the client to avoid hyperextending the neck. The client may elevate the head of the bed as desired and should perform deep breathing and coughing to help prevent pneumonia. Subtotal thyroidectomy doesn't affect swallowing.

The healthcare provider administers NPH insulin to a patient who has diabetes at 6:00 AM. When will the patient be at highest risk of experiencing hypoglycemia? Please choose from one of the following options. 7:00 AM 7:30 AM 9:00 AM 10:00 AM

D. 10:00 AM Rationale: 1 / 3 NPH insulin is an intermediate acting insulin which is usually administered once or twice daily. Hint #22 / 3 A patient is most likely be become hypoglycemic when the serum insulin level begins to peak. Hint #33 / 3 The peak effect of NPH insulin is at 4 -12 hours, so the healthcare provider will monitor the patient for hypoglycemia beginning at 10:00 AM.

With which client is it most important for the nurse to use latex-free gloves? A. 38-year-old woman taking oral contraceptives B. 68-year-old man with total hip replacement C. 38-year-old man allergic to shellfish and nuts D. 28-year-old woman with spina bifida

D. 28-year-old woman with spina bifida Rationale: People who have spina bifida have lifelong exposure to latex products and frequently develop latex hypersensitivities. Such people are at an increased risk for an anaphylactic reaction when they have major surgery, especially abdominal, and the surgeons are using latex gloves when entering the abdominal cavity. However, any further exposure to latex can increase the risk, and only latex-free products should be used for these clients.

A complete blood count with differential is performed in a client with chronic sinusitis. Which finding does the nurse expect? A. Segmented neutrophils, 62% B.Lymphocytes, 28% C. Bands, 5% D. Basophils, 4%

D. Basophils, 4% Rationale: The normal count for basophils (basos) is 0.5%; an elevated count indicates inflammation, which is common with chronic sinusitis. Segmented neutrophils (segs) are mature neutrophils, which, along with macrophages, eliminate invaders (infection) by phagocytosis; 62% is a normal neutrophil count. For lymphocytes (lymphs), 28% is a normal count in the differential. For bands, 4% is a normal count; bands are elevated only when an infection is present and the bone marrow cannot keep up with mature segmented neutrophils.

A nurse is reviewing the laboratory values of a client who has diabetic ketoacidosis. The nurse should understand that which of the following laboratory values is consistent with diabetic ketoacidosis? A. Blood glucose 30 mg/dL B. Negative urine ketones C. Blood pH 7.38 D. Bicarbonate level 12 mEq/L

D. Bicarbonate level 12 mEq/L Clients with DKA should have a bicarb level of less than 15 mEq/L b/c of an increased production of regulatory hormones that lead to metabolic acidosis

The nurse prepares to administer zafirlukast (Accolate) to a client with allergic rhinitis. Zafirlukast works by which mechanism? A. Blocking histamine from binding to receptors B. Preventing synthesis of mediators C. Preventing mast cell membranes from opening D. Blocking the leukotriene receptor

D. Blocking the leukotriene receptor Rationale: Zafirlukast is a leukotriene antagonist that prevents the occurrence of allergic rhinitis by blocking the leukotriene receptor. Antihistamines such as diphenhydramine (Benadryl) block histamines from binding to receptors; zafirlukast is not an antihistamine. Corticosteroids prevent synthesis of mediators; zafirlukast is not a corticosteroid. Mast cell-stabilizing drugs such as cromolyn sodium (Nasalcrom) prevent mast cell membranes from opening when an allergen binds to immunoglobulin E; zafirlukast is not a mast cell-stabilizing drug.

A client has been prescribed oseltamivir (Tamiflu) after complaining of influenza-like symptoms. What information should the nurse provide for this client? Select one or more: A. This medication is given by inhalation. B. This medication will keep you from getting the flu. C. This medication will be helpful if you have influenza or a cold. D. Get this prescription filled and begin taking the medication immediately.

D. Get this prescription filled and begin taking the medication immediately.

A nurse is monitoring a client who has Graves' disease for the development of thyroid storm. the nurse should report which of the following findings to the provider? A. Constipation B. Headache C. Bradycardia D. Hypertension

D. Hypertension

The nurse is reviewing the medical record of a client who is prescribed a decongestant. The nurse plans to contact the client's health care provider if the client has which condition? A. Cataracts B. Crohn's disease C.Diabetes mellitus D. Hypertension

D. Hypertension Rationale: Decongestants have actions similar to adrenergic drugs, causing vasoconstriction, which can increase blood pressure. Decongestants are not contraindicated in clients with cataracts, Crohn's disease, or diabetes mellitus.

The drug that would most likely be used in the treatment of tuberculosis is Select one: A. Erythromycin (E-mycin). B. Vancomycin (Vancocin). C. Gentamicin (Garamycin). D. Isoniazid (INH).

D. Isoniazid (INH).

A client comes to the emergency department with a fever of 104°F. The nurse anticipates which actions to help identify the correct antibiotic? Select one: A. Obtaining a complete blood count (CBC) test. B. Obtaining liver and renal function tests. C. Obtaining a sterile urine specimen. D. Obtaining blood for culture and sensitivity.

D. Obtaining blood for culture and sensitivity.

Which nursing action is most appropriate for the nurse working in an allergy clinic to delegate to a nursing assistant? A. Plan the schedule for desensitization therapy for a client with allergies. B. Monitor the client who has just received skin testing for signs of anaphylaxis. C. Educate a client with a latex allergy about other substances with cross-sensitivity to latex. D. Remind the client to stay at the clinic for 30 minutes after receiving intradermal allergy testing.

D. Remind the client to stay at the clinic for 30 minutes after receiving intradermal allergy testing. Rationale: Reminding a client about safety policies is within the scope of practice of a nursing assistant. Planning care and assessing for complications require broader education and scope of practice and should be done by the registered nurse. Client education is a registered nursing responsibility, which requires broader education and scope of practice.

What is the primary function of the islets of Langerhans in the pancreas? Select one: A. Secretion of enzymes B. Absorption of insulin C. Acting as exocrine D. Secretion of glucagon and insulin

D. Secretion of glucagon and insulin

The nurse should teach patients that photosensitivity and teeth discoloration are potential adverse effects of: A. Aminoglycosides B. Metronidazole (Flagyl) C. Cephalosporins D. Tetracyclines

D. Tetracylines

Which of the following instruments is used to record intraocular pressure? A. Goniometer B. Ophthalmoscope C. Slit lamp D. Tonometer

D. Tonometer A tonometer is a device used in glaucoma screening to record intraocular pressure. A goniometer measures joint movement and angles. An ophthalmoscope examines the interior of the eye. especially the retina. A slit lamp evaluates structures in the anterior chamber in the eye.

What is the most important question the nurse asks the patient prescribed to begin highly active antiretroviral therapy? A. Do you have any symptoms now of active infection? B. Is there any possibility that you are pregnant? C. Are your currently sexually active? D. What other medications do you take?

D. What other medications do you take? Rationale: Before giving an antiretroviral drug, obtain a list of all other drugs the patient also takes because antiretroviral drugs interact with many other drugs. Check with the pharmacist for possible interactions and the need to consult the prescriber about dosage or changing the patient's other drug with the prescriber.

The client with a history of asthma is prescribed a leukotriene receptor antagonist to prevent allergic rhinitis. The nurse anticipates that which drug will be prescribed? A. Cromolyn sodium (Nasalcrom) B. Desloratadine (Clarinex) C. Fexofenadine (Allegra) D. Zafirlukast (Accolate)

D. Zafirlukast (Accolate) Rationale: Zafirlukast (Accolate) is a leukotriene receptor antagonist; it works by blocking the leukotriene receptor and is used to prevent allergic rhinitis. Cromolyn sodium (Nasalcrom) is a mast cell-stabilizing drug. Desloratadine (Clarinex) and fexofenadine (Allegra) are nonsedating antihistamines.

A male client with a tentative diagnosis of hyperosmolar hyperglycemic nonketotic syndrome (HHNS) has a history of type 2 diabetes that is being controlled with an oral diabetic agent, tolazamide (Tolinase). Which of the following is the most important laboratory test for confirming this disorder? A. Serum potassium level B. Serum sodium level C. Arterial blood gas (ABG) values D. Serum osmolarity

D. serum osmolarity Serum osmolarity is the most important test for confirming HHNS; it's also used to guide treatment strategies and determine evaluation criteria. A client with HHNS typically has a serum osmolarity of more than 350 mOsm/L. Serum potassium, serum sodium, and ABG values are also measured, but they aren't as important as serum osmolarity for confirming a diagnosis of HHNS. A client with HHNS typically has hypernatremia and osmotic diuresis. ABG values reveal acidosis, and the potassium level is variable.

Vasopressin is used in the treatment of:

Diabetes insipidus.

When planning care for a client receiving a sulfonamide antibiotic, it is important for the nurse to perform which intervention? Take the medication with dairy products such as milk or yogurt. Advise the client to report any tinnitus to the health care provider Avoid direct sun exposure and tanning beds. Encourage fluid intake of 2000 to 3000 mL/day.

Encourage fluid intake of 2000 to 3000 mL/day. Clients should be encouraged to drink plenty of fluids (2000 to 3000 mL/24 hours) to prevent drug-related crystalluria associated with sulfonamide antibiotics.

Macrolide

Erythromycin (E-mycin, Erthrocin)

Acquired resistance develops from taking too high a dose of antibiotics and is less likely when smaller doses are administered over a long period?

False

Macrolides are an effective antibiotic class; however, patients must take these drugs for a longer period of time due to their relatively short half-life?

False

Ototoxicity is a common adverse effect of erythromycin and clarithromycin (Biaxin)?

False

Which sign or symptom is most typical of an untreated client with type 1 diabetes?

Fatigue is a typical sign/symptom of type 1 DM due to sustained hyperglycemia.

In comparing and contrasting the signs and symptoms typical of type 1 diabetes mellitus (DM) with sustained hyperglycemia, a nurse can expect to observe for:

Fatigue.

A patient receiving valproic acid (Depakote) should be monitored for which adverse effects? (Select all that apply.) Select all that apply. Tremors Insomnia Hepatoxicity Weight gain Hypoglycemia

Flush the line with normal saline before and after administration to prevent precipitation. Phenytoin is very irritating to veins and incompatible with all fluids except normal saline. Flushing with normal saline before and after minimizes precipitation. You do not need an infusion pump when administering via IV push, and administration via central lines is preferred.

The nurse instructs a patient receiving phenytoin (Dilantin) to visit the dentist regularly and perform frequent oral hygiene. What common adverse effect is the nurse educating the patient about for this medication? Oral candidiasis Increased incidence of dental caries Increased risk of tooth abscess Gingival hyperplasia

Gingival hyperplasia A well-known adverse effect of long-term oral phenytoin therapy is overgrowth of gum tissue, or gingival hyperplasia. This can be minimized by frequent oral hygiene.

A nurse is reviewing the blood work of a client who has recently begun treatment for type 2 diabetes. Which results would indicate that the client is on target with therapy?

HBA1C level is 6.3%. Fasting blood glucose is 100 g/dL.

The nurse makes a home visit to the client with diabetes mellitus. During the visit, the nurse notes that the client's additional insulin vials are not refrigerated. What is the best action by the nurse at this time?

Have the client place the insulin vials in the refrigerator.

A new vaccination, Zostavax (Zoster Vaccine Live), has been approved to prevent the development of what condition in adults older than the age of 60 years? Herpes zoster Influenza A Herpes simplex Avian influenza

Herpes zoster Zoster vaccine (Zostavax) is a vaccine for the prevention of herpes zoster. Herpes zoster, also known as shingles, is an extremely painful condition caused by the varicella-zoster virus that also causes chickenpox. The vaccine is approved for clients 60 years of age or older to prevent reactivation of the zoster virus that causes shingles, although clients as young as 50 years may receive it. Zostavax is a one-time vaccine.

The client receives hydrocortisone therapy. The nurse will primarily assess for which electrolyte disturbance?

Hypernatremia and hyperglycemia

The client receives glucocorticoid therapy. What will the best assessment of the nurse reveal?

Hypertension

The nurse has finished teaching a client with diabetes mellitus how to administer insulin. The nurse evaluates that learning has occurred when the client makes which statement?

I should only use a calibrated insulin syringe for the injections."

The nurse has provided education to a client about fungal skin infections. Further client teaching is necessary when the client includes which condition in the discussion of fungal skin infections? Impetigo Vaginal yeast infection Athlete's foot Thrush

Impetigo Impetigo is a bacterial skin infection and would not be classified as a fungal skin infection. If the client included this in the discussion, further teaching is needed. All other skin infections listed are fungal and would be treated with antifungal medications.

In caring for a client with type 2 diabetes mellitus (DM), the nurse will teach the client to report:

Increased blood pressure and pulse.

A patient with narcolepsy is prescribed methylphenidate (Ritalin). Which adverse effects should the nurse include in the teaching of this drug? (Select all that apply.) Select all that apply. Insomnia Headache Weight Loss Decreased blood pressure Increased appetite

Insomnia, Headache, and Weight loss The adverse effects of methylphenidate on the cardiovascular system include increased heart rate and blood pressure. Other adverse effects include angina, anxiety, insomnia, headache, tremor, blurred vision, increased metabolic rate, GI distress, dry mouth, and worsening of or new onset of psychiatric disorders (including mania, psychoses, or aggression).

The client receives metformin Glucophage What will the best plan by the nurse include with regard to patient education with this drug?

It decreases sugar production in the liver. It reduces insulin resistance.

The physician writes orders for the client with diabetes mellitus. Which order would the nurse validate with the physician?

Lantus insulin 20U BID.

The physician writes orders for the client with diabetes mellitus. Which order would the nurse validate with the physician?

Lantus insulin is usually prescribed in once-a-day dosing so an order for BID dosing should be validated with the physician.

The primary goal of pharmacotherapy in hyperthyroidism (Graves' disease) is to:

Lower the activity of the thyroid hormone.

Borrelia

Lyme disease

While teaching a patient newly diagnosed with a seizure disorder, what does the nurse state as the goal of pharmacologic therapy of this medication? Eradicate all seizure activity and discontinue prescribed medication after the patient is seizure free for 3 months. Maximize drug dosages to control seizure activity. Reduce seizure occurrence to one per week. Maximally reducing seizure activity while minimizing the adverse effects of medication therapy.

Maximally reducing seizure activity while minimizing the adverse effects of medication therapy. Anticonvulsant medications are used to prevent the convulsive seizures typically associated with epilepsy. These medications have many adverse effects. The goal of antiepileptic drug therapy is to control or prevent seizures while maintaining quality of life with minimal adverse effects.

The nurse understands that which of the following drugs falls under the classification of biguanides?

Metformin HCI is the only drug that falls within the classification of biguanides.

The client has diabetes mellitus type 2. The nurse has taught the client about the illness and evaluates that learning has occurred when the client makes which statement?

My cells cannot use the insulin my pancreas makes."

A client with diabetes mellitus is taking oral agents, and is scheduled for a diagnostic test that requires him to be NPO (nothing by mouth). What is the best plan of the nurse with regard to giving the client his oral medications?

Notify the physician and request orders.

What is the MOST common drug used to treat oral candidiasis? Oseltamivir (Tamiflu) Griseofulvin (Fulvicin P/G) Amantadine (Symmetrel) Nystatin (Mycostatin)

Nystatin (Mycostatin) Nystatin is an antifungal drug that is used for a variety of candidal infections. It is applied topically as a cream, ointment, or powder. It is also available as a troche and an oral liquid or tablet.

When providing health promotion teaching at a senior citizen center, the nurse would include information about which medication used to decrease the duration of influenza A and B? Oseltamivir (Tamiflu) Ganciclovir (Cytovene) Enfuvirtide (Fuzeon) Indinavir (Crixivan)

Oseltamivir (Tamiflu) Oseltamivir (Tamiflu) and zanamivir (Relenza) are active against influenza virus types A and B and have been shown to reduce the duration of influenza infection by several days.

Streptococci

Pharyngitis, pneumonia, skin infections, septicemia, endocarditis

Klebsiella

Pneumonia, UTI

Pneumococci

Pneumonia, otitis media, meningitis, bacteremia, endocarditis

The client has type 1 diabetes mellitus and receives insulin. Which laboratory test will the nurse assess?

Potassium

The mechanism of action of regular insulin is to:

Promote entry of glucose into the cells.

The physician orders intravenous (IV) insulin for the client with a blood sugar of 563. The nurse administers insulin lispro Humalog intravenously (IV). What does the best evaluation by the nurse reveal?

Regular insulin is the only insulin that can be given intravenously

The nurse plans to administer intranasal desmopressin (DDAVP) to the client. What will the best plan of the nurse include?

Rotation of nares must be documented on the medication administration record (MAR).

The client is very distraught that her son is of very short stature. What is the best plan by the nurse?

Tell the client that treatment with growth hormone might help, and refer her to an endocrinologist.

A client visits the health care provider for treatment of tinea pedis (athlete's foot). Which medication would the nurse MOST likely instruct the client to take to treat this condition? Voriconazole (Vfend) Caspofungin (Cancidas) Amphotericin B (Amphocin) Terbinafine (Lamisil)

Terbinafine (Lamisil) Terbinafine (Lamisil) is classified as an allylamine antifungal drug and is currently the only drug in its class. It is available in a topical cream, gel, and spray for treating superficial dermatologic infections, including tinea pedis (athlete's foot), tinea cruris (jock itch), and tinea corporis (ringworm).

The nurse teaches a class for the public about diabetes mellitus. Which individual does the nurse assess as being at highest risk for developing diabetes?

The 42-year-old client who is 50 pounds overweight

The client receives treatment with radioactive iodine (Iodine-131) therapy. What will the best evaluation by the nurse reveal?

The client will most likely require thyroid replacement therapy.

The physician orders intravenous (IV) insulin for the client with a blood sugar of 563. The nurse administers insulin lispro (Humalog) intravenously (IV). What does the best evaluation of the nurse reveal?

The nurse should have used regular insulin (Humalin R).

The physician orders insulin lispro (Humalog), 10 units for the client. When will the nurse administer this medication? 5 minutes before a meal

The onset of action for insulin lispro Humalog is 10 to 15 minutes so it must be given when the client is eating

The nurse is caring for a patient with Parkinson's disease who has been taking entacapone (Comtan) for the past week to treat an on-off phenomenon. The patient expresses concern over brown-orange urine. What information will the nurse provide to the patient? Laboratory tests are needed to determine kidney function. Brown discoloration of urine may indicate a urinary tract problem. This is a normal adverse effect of entacapone (Comtan) and is not harmful. Abdominal ultrasonography will be performed to evaluate for liver damage or failure.

This is a normal adverse effect of entacapone (Comtan) and is not harmful. Patients should be instructed that entacapone (Comtan) can turn urine a brownish-orange color, and this is not a harmful adverse effect.

What is the goal of pharmacologic therapy in treating Parkinson's disease? To balance cholinergic and dopaminergic activity in the brain To increase the amount of acetylcholine at the presynaptic neurons To decrease the amount of dopamine available in the substantia nigra To block dopamine receptors in both presynaptic and postsynaptic neurons

To balance cholinergic and dopaminergic activity in the brain Parkinson's disease results from a decrease in dopaminergic (inhibitory) activity, leaving an imbalance with too much cholinergic (excitatory) activity. By increasing dopamine, the neurotransmitter activity becomes more balanced and symptoms become controlled.

Large numbers of resistant bacterial strains limit the therapeutic usefulness of many penicillins?

True

Patients may experience photosensitivity when taking tetracyclines?

True

Sulfonamides are and older class of antibiotics that is use primarily to treat infections of the urinary tract?

True

The client has been diagnosed with Cushing's syndrome. What will the best assessment by the nurse reveal?

Upper body obesity and thinning of the arms and legs.

A nurse has provided education regarding type 2 diabetes to a newly diagnosed client. Which statements would the nurse interpret as indicating need for additional education?

Well, at least the medications I will be on will help me lose weight." "I can take an oral medication and will never have to inject myself."

A client with diabetes mellitus type 1 is found unresponsive in the clinical setting. Which nursing action is a priority?Treat the client for hypoglycemia.

When a client with diabetes mellitus type 1 is found unresponsive, the nurse should focus on and treat for hypoglycemia, as this is more likely than hyperglycemia.

The client has been diagnosed with diabetes mellitus type 1. He asks the nurse what this means. What is the best response by the nurse?

Without insulin you will develop ketoacidosis (DKA)." "The endocrine function of your pancreas is to secrete insulin, but it isn't working.""It means your pancreas cannot secrete insulin."

A patient with renal failure has severe anemia, and there is an order for darbepoetin (Aranesp). As the nurse assess the patient, which condition listed will the nurse consider a contraindication to use of this medication? a. Uncontrolled hypertension b. diabetes mellitus c. hypothyroidism d. angina

a

The nurse provides discharge instructions to a patient prescribed cholestyramine (Questran). Which statement by the patient indicates teaching was effective for this drug? a. "I will increase fiber in my diet and drink more fluids." b. "This drug can cause flushing, itching and gastrointestinal upset." c. "I will take Questran 1 hour before my other medications." d. "I will notify my health care provider if I have muscle pain."

a. "I will increase fiber in my diet and drink more fluids." Cholestyramine can cause constipation; thus increasing dietary fiber and fluid intake is appropriate. All other drugs should be taken 1 hour before or 4 hours after cholestyramine to facilitate proper absorption.

The nurse will assess a patient receiving gemfibrozil (Lopid) and warfarin (Coumadin) for the increased risk of which adverse effect? a. bleeding b. clotting c. vitamin K toxicity d. deep vein thrombosis

a. Bleeding Gemfibrozil can bind with vitamin K in the intestinal tract, reducing vitamin K absorption. Because vitamin K is the antidote for warfarin, a lack of vitamin K increases the anticoagulant effect of warfarin and thus the risk of bleeding.

The nurse is providing care to a client prescribed a nonselective adrenergic agonist bronchodilator. Which condition documented in the client's medical history would alert the nurse to question this prescription? a. Coronary artery disease b. Thrombocytopenia c. Chronic obstructive pulmonary disease d. Mycobacterium tuberculosis

a. Coronary artery disease Nonselective adrenergic agonist bronchodilators stimulate beta1 receptors in the heart and beta2 receptors in the lungs. Stimulation of beta1 receptors can increase heart rate and contractility, increasing oxygen demand. This increased oxygen demand may lead to angina or myocardial ischemia in clients with coronary artery disease.

Discharge teaching to a patient receiving a beta-agonist bronchodilator should emphasize reporting which side effect? a. tachycardia b. nonproductive cough c.hypoglycemia d. sedation

a. Tachycardia A beta-agonist bronchodilator stimulates the beta receptors of the sympathetic nervous system, resulting in tachycardia, bronchodilation, hyperglycemia, and increased alertness.

The nurse would teach a client prescribed metronidazole (Flagyl) to avoid ingestion of which drink? a. Wine b. Coffee c. Milk d. Orange juice

a. Wine A disulfiram-like (Antabuse) reaction may occur with concurrent ingestion of metronidazole and alcohol, leading to facial flushing, tachycardia, palpitations, nausea, and vomiting.

A patient who has a history of asthma is experiencing an acute episode of shortness of breath and needs to take medication for immediate relief. The nurse will choose which medication that is appropriate for this situation? a. a beta agonist, such as albuterol b. a leukotriene receptor antagonist, such as montelukast c. a corticosteroid, such as fluticasone d. an anticholinergic, such as ipratropium

a. a beta agonist, such as albuterol

Vasopressin is used in the treatment of a. diabetes insipidus b. dehydration c. electrolyte imbalances d. diabetes mellitus

a. diabetes insipidus

The nurse teaches the patient taking dexamethasone for the next two moths safe behavior : a. do not stop the medication abruptly b. do not take the medication with food c. it is safe to switched the medication time to morning then night if desired. d. do not take calcium supplements with this medication.

a. do not stop the medication abruptly

The client receives hydrocortisone therapy. The nurse will primarily assess for which electrolyte disturbance? a. hypernatremia and hyperglycemia b. hypernatremia and hyperkalemia c. hypocalcemia and hyperkalemia d. hypoglycemia and hyponatremia

a. hypernatremia and hyperglycemia

The client has hyperthyroidism. The nurse teaches the client to avoid which food selection? a. soy sauce b. milk products c. high-calorie foods d. caffeine-free soda

a. soy sauce

precose(acarbose)

alpha glucosidase inhibitor

DDAVP(desmopressin)

antidiuretic hormone

PTU (propylthriouracil)

antithyroid medication

The nurse is assessing a patient who is to receive folic acid supplements. It is important to rule out which condition before giving the folic acid? a. Malabsorption syndromes b. pernicious anemia c. tropical sprue d. pregnancy

b

The physician orders insulin lispro (Humalog), 10 units for the client. When will the nurse administer this medication? a. Thirty minutes before meals b. With the meal c. Twenty minutes after the meal d. Thirty minutes after the meal

b

Client teaching regarding the use of leukotriene receptor antagonists (LTRAs) drugs such as zafirlukast (Accolate) would include which statement by the nurse? a. "It will take about 3 or 4 weeks before you notice a therapeutic effect." b. "This medication works by preventing the inflammation that causes your asthma attack." c. "Increase fiber and fluid in your diet to prevent the common adverse effect of constipation." d. "Take the medication when you are short of breath and begin wheezing."

b. "This medication works by preventing the inflammation that causes your asthma attack." LTRAs drugs block the inflammatory response of leukotrienes and thus the trigger for asthma attacks. Response to these drugs is usually noticed within 1 week. They are not used to treat acute asthma attacks. Diarrhea, not constipation, is a common adverse effect of montelukast and zafirlukast

To decrease the skin flushing adverse effect reaction of niacin (nicotinic acid), which action should the nurse take? a. Administer niacin with a liquid antacid. b. Administer aspirin 30 minutes before each dose. c. Give niacin with all other morning medications. d. Apply cold compresses to the head and neck.

b. Administer aspirin 30 minutes before each dose. To help minimize the adverse effect flushing of the skin, the patient should take a small dose of aspirin or nonsteroidal antiinflammatory drugs 30 minutes before taking niacin, but only as prescribed or recommended by the health care provider.

The nurse would question a prescription for colesevelam (Welchol) in a patient with which condition? a. hepatic disease b. bowel obstruction c. renal disease d. glaucoma

b. Bowel obstruction Colesevelam (Welchol) is contraindicated in patients with a history of bowel obstruction.

A client states to the nurse, "I must take my iron supplement with a meal to avoid stomach upset." To increase uptake of oral iron, which food group should the nurse instruct the client to avoid? a. Proteins b. Dairy c. Vegetables d. Fruits

b. Dairy Many individuals find that they need to take oral iron products with meals or food because of the commonly encountered adverse effect of gastrointestinal upset even though altered absorption occurs. If antacids or milk products are used, schedule them at least 1 to 2 hours before or after the oral dosage of iron or avoid taking with dairy products.

A patient with elevated triglyceride levels unresponsive to HMG-CoA reductase inhibitors will most likely be prescribed which drug? a. Colestipol (Colestid) b. Gemfibrozil (Lopid) c. Simvastatin (Zocor) d. Cholestyramine (Questran)

b. Gemfibrozil (Lopid) Gemfibrozil, a fibric acid derivative, promotes catabolism of triglyceride-rich lipoproteins.

Which instruction should the nurse include when teaching a client who is prescribed an expectorant? a. Take a drug with oral codeine when cough worsens. b. Increase fluid intake to decrease viscosity of secretions. c. Take the medication once a day at bedtime. d. Restrict fluids to decrease mucus production.

b. Increase fluid intake to decrease viscosity of secretions. Expectorant drugs are used to decrease viscosity of secretions and allow them to be more easily expectorated. Increasing fluid intake helps this action.

What is the mechanism of action of ezetimibe (Zetia)? a. Binds to bile acids in the intestine, inhibiting its reabsorption into the blood b. Inhibits absorption of dietary and biliary cholesterol in the small intestine. c. Decreases the adhesion of cholesterol in the arteries. d. Inhibits the biosynthesis of cholesterol in the liver.

b. Inhibits absorption of dietary and biliary cholesterol in the small intestine. Ezetimibe selectively inhibits absorption of cholesterol in the small intestine.

The nurse will plan to inject iron dextran by which technique? a. Subcutaneous injection with a ½-inch, 25-gauge needle b. Intramuscular (IM) injection using the Z-track method c. Intradermal injection with a sunburst technique of administration d. IM injection with a ½-inch, 18-gauge needle

b. Intramuscular (IM) injection using the Z-track method Iron dextran should be administered deep in a large muscle mass using the Z-track method and a 23-gauge, 1½-inch needle to prevent skin irritation and potential necrosis.

The client's culture has grown gram-positive cocci, and the health care provider prescribes two different antibiotics, one of which is gentamicin (Garamycin). To treat this type of infection, which type of antibiotic is typically prescribed together with gentamicin (Garamycin)? a. Aminoglycoside b. Penicillin c. Fluoroquinolone d. Cephalosporin

b. Penicillin In gram-positive cocci, gentamicin is usually given in combination with a penicillin antibiotic. The other antibiotics are not typically prescribed with gentamicin for this culture result.

Which herbal product, when taken with theophylline, can decrease theophylline's serum drug levels? a. Peppermint oil b. St. John's Wort c. garlic d. Echinacea

b. St. John's wort has been shown to enhance the rate of theophylline metabolism, thus decreasing serum levels.

this hormone makes up 95% of the mineralcorticoid secreted by the adrenal glands. a. hydrocortisone b. aldosterone c. antidiuretic hormone d. DDAVP

b. aldosterone

The nurse is reviewing a patient's medication orders for prn medications that can be given to a patient who has bronchitis with a productive cough. Which drug will the nurse choose? a. an antitussive b. an expectorant c. an antihistamine d. a decongestant

b. an expectorant

The physician orders insulin lispro (Humalog), 10 units for this client. When will the nurse administer this medication? a. thirty minutes before meals b. five minutes before a meal c. when the meal trays arrive on the floor d. fifteen minutes after meals

b. five minutes before a meal

A client has been taking propylthiouracil (PTU) for several weeks. Which of the client's statements would the nurse interpret as indicating that the drug is not having its desired effects? a. i am sleeping so much better b. i cant seem to gain any weight no matter how much i eat c. i dont feel as anxious as i once did d. my heart doesnt seem to be racing anymore

b. i cant seem to gain any weight no matter how much i eat

The mechanism of action of regular insulin is to a. stimulate the pancreas to produce insulin b. promote entry of glucose into the cells c. promote the entry of glucose into the bloodstream d. stimulate the pancreas to secrete more insulin

b. promote entry of glucose into the cells

The nurse understands that which of the following drugs fall under the classification of the meglitinides? a. metformin (glucophage) b. repaglinide( prandin) c. tolubramide (orinase) d. acarbose (precose)

b. repaglinide( prandin)

The client has diabetes type 1 and receives insulin for glycemic control. The client tells the nurse that she likes to she a glass of wine with dinner . What will be the plan by the nurse for client education include? a. the alcohol could cause pancreatic disease and decrease insulin production b. the alcohol could predispose you to hypoglycemia c. the alcohol could cause serious liver disease? d. the alcohol could predispose you to hyperglycemia

b. the alcohol could predispose you to hypoglycemia

fortamet (metformin)

biguanide

Insulin is released when

blood glucose increases.

A client was recently prescribed levothyroxine, but is unsure of the instructions on how to take the medication. The nurse is providing the client with instructions on how the medication should be taken. The nurse determines that the teaching was effective when the client states: a. "I need to take this medication every day, but thankfully it doesn't matter when I take it" b. "This medication should help with my hyperthyroidism" c. "I understand that it is best to take this medication the same time every day and on an empty stomach" d. "It isn't likely that this medication will interact with any of my other medications since it is a natural hormone in my body."

c

A treatment frequently used to treat hyperthyroidism using radioactive iodine works by: a. Destroying the pituitary gland responsible for secreting TSH. b. Replacing the thyroid hormone with a synthetic version. c. Reducing thyroid hormone secretion by destroying the thyroid tissue. d. Increasing the effectiveness of the negative feedback mechanism.

c

During a teaching session for a patient who will be receiving a new prescription for the LTRA montelukast (singulair), the nurse will tell the patient that the drug has which therapuetic effect? a. improves the respiratory drive b. Loosens and removes thickened secretions c. Reduces inflammation in the airway d. stimulates immediate bronchodilation

c

The client has type 1 diabetes and receives insulin. He asks the nurse why he can't just take pills instead. What is the best response by the nurse? a. "I know it is tough, but the shots won't hurt." b. "Have you talked to your doctor about taking pills instead?" c. "Insulin can't be in a pill because it is destroyed in stomach acid." d. "Insulin must be injected because it needs to work quickly."

c

The physician orders intravenous (IV) insulin for the client with a blood sugar of 563. The nurse administers insulin glargine intravenously (IV). What does the best evaluation by the nurse reveal? a. The nurse used the correct insulin. b. The nurse should have contacted the physician. c. The nurse should have used regular insulin (Humulin R). d. The nurse could have given the insulin subcutaneously.

c

Which point will the nurse emphasize to a patient who is taking an atilipemic medication in the statin class? a. the drug needs to be taken on an empty stomach before meals. b. a low-fat diet is not necessary while taking these medications c. it is important to report muscle pain immediately d. improved cholesterol levels will be evident within 2 weeks.

c

When iron sucrose is administered, which nursing interventions are correct? select all that apply a. administer a test dose before giving the full dose b. give via deep intramuscular injection into a large muscle mass using the Z-track method c. administer large doses over 2.5-3.5 hours intravenously. d. monitor the patient for hypertention e. monitor the patient for hypotension

c, e

Which discharge instruction should the nurse include for a patient prescribed an antilipemic medication? a. "It is important to take a double dose to make up for a missed dose." b. "Stop taking the medication if it causes nausea and vomiting." c. "Continue your exercise program and maintain a low-fat diet." d. "Lifestyle changes are no longer necessary when taking this medication."

c. "Continue your exercise program and maintain a low-fat diet." Antilipemic medications are in addition to, not a replacement of, therapeutic lifestyle changes used to decrease serum cholesterol. Maintain a low-fat, low-cholesterol diet is an integrated part of a change in lifestyle.

Which statement by the client demonstrates an understanding about beclomethasone diproprionate (Beconase) mechanism of action? a. "I will monitor my blood sugar because I may develop drug-induced diabetes." b. "I only need to take this medication when my symptoms get bad." c. "This medication will help prevent the inflammatory response of my allergies." d. "I will need to taper off the medication to prevent acute adrenal crisis."

c. "This medication will help prevent the inflammatory response of my allergies." Beclomethasone diproprionate is a steroid spray administered nasally. It is used to prevent and treat allergy symptoms. Its effect is localized, and therefore the client does not have systemic adverse effects with the recommended dose. There is no need to taper off gradually as with oral corticosteroids.

The nurse will question the use of a fluoroquinolone antibiotic in a client already prescribed which medication? a. Furosemide (Lasix) b. Omeprazole (Prilosec) c. Amiodarone (Cordarone) d. Metoprolol (Lopressor)

c. Amiodarone (Cordarone) Dangerous cardiac dysrhythmias are more likely to occur when quinolones are taken by clients receiving class Ia and class III antidysrhythmic drugs such as disopyramide and amiodarone. For this reason, such drug combinations should be avoided.

Which statement by the nurse explains to the patient the action of cholestyramine (Questran) to decrease blood lipid levels? a. Inhibits lipolysis in adipose tissue and decreases the hepatic synthesis of triglycerides in the liver. b. Stimulates the biliary system to increase excretion of dietary cholesterol. c. Binds to bile in the intestinal tract, forming an insoluble complex that is excreted in the feces. d. Inhibits absorption of dietary cholesterol in the small and large intestine.

c. Binds to bile in the intestinal tract, forming an insoluble complex that is excreted in the feces. Cholestyramine is an anion exchange resin that binds to bile acids in the small intestine to form an insoluble complex that is excreted in the feces. The liver must then use cholesterol to synthesize more bile.

The nurse knows iron supplementation has which common adverse effect? a. flatus b. heartburn c. Constipation d. fatigue

c. Constipation Constipation and change in the color of stool to darker or green are the most common complaints with iron supplementation and are expected adverse effects.

When providing general education on use of over-the-counter (OTC) medications for allergies, which instruction should the nurse to include? a. Discontinue use 4 days before allergy testing. b. It may cause dry mouth and difficulty urinating. c. Do not use in children younger than 2 years of age unless prescribed. d. The medication treats the signs and symptoms but is not a cure.

c. Do not use in children younger than 2 years of age unless prescribed. OTC medications for allergies are not recommended for children younger than 2 years of age unless prescribed by a health care provider.

The formation of erythrocytes and maturation of the red blood cell (RBC) is driven by what hormone? a. progesterone b. testosterone c. Erythropoietin d. free thyroxin

c. Erythropoietin Erythropoiesis is the process of erythrocyte formation and this involves the maturation of a nucleated RBC precursor into a hemoglobin-filled, nucleus-free erythrocyte. This process is driven by the hormone erythropoietin, which is produced by the kidneys. Erythropoietin is also produced commercially and is used to treat anemia in certain specific circumstances.

Which statement by a client best indicates an understanding of the teaching on flunisolide (AeroBid)? a. "I will wash the plastic inhaler casing once a month." b. "I will not use my albuterol inhaler while I am taking AeroBid." c. "I will rinse my mouth with water after each use." d. "I will take two puffs to treat an acute asthma attack."

c. I will rinse my mouth with water after each use." Flunisolide is an inhaled corticosteroid. Rinsing the mouth immediately after each use of the inhaler or nebulizer will help prevent oral candidal infections. It is not used to treat an acute asthma attack and should be taken with the client's bronchodilator medications. The plastic inhaler casing is washed in warm, soapy water every week.

This type of insulin is never to be mixed with any other insulin a. NPH b. Humalog c. Lantus d. Novalog

c. Lantus

Which laboratory value would the nurse assess before administering zafirlukast (Accolate) to a client? a. Renal function tests b. complete blood count c. Liver enzymes d. cardiac enzymes

c. Liver enzymes Because use of zafirlukast may lead to liver dysfunction, liver enzyme levels should be monitored regularly, especially early in the course of therapy.

Pramlintide (Symlin) is prescribed as supplemental drug therapy to the treatment plan for a patient with type 1 diabetes mellitus. What information should the nurse include when teaching the patient about the action of this medication? a. Pramlintide stimulates glucose production. b. Pramlintide increases glucagon excretion. c. Pramlintide slows gastric emptying. d. Pramlintide corrects insulin receptor sensitivity.

c. Pramlintide slows gastric emptying. Pramlintide is a synthetic form of the naturally occurring hormone amylin. It works by slowing gastric emptying, suppressing glucagon secretion and hepatic glucose production, and increasing satiety (sense of having eaten enough). It is only administered via subcutaneously injection.

The nurse receives laboratory values for a client with a theophylline level of 14 mcg/mL. How does the nurse interpret this theophylline level? a. toxic b. Life threatening c. therapuetic d. subtherapuetic

c. The therapeutic theophylline level is 10 to 20 mcg/mL.

The nurse would question the prescription for epoetin alfa (Epogen) in a client with which condition? a. End-stage renal disease b. Acquired immunodeficiency syndrome (AIDS) c. Uncontrolled hypertension d. Chemotherapy-induced anemia

c. Uncontrolled hypertension Hypertension is a frequent adverse effect of epoetin; hence the drug should not be given to clients with uncontrolled hypertension. End-stage renal disease, chemotherapy-induced anemia, and anemia associated with zidovudine treatment for AIDS are indications for the therapeutic use of epoetin.

The nurse knows that an antitussive cough medication would be the best choice for which patient? a. a patient with a productive cough b. a patient with chronic para-nasal sinusitis c. a patient who has had recent abdominal surgery d. a patient who has influenza

c. a patient who has had recent abdominal surgery

In drawing up insulin in a syringe, in which the order is for NPH 12/ regular insulin 10, the nurse appropriately does which of the following? a. draws each type of insulin in separate syringes b. draws the NPH insulin first then the regular in the same syringe c. draws the regular insulin first then the NPH in the same syringe d. gives the regular insulin 30 minutes prior to the NPH insulin

c. draws the regular insulin first then the NPH in the same syringe

Off label use for medications occur in many situations. For instance, the medication Metformin (fortamet) is used as an off label medication for one of the following situtations: cardiac patients who are also diabetic b. for male BPH control due to the adrenal gland effect on the prostate gland c. for women with polycystic ovary syndrome d. for adolescents who are behind in puberty development

c. for women with polycystic ovary syndrome

When giving decongestants, the nurse must remember that these drugs have alpha-adrenergic-stimulating effects that may result in which effect? a. fever b. bradycardia c. hypertension d. CNS depression

c. hypertension

The nurse would question a prescription for pseudoephedrine (Sudafed) in a client with a history of which condition? a. pneumonia b. peptic ulcer diseasse c. hypertension d. osteoporosis

c. hypertension (Adrenergic drugs are contraindicated in clients with hypertension, narrow-angle glaucoma, diabetes, uncontrolled cardiovascular disease, hyperthyroidism, prostatitis, or a known hypersensitivity to such drugs.)

The primary goal of pharmacotherapy in hyperthyroidism (Grave's disease) is to a. increase metabolism b. increase synthesis of corticosteroid c. lower the activity of the thyroid hormone d. increase the activity of the thyroid hormone

c. lower the activity of the thyroid hormone

The antidiabetic medication Diabinese(chlorpropamide) is considered one of the first generation sulfonylureas. This class of medication works by which of the following? a. blocks enzymes in the small intestine responsible for breaking down of carbohydrates into monosaccharides b. enhances incretin stimulation to signal the pancreas to produce insulin and block the liver from producing glucagon c. stimulates the release of insulin from pancreatic islet cells and by increasing sensitivity of receptors on target cells d. reduces blood glucose by decreasing insulin resistance and inhibiting hepatic gluconeogenesis.

c. stimulates the release of insulin from pancreatic islet cells and by increasing sensitivity of receptors on target cells

The nurse understands that which of the following drugs fall under the classification of sulfonylureas? a. metformin (glucophage) b. repaglinide( prandin) c. tolubramide (orinase) d. acarbose (precose)

c. tolubramide (orinase)

A major disadvantage of sliding scale insulin is that: a. It is difficult for nurses to manage. b. The insulin must be given IV form. c. It requires hospital-grade insulin d. It delays insulin administration until hyperglycemia occurs, causing a large fluctuation of blood glucose.

d

When administering oral iron tablets, the nurse should keep in mind that the most appropriate substance, other than water, to give these tablets is? a. pudding b. an antacid c. milk d. orange juice

d

The nurse performs discharge teaching with a client who is prescribed the anticholinergic inhaler ipratropium bromide (Atrovent). Which statement by the client indicates to the nurse that teaching has been successful? a. "Nausea and vomiting are common adverse effects of this medication." b. "I may gain weight as a result of taking this medication." c. "I will not drink grapefruit juice while taking this drug." d. "This inhaler is not to be used alone to treat an acute asthma attack."

d. "This inhaler is not to be used alone to treat an acute asthma attack." Although ipratropium works to prevent bronchoconstriction and thus secondarily leads to bronchodilation, a direct-acting bronchodilator is needed to treat an acute asthma attack.

Which statement by the nurse should be included when teaching a client about the proper use of metered-dose inhalers? a. "After you inhale the medication once, repeat until you obtain relief." b. "Make sure that you puff out air several times after you inhale the medication." c. "Hold the inhaler in your mouth, take a deep breath, and then compress the inhaler." d. "Wait 1 to 2 minutes before you take a second puff of the same drug."

d. "Wait 1 to 2 minutes before you take a second puff of the same drug." If a second puff of the same drug is ordered, instruct the client to wait 1 to 2 minutes between puffs. If a second type of inhaled drug is prescribed, instruct the client to wait 2 to 5 minutes between the medications or to take as prescribed.

When performing discharge teaching for a client prescribed oral linezolid (Zyvox) to treat methicillin-resistant Staphylococcus aureus (MRSA), the nurse should emphasize which important information? a. Stop the drug as soon as you feel better. b. Take the drug with an antacid to avoid gastrointestinal (GI) upset. c. Report any occurrence of constipation or facial flushing. d. Avoid ingestion of foods containing tyramine.

d. Avoid ingestion of foods containing tyramine. Hypertension may occur in clients consuming tyramine-containing foods such as aged cheese or wine, soy sauce, smoked meats or fish, and sauerkraut while taking linezolid. Linezolid causes diarrhea, not constipation, and should be taken with food to decrease GI distress. An antacid would interfere with absorption.

For a client receiving an intravenous (IV) infusion of gentamicin (Garamycin), the nurse would monitor which laboratory values? a. Serum glutamic-oxaloacetic transaminase and alanine transaminase b. Prothrombin time and partial thromboplastin time c. Hematocrit and hemoglobin d. Blood urea nitrogen (BUN) and creatinine

d. Blood urea nitrogen (BUN) and creatinine Gentamicin has a high potential for nephrotoxicity. Nephrotoxicity typically occurs in 5% to 25% of clients. Thus, the client's renal function test results for BUN and creatinine must be monitored closely throughout therapy.

The nurse should assess a client for nephrotoxicity and ototoxicity when administering which antimicrobial? a. Erythromycin b. Clindamycin (Cleocin) c. Cefazolin (Ancef) d. Gentamicin (Garamycin)

d. Gentamicin (Garamycin) Aminoglycoside antibiotics, including gentamicin, have a high risk for nephrotoxicity and ototoxicity.

What is the role of corticosteroids in the treatment of acute respiratory disorders? a. They increase gas exchange in the alveoli. b. They stimulate the immune system. c. They directly dilate the bronchi. d. They decrease inflammation.

d. They decrease inflammation. Corticosteroids can suppress the immune system. They do not directly affect bronchodilation but rather prevent bronchoconstriction as a response to inflammation.

The nurse understands that which of the following drugs fall under the classification of alpha-glucosidase inhibitors? a. metformin (glucophage) b. repaglinide( prandin) c. tolbutamide (orinase) d. acarbose (precose)

d. acarbose (precose)

The nurse plans to administer intranasal desmopressin (DDAVP) to the client. What will the best plan by the nurse include? a. instruct client to blow his nose following admin b. be sure to have fresh water at bedside c withhold other medications so absorption of desmopressin will not be affected d. direct the spray high into the nasal cavity

d. direct the spray high into the nasal cavity

a patient is taking a decongestant to help reduce symptoms of a cold. The nurse will instruct the patient to observe for which possible symptom, which may indicate and adverse effect of this drug? a. increased cough b. dry mouth c. slower heart rate d. heart palpations

d. heart palpations

the antidiabetic medication Avandia(rosiglitazone) is considered one of the thiazolidinediones. This class of medication works by the following: a. blocks enzymes in the small intestine responsible for breaking down of carbohydrates into monosaccharides b. enhances incretin stimulation to signal the pancreas to produce insulin and block the liver from producing glucagon c. stimulates the release of insulin from pancreatic islet cells and by increasing sensitivity of receptors on target cells d. reduces blood glucose by decreasing insulin resistance and inhibiting hepatic gluconeogenesis.

d. reduces blood glucose by decreasing insulin resistance and inhibiting hepatic gluconeogenesis.

A patient is prescribed selegiline (Eldepryl), an MAO-B inhibitor, as adjunctive therapy in treatment of Parkinson's disease. What potential adverse effects should the nurse include in education for this patient? (Select all that apply.) Select all that apply. Weight gain Headache Blood pressure changes Depression Insomnia

headache, blood pressure changes, depression, insomnia MAO-B inhibitors have many potential adverse effects, including dizziness, insomnia, hallucinations, ataxia, agitation, depression, paresthesia, somnolence, headache, dyskinesia, nausea, diarrhea, hypotension or hypertension, chest pain, weight loss, dermatologic reactions, rhinitis, and pharyngitis.

januvia(sitagliptin)

incretin mimetic enhancer

The most significant drug interactions with use of antivirals occur when antivirals are administered via which route? Optically Rectally Topically Intravenously

intravenously Significant drug interactions that occur with antiviral drugs arise most often when they are administered via systemic routes such as intravenously and orally. Many of these drugs are also applied topically to the eye or body, however, and the incidence of drug interactions associated with these routes of administration is much lower.

What conditions are considered contradictions for use of antifungal medications? (Select all that apply.) Select all that apply. Heart failure Respiratory failure Pancreatic failure Liver failure Kidney failure

liver failure, and kidney failure Drug allergy, liver failure, kidney failure, and porphyria (for griseofulvin) are the most common contraindications for antifungal drugs.

Humilin R

short acting insulin

The client injects his insulin as prescribed, but then gets busy and forgets to eat. What is the nurse's most likely assessment finding?

the client will have moist skin

synthroid( levothyroxine)

thyroid agent

A potential treatment for Addison disease is fludrocortisone. True False

true

The nurse teaches the client about glucocorticoid therapy.DThe nurse evaluates that additional teaching is required when the client makes which statement?

"I can take the medication at any time as long as I don't forget it."

Which client statement regarding his or her diagnosis of HIV infection indicates a need that further teaching is necessary? "I must take these medications exactly as prescribed for the rest of my life." "I don't need to use condoms as long as I take my medication as prescribed." "I will notify my health care provider immediately if I bruise or bleed more easily than normal." "I should remain upright for 30 minutes after taking my zidovudine to prevent esophageal ulceration." Submit

"I don't need to use condoms as long as I take my medication as prescribed." Antiretroviral drugs do not stop the transmission of HIV, and clients need to continue standard precautions and safe sex practice, including condom use. Potential serious adverse effects of zidovudine are bone marrow suppression and esophageal ulceration.

The client has type 1 diabetes and receives insulin. He asks the nurse why he can't just take pills instead. What is the best response by the nurse?

"Insulin can't be in a pill because it is destroyed in stomach acid."

The patient asks the nurse to explain the difference between carbidopa-levodopa (Sinemet) and ropinirole (Requip). How will the nurse respond? "Both drugs have the same pharmacodynamic and adverse effect profiles." "Ropinirole is a dopamine agonist that has fewer adverse effects than carbidopa-levodopa." "Carbidopa-levodopa is less effective than ropinirole in treating the symptoms of Parkinson's disease." "Whereas carbidopa-levodopa acts as a dopamine agonist, ropinirole directly replaces dopamine."

"Ropinirole is a dopamine agonist that has fewer adverse effects than carbidopa-levodopa." Ropinirole stimulates specific dopamine receptors in the brain. It is more specific for the receptors associated with parkinsonian symptoms, the D2 subfamily of dopamine receptors. This in turn may have more specific antiparkinson effects, with fewer adverse effects associated with generalized dopaminergic stimulation.

A patient with Parkinson's disease who has been positively responding to carbidopa-levodopa (Sinemet) asks the nurse, "Why have I suddenly developed a relapse of my symptoms." Which explanation by the nurse is appropriate? "This is called the "on-off phenomenon." Your health care provider can change your medication regimen to help manage this effect." "This is very common and it is temporary. Continue to take the medicine, and you will feel better in a few weeks." "You have developed drug tolerance to your current medication, and a higher dose will need to be prescribed." "You have developed medication toxicity. Thus, the drug dose will be reduced to avoid potentially toxic drug blood levels."

"This is called the "on-off phenomenon." Your health care provider can change your medication regimen to help manage this effect." Patients who have been taking levodopa for a period of time may experience periods of symptom return. Changing to Sinemet controlled release or adding another medication can help reduce the on-off phenomenon.

The client has diabetes insipidus and receives vasopressin (Pitressin).DThe nurse completes medication education and evaluates that learning has occurred when the client makes which statement?

"This medication increases water reabsorption in my kidneys."

You could have nerve problems that lead to numbness or tingling in your feet or hands." "One of the most serious complications is diabetic ketoacidosis."

/

The client has had malaria for many years. He asks the nurse why medications might not cure his illness when there are several drugs available. What is the best response by the nurse? 1. When cysts occur late in the disease, the parasite is too resistant for medications to be effective 2. Once the parasite starts multiplying in your liver medications are usually ineffective 3. When erythrocytes rupture the parasites are too numerous for medications to be effective 4. Late in the illness the immune system is too overwhelmed for medications to be effective

1

The client receives acyclovir (Zovirax for treatment of genital herpes. What is a priority assessment by the nurse? 1. Increased serum cratinine 2. Respiratory distress 3. Thromboctopenia 4. Auditory and visual hallucinations

1

The client receives oral nystatin (Mycostatin) suspension for an oral candidiasis infection. She tells the nurse she cannot continue to swish and swallow because her nausea is too great. What is the best response by the nurse? 1. It is all right to swish the medication and then spit it out 2. Try drinking a 7- up after you swallow the medication 3. You can take a phenergan suppository before the nystatin (Mycostatin) 4. I will ask your doctor if a pill form can be substituted

1

The nurse understands that which of the following drugs falls under the classification of biguanides? 1. Metformin HCI (Glucophage) 2. Repaglinide (Prandin) 3. Tolbutamide (Orinase) 4. Acarbose (Precose)

1 Rationale 1: Metformin HCI is the only drug that falls within the classification of biguanides. Rationale 2: Repaglinide falls within the classification of meglitinides. Rationale 3: Tolbutamide falls within the classification of sulfonylureas. Rationale 4: Acarbose falls within the classification of alpha-glucosidase inhibitors.

The nurse has finished teaching a client with diabetes mellitus how to administer insulin. The nurse evaluates that learning has occurred when the client makes which statement? 1. I should only use a calibrated insulin syringe for the injections. 2. I should check my blood sugar immediately prior to the administration. 3. I should use the abdominal area only for insulin injections. 4. I should provide direct pressure over the site following the injection.

1 Rationale 1: To ensure the correct insulin dose, a calibrated insulin syringe must be used. Rationale 2: There is no need to check blood glucose immediately prior to the injection. Rationale 3: Insulin injections should also be rotated to the arm and thigh, not just the abdominal area. Rationale 4: There is no need to apply direct pressure over the site following an insulin injection.

The nurse makes a home visit to a client with diabetes mellitus. During the visit, the nurse notes that the clients 3-month supply of insulin vials that were delivered a week ago are not refrigerated. What is the best action by the nurse at this time? 1. Have the client place the insulin vials in the refrigerator. 2. Have the client discard the vials. 3. Instruct the client to label each vial with the date when opened. 4. Tell the client this is too much insulin to have on hand.

1 Rationale 1: Vials can be stored at room temperature up to one month. For longer storage, they should be refrigerated. Rationale 2: There is no need to discard the vials. Rationale 3: Writing the date of opening on the vial is good practice, but does not address the need to refrigerate additional vials. Rationale 4: There is no indication that this is too much insulin to have on hand.

A mother is distressed because her child has been diagnosed with ascariasis. What information should the nurse provide? Select all that apply. 1. Children get worms from playing in infected soil 2. Treatment will consist of medication for 3 days 3. No treatment will be necessary after I give this one pill

1 and 2

The client receives metformin (Glucophage). What will the best plan by the nurse include with regard to patient education with this drug?Select all that apply. 1. It decreases sugar production in the liver. 2. It inhibits absorption of carbohydrates. 3. It stimulates the pancreas to produce more insulin. 4. It reduces insulin resistance. 5. It increases energy use.

1,4 Rationale 1: Metformin (Glucophage) decreases sugar production (gluconeogenesis) in the liver. Rationale 2: Metformin (Glucophage) does not inhibit the absorption of carbohydrates. Rationale 3: Metformin (Glucophage) reduces insulin resistance. Rationale 4: Metformin (Glucophage) reduces insulin resistance. Rationale 5: Metformin (Glucophage) does not increase energy use.

A client has been taking propylthiouracil (PTU) for several weeks. Which client statements would the nurse interpret as indicating that the drug is having its desired effects? Select all that apply. 1. "I am sleeping so much better." 2. "I can't seem to gain any weight no matter how much I eat." 3. "I don't feel as anxious as I once did." 4. "My heart doesn't seem to be racing anymore." 5. "My skin in not bruising as much."

1. "I am sleeping so much better." 3. "I don't feel as anxious as I once did." 4. "My heart doesn't seem to be racing anymore."

The nurse teaches the client about glucocorticoid therapy. The nurse evaluates that additional teaching is required when the client makes which statement? 1. "I can take the medication at any time as long as I don't forget it." 2. "I will monitor my blood sugar on a regular basis." 3. "I will eat a diet that is high in protein." 4. "I should take my medication after I have eaten."

1. "I can take the medication at any time as long as I don't forget it."

The nurse makes a home visit to a client with diabetes mellitus. During the visit, the nurse notes that the client's 3-month supply of insulin vials that were delivered a week ago are not refrigerated. What is the best action by the nurse at this time? 1. Have the client place the insulin vials in the refrigerator. 2. Have the client discard the vial 3. Instruct the patient to label each vial with the date when opened. 4. Tell the client this is too much insulin to have on hand.

1. Have the client place the insulin vials in the refrigerator. Vials can be stored at room temperature up to one month. For longer storage, they should be refrigerated.

The client receives hydrocortisone therapy. The nurse will primarily assess for which electrolyte disturbance? 1. Hypernatremia and hyperglycemia 2. Hypernatremia and hyperkalemia 3. Hypocalecmia and hyperkalemia 4. Hypoglycemia and hyponatremia

1. Hypernatremia and hyperglycemia

The client receives metformin Glucophage What will the best plan by the nurse include with regard to patient education with this drug? Select all that apply: 1. It decreases sugar production in the liver 2. It inhibits absorption of carbohydrates 3. It stimulates the pancreas to produce more insulin 4. It reduces insulin resistance 5. It increases energy use

1. It decreases sugar production in the liver. 4. It reduces insulin resistance.

The physician writes orders for the client with diabetes mellitus. Which order would the nurse validate with the physician? 1. Lantus insulin 20U BID 2. Administering regular insulin 30 min prior to meals 3. 5 units of Humalog/10 units NPH daily 4. Metformin (Glucophage) 1000 mg per day in divided doses

1. Lantus insulin 20U BID Lantus insulin is usually prescribed in once-a-day dosing so an order for BID dosing should be validated with the physician.

The nurse is caring for several clients. Which client will the nurse anticipate is most likely to experience an alteration in drug metabolism? 1. A 3 day old premature infant 2. a 22 year old pregnant female 3. a 32 year old man with kidney stones 4. a 50 year old executive with hypertension

1. a 3 day old premature infant Rationale: Infants do not develop a mature microsomial enzyme sysem until they are a year old and therefore do not metabolize drugs very efficiently. Pregnancy does not significantly affect drug metabolism, the concern with pregnancy is primarily focused on alterations in distributions due to the fetal-placental barrier. The presence of kidney stones would not influence drug metabolism. Hypertension is not a factor that directly results in abnormal metabolism

The nursing instructor teaches the student nurses about the endocrine system. The nursing instructor evaluates that learning has occurred when the student nurses make which statement(s)? Select all that apply.

1."The endocrine system is a major controller of homeostasis." 2."The hypothalamus secretes releasing hormones." 3."Hormones released by the endocrine system influence every organ in the body." 4.The pituitary gland secretes TSH (thyroid stimulating hormone)."

10. Drug therapy is being considered for an HIV-infected patient who has a CD4+ cell count of 400/µl. The nursing assessment that is most important in determining whether therapy will be used is the patient's: A. Social support system offered by significant others and family. B. Socioeconomic status and availability of medical insurance. C. Understanding of the multiple side effects that the drugs may cause. D. Willingness and ability to comply with stringent medication schedules.

10. D. Rationale: Drug resistance develops quickly unless the patient takes multiple drugs on a stringent schedule, and this endangers both the patient and the community. The other information is also important to consider, but patients who are unable to manage and follow a complex drug treatment regimen should not be considered for ART.

A patient receiving phenytoin (Dilantin) has a serum drug level drawn. Which level will the nurse note as therapeutic? 6 mcg/mL 12 mcg/mL 8 mcg/mL 30 mcg/mL

12 mcg/mL Therapeutic serum drug levels for phenytoin are between 10 and 20 mcg/mL.

The client receives acyclovir (Zovirax) as treatment for herpes simplex type 1 virus (genital herpes). What is the best expected outcome for this client? 1. The client will report a decrease in the number of lesions in the affected area 2. The client will report decreased pain using the approved pain scale. 3. The client will identify the names of sexual contacts in the past month. 4. The client will prevent a reoccurrence of infection in the affected areas.

2

A client with diabetes mellitus is taking oral agents, and is scheduled for a diagnostic test that requires him to be NPO (nothing by mouth) and to have contrast dye. What is the best plan by the nurse with regard to giving the client his oral medications? 1. Notify the diagnostic department and request orders. 2. Notify the physician and request orders. 3. Administer the oral agents immediately after the test. 4. Administer the oral agents with a sip of water before the test.

2 Rationale 1: The radiologist in the diagnostic department might give orders, but it would be best to check with the clients physician first. Rationale 2: It is best to notify the clients physician and request orders. Rationale 3: Some oral medications should not be given for up to 2 days after receiving IV contrast. Rationale 4: The client should not receive the medication during NPO (nothing by mouth) status unless directed by the physician. Some oral medications should not be given for 2 days before receiving IV contrast.

A nurse is preparing and education session for a group of public school teachers regarding prevention of influenza. What topics should be included in this session? Select all that apply. 1. There is little we can do to prevent a pandemic 2. They need to discuss immunization with their person health care providers due to contact with children 3. Encourage frequent hand washing for all students and staff 4. Encourage students and staff to stay home if they are ill 5. Anti-influenza drugs should be reserved only for adults who have been diagnosed with influenza

2,3, and 4

The client has been diagnosed with diabetes mellitus type 1. He asks the nurse what this means. What is the best response by the nurse? Select all that apply. 1. The exocrine function of your pancreas is to secrete insulin and it is not working. 2. Without insulin you will develop ketoacidosis (DKA). 3. The endocrine function of your pancreas is to secrete insulin, but it isn't working. 4. Your alpha cells should be able to secrete insulin, but cannot. 5. It means your pancreas cannot secrete insulin.

2,3,5 Rationale 1: The endocrine, not the exocrine, function of the pancreas is to secrete insulin. Rationale 2: A consequence of diabetes mellitus type 1 is that without insulin, severe metabolic disturbances, such as diabetic ketoacidosis (DKA) will result. Rationale 3: The endocrine function of the pancreas is to secrete insulin. Rationale 4: Insulin is secreted by the beta, not the alpha, cells of the pancreas. Rationale 5: One function of the pancreas is to secrete insulin.

A nurse is reviewing the blood work of a client who has recently begun treatment for type 2 diabetes. Which results would indicate that the client is on target with therapy? Select all that apply. 1. HBA1C level is 8.4%. 2. HBA1C level is 6.3%. 3. Fasting blood glucose is 130 g/dL. 4. Fasting blood glucose is 100 g/dL. 5. Fasting blood glucose is 68 g/dL.

2,4 Rationale 1: This level is too high. Rationale 2: Target level is 6.5% or less. Rationale 3: This is too high. Rationale 4: Goal is 110 g/dL. Rationale 5: A FBG of 68 g/dL is too low.

A client receives metronidazole (Flagyl) for treatment of a vaginal yeast infection. What should the nurse include in medication education for this client? Select all that apply. 1. Drink as least 3000ml of fluid per day with this medication 2. You may experience a metallic taste while on this medication 3. Do not take this medication with milk or milk products. 4. Do not drink alcohol with this medicaiton 5. Do not become pregnant while taking this medication

2,4, and 5

A client has been prescribed exenatide (Byetta). What medication education should the nurse provide? Select all that apply. 1. Drink an 8 ounce glass of water with this pill. 2. You should take this medication twice each day. 3. You may experience dryness of the mouth while taking this drug. 4. You may develop diarrhea while taking this drug. 5. This drug will help you secrete more insulin.

2,4,5 Rationale 1: Exenatide is an injectable drug. Rationale 2: Exenatide is often injected twice daily. Rationale 3: This is not an expected adverse reaction. Rationale 4: Diarrhea is an expected adverse effect of this drug. Rationale 5: One of the actions of this drug is to increase secretion of insulin.

The mother of a 4-year-old boy states, I cant believe my son has type 1 diabetes. We eat well and I was so careful during the pregnancy. What could have caused this? How should the nurse respond? Select all that apply. 1. There must have been something you were exposed to during your pregnancy that caused it. 2. Are there others in your family that have type 1 diabetes? 3. He must have been allowed to eat too much sugar. 4. We are not certain what causes type 1 diabetes. 5. It is thought to be a combination of factors.

2,4,5 Rationale 1: There is no known factor that would cause type 1 diabetes if the mother was exposed. Rationale 2: There is a genetic factor associated with type 1 diabetes. Rationale 3: This is not a therapeutic statement and supports the urban myth that diabetes is caused by sugar ingestion. Rationale 4: The specific factors that cause Type 1 diabetes are still undiscovered. Rationale 5: It is thought that type 1 diabetes is caused by a number of interrelated factors.

2. Four years after seroconversion, an HIV-infected patient has a CD4+ cell count of 800/µl and a low viral load. The nurse teaches the patient that: A. The patient is at risk for development of opportunistic infections because of CD4+ cell destruction. B. The patient is in a clinical and biologic latent period, during which very few viruses are being replicated. C. Anti-HIV antibodies produced by B cells enter CD4+ cells infected with HIV to stop replication of viruses in the cells. D. The body currently is able to produce an adequate number of CD4+ cells to replace those destroyed by viral activity.

2. D. Rationale: The patient is the early chronic stage of infection, when the body is able to produce enough CD4+ cells to maintain the CD4+ count at a normal level. The risk for opportunistic infection is low because of the normal CD4+ count. Although the viral load in the blood is low, intracellular reproduction of virus still occurs. Anti-HIV antibodies produced by B cells attack the viruses in the blood, but not intracellular viruses.

The mechanism of action of regular insulin is to 1. Stimulate the pancreas to produce insulin 2. Promote entry of glucose into the cells 3. Promote the entry of glucose into the bloodstream 4. Stimulate the pancreas to secrete more insulin

2. Promote entry of glucose into the cells The action of regular insulin is to promote entry of glucose into the cells, thereby lowering glucose.

The nurse teaches a class for the public about diabetes mellitus. Which individual does the nurse assess as being at highest risk for developing diabetes? 1. The 38-year-old client who smokes one pack of cigarettes per day 2. The 42-year-old client who is 50 pounds overweight 3. The 50-year-old client who does not get any physical exercise 4. The 56-year-old client who drinks three glasses of wine each evening

2. The 42-year-old client who is 50 pounds overweight Obesity increases the likelihood of developing diabetes mellitus due to overstimulation of the endocrine system.

The client has diabetes type 1 and receives insulin for glycemic control. The client tells the nurse that she likes to have a glass of wine with dinner. What will the best plan by the nurse for client education include? 1. The alcohol could cause pancreatic disease and decrease insulin production 2. The alcohol could predispose you to hypoglycemia 3. The alcohol could cause serious liver disease 4. The alcohol could predispose you to hyperglycemia

2. The alcohol could predispose you to hypoglycemia.

A client with DM is taking oral agents, and is scheduled for a diagnostic test that requires him to be NPO and to have contrast dye. What is the best plan by the nurse with regard to giving the client his oral medications? 1. Notify the diagnostic department and request orders 2. Notify the physician and request orders 3. Administer the oral agents immediately after the test 4. Administer the oral agents with a sip of water before the test

2. notify the physician and request orders

The client has type 1 diabetes mellitus and receives insulin. Which laboratory test will the nurse assess? 1. Potassium 2. Serum amylase 3. AST 4. Sodium

2.Potassium Insulin causes potassium to move into the cell and may cause hypokalemia.

A patient has been diagnosed with trichomoniasis. The nurse would give which medication without questioning to prescription? 1. Nitazoxanide (Alinia) 2. Praziquantel (Biltricide) 3. Metronidazole (Flagyl) 4. Chloroquine (Aralen)

3

The client asks the nurse why there aren't better drugs for human immunodeficiency virus (HIV) infection when so much money is spent on research. What is the best response by the nurse? 1. Developing new drugs is difficult because we still do not understand the virus 2. Antiviral drugs are almost impossible to create 3. Developing new drugs is difficult because the virus mutates so readily 4. Developing new drugs is difficult because people think acquired immune deficiency syndrome (AIDS) is a rare disease

3

A patient has developed chronic hepatitis after being exposed hepatitis C. The nurse is discussing treatment of this condition. What information should the nurse include? Select all that apply. 1. There is not much we can do except support your general health. 2. You should advise your family to take the hepatitis C vaccine. 3. Most current research supports the use of multidrug therapy. 4. A new drug called pegylation has been shown to prevent liver damage. 5. There are some new medications that help keep the HCV virus from reproducing.

3 and 5

A client has just recovered from a long course of therapy for shingles. The client says. "What can I do to keep from having that again?" What information should the nurse provide? Select all that apply. 1. There is little you can do to prevent outbreaks. 2. Once you have had shinges you are not likely to get it again. 3. Try to keep as physically healthy as possible 4. Try to keep you immune system strong by taking good care of your physical and emotion health. 5. Try to find healthy outlets for your stress

3, 4, and 5

A client has diabetes insipidus and receives desmopressin (DDAVP). The nurse completes medication education and evaluates that learning has occurred when the client makes which statement? 1. "This medication is a potent vasodilator; my blood pressure can fall." 2. "This medication promotes diuresis in my body; my blood pressure can fall." 3. "This medication increases water reabsorption in my kidneys." 4. "This medication suppresses hormone secretion from my posterior pituitary gland."

3. "This medication increases water reabsorption in my kidneys."

3. A patient who tested positive for HIV 3 years ago is admitted to the hospital with Pneumocystis jiroveci pneumonia (PCP). Based on diagnostic criteria established by the Centers for Disease Control and Prevention (CDC), the patient is diagnosed as having: A. Early chronic infection. B. HIV infection. C. AIDS. D. Intermediate chronic infection.

3. C. Rationale: Development of PCP pneumonia meets the diagnostic criterion for AIDS. The other responses indicate an earlier stage of HIV infection than is indicated by the PCP infection.

The client is very distraught that her 12-year-old son is of very short stature. What is the best plan by the nurse? 1. Tell the client that treatment with growth hormone might result in acromegaly. 2. Tell the client that treatment for short stature is too expensive for many to afford. 3. Tell the client that treatment with growth hormone might help, and refer her to an endocrinologist. 4. Tell the client that treatment with growth hormone can add 6 inches of height to her son.

3. Tell the client that treatment with growth hormone might help, and refer her to an endocrinologist.

The physician orders intravenous (IV) insulin for the client with a blood sugar of 563. The nurse administers insulin lispro (Humalog) intravenously (IV). What does the best evaluation by the nurse reveal? 1. The nurse used the correct insulin 2. The nurse should have contacted the physician 3. The nurse should have used regular insulin (Humulin R) 4. The nurse could have given the insulin subcutaneously

3. The nurse should have used regular insulin (Humulin R) Regular insulin is the only insulin that can be given intravenously

30. The client has just begun highly active antiretroviral therapy (HAART) therapy for the treatment of acquired immune deficiency syndrome (AIDS). Which teaching point is a priority for this client? A. The goal of highly active antiretroviral therapy (HAART) therapy is to reduce plasma human immunodeficiency virus (HIV) ribonucleic acid (RNA) to the lowest possible level. B. Knowing which medications target which phases of the human immunodeficiency virus (HIV) replication cycle. C. Taking medications as scheduled is vital to successful treatment. D. Medications must be taken for three years after viral load is not measurable.

30. Answer: C. Taking medications as scheduled is vital to successful treatment. Rationale: Taking medicines, as scheduled, is vital to maintain adequate treatment and prevent resistance to the medication. The goal of highly active antiretroviral therapy (HAART) is to reduce plasma human immunodeficiency virus (HIV), but this is not as important as medication compliance. The client may be interested in knowing which medications target which phases of the human immunodeficiency virus (HIV) ribonucleic acid (RNA) reproduction cycle, but this is not as important as medication compliance. Medications must be continued for the lifetime of the client.

31. The client with acquired immune deficiency syndrome (AIDS) asks the nurse why he must take so many medications. What is the best response by the nurse? A. "To provide you with the most effective treatment for your illness." B. "To decrease the possibility of the virus developing resistance to the medications." C. "Because the earlier we start multiple medications the better for you." D. "Research has shown single medications to be ineffective."

31. Answer: B. "To decrease the possibility of the virus developing resistance to the medications." Rationale: Decreasing the possibility of resistance is the key; single drugs can be effective, but viral resistance is a problem. Multiple medications will provide the best treatment, but this is not as good an answer as avoiding drug resistance. It is debatable if multiple drugs should be used early in the course of the disease. Single medications can be effective, but drug resistance is more likely the reason.

The order for a 4-year-old patient reads: "Give gauifenesin, 80 mg PO, every 4 hrs as needed for cough. Maximum of 600 mg/24hours." The medication comes in a bottle that has 100 mg/5ml. How many milliliters will the nurse giver per dose?

4 ml

The client has diabetes mellitus type 2. The nurse has taught the client about the illness and evaluates that learning has occurred when the client makes which statement? 1. My beta cells just cannot produce enough insulin for my cells 2. My peripheral cells have increased sensitivity to insulin 3. My cells have increased their receptors, but there is not enough insulin 4. My cells cannot use the insulin my pancreas makes

4. "My cells cannot use the insulin my pancreas makes."

5. A patient who is diagnosed with AIDS and has developed Kaposi's sarcoma tells the nurse, "I have lots of thoughts about dying. Do you think I am just being morbid?" Which response by the nurse is most appropriate? A. "Thinking about dying will not improve the course of AIDS." B. "Although your diagnosis is serious, there are more treatments available now." C. "Try to focus on the good things in life because stress impairs the immune system." D. "Tell me what kind of thoughts you have about dying."

5. D. Rationale: More assessment of the patient's psychosocial status is needed before taking any other action. The statements, "Thinking about dying will not improve the course of AIDS" and "Try to focus on the good things in life ..." discourage the patient from sharing any further information with the nurse and decrease the nurse's ability to develop a trusting relationship with the patient. The statement, "Although your diagnosis is serious, there are more treatments available now" is correct, but without further assessment, it is impossible to know whether this responds to the patient's concerns.

6. A pregnant woman with a history of asymptomatic HIV infection is seen at the clinic. Which information will the nurse include when teaching the patient? A. Although infants of HIV-infected mothers always test positive for HIV antibodies, most infants are not infected with the virus. B. Because she has not developed AIDS, the infant will not contract HIV during intrauterine life. C. The infant will be started on zidovudine (AZT) after delivery to prevent HIV infection. D. It is likely that her newborn will develop HIV infection unless she takes antiretroviral drugs during the pregnancy.

6. A. Rationale: Because antibodies are transmitted from the mother to the fetus during intrauterine life, all infants of HIV-positive mothers will test positive at birth. Ongoing antibody (or viral) testing is needed to determine whether the infant is infected with HIV. Transmission of the virus can occur during fetal life even if the mother does not have AIDS. Infants of HIV-positive mothers are not routinely started on antiretroviral therapy (ART). Only 25% of infants born to HIV-positive mothers develop HIV infection, even when the mother does not use ART during pregnancy.

7. Interventions such as promotion of nutrition, exercise, and stress reduction should be promoted by the nurse for patients who have HIV infection, primarily because these interventions will: A. Promote a feeling of well-being in the patient. B. Prevent transmission of the virus to others. C. Improve the patient's immune function. D. Increase the patient's strength and self-care ability.

7. C. Rationale: The primary goal for the patient with HIV infection is to increase immune function, and these interventions will promote a healthy immune system. They may also promote a feeling of well-being and increase strength, but these are not the priority goals for HIV-positive patients. These activities will not prevent the risk for transmission to others because the patient will still be HIV positive.

Dr. Wijangco orders insulin lispro (Humalog) 10 units for Alicia, a client with diabetes mellitus. When will the nurse administer this medication? A When the client is eating B Thirty minutes before meals C fifteen minutes before meals D When the meal trays arrive on the floor

A Question 12 Explanation: The onset action for the insulin lispro (Humalog) is 10 to 15 minutes so it must be given when the client is eating to prevent hypoglycemia. It must be given when the client is eating, not when the meal trays arrive on the floor and not thirty minutes before meals.

Which of the following is accurate pertaining to physical exercise and type 1 diabetes mellitus? A Physical exercise can slow the progression of diabetes mellitus. B Strenuous exercise is beneficial when the blood glucose is high. C Patients who take insulin and engage in strenuous physical exercise might experience hyperglycemia. D Adjusting insulin regimen allows for safe participation in all forms of exercise.

A Question 17 Explanation: Physical exercise slows the progression of diabetes mellitus because exercise has beneficial effects on carbohydrate metabolism and insulin sensitivity. Strenuous exercise can cause retinal damage and can cause hypoglycemia. Insulin and foods both must be adjusted to allow safe participation in exercise.

Serafica who has diabetes mellitus type 1 is found unresponsive in the clinical setting. Which nursing action is a priority? arrange from 1 to 4. Treat the client for hypoglycemia. Call the physician STAT. Assess the vital signs. Call a code. A 1, 2, 3, 4 B 1, 3, 2, 4 C 3, 1, 2, 4 D 4, 3, 2, 1

A Question 7 Explanation: When a patient with diabetes mellitus type 1 is unresponsive, the nurse should focus on and treat for hypoglycemia, as this is more likely than hyperglycemia. This is an emergency situation where the nurse must act before calling the physician. Vital signs should be taken after the client is treated for hypoglycemia. Assessment for ABCs should precede calling a code; there is no information that the client is not breathing.

Which of the following medications are most likely to cause hypothyroidism? (Select all that apply.) A Acetylsalicylic acid (aspirin) B Furosemide (Lasix) C Docusate sodium (Colace) D Rifampin (Rifadin

A, B, D Question 28 Explanation: Acetylsalicylic acid (aspirin), Furosemide (Lasix), and Rifampin (Rifadin) are most likely to cause hypothyroidism.

Steven John has type 1 diabetes mellitus and receives insulin. Which laboratory test will the nurse assess? A Potassium B AST (aspartate aminotransferase) C Serum amylase D Sodium

A. Question 5 Explanation: Insulin causes potassium to move into the cell and may cause hypokalemia. There is no need to monitor the sodium, serum amylase, and AST levels.

Which of the following actions might influence antibiotic absorption within the stomach? A. Taking an antacid along with the antibiotic B. Drinking a glass of water with the antibiotic C. Taking an antibiotic suspension without shaking up the medicine vial D. Taking the antibiotic just before bed

A. Taking an antacid along with the antibiotic

A client is prescribed prednisone for treatment of a type I hypersensitivity reaction. The nurse plans to monitor the client for which adverse effects? (Select all that apply.) A.Fluid retention B.Gastric distress C. Hypotension D.Infection E.Osteoporosi

A.Fluid retention B.Gastric distress D.Infection E.Osteoporosi Rationale: Prednisone is a corticosteroid that may cause fluid and sodium retention. It can cause gastric distress and irritation and usually is taken with food or an antacid. Prednisone decreases the immune response, increasing the susceptibility for infection. It can also cause osteoporosis. Hypertension (not hypotension) is an adverse effect of prednisone.

Which member of the health care team demonstrates reducing the risk for infection for a client with acquired immune deficiency syndrome (AIDS)? A.The dietary worker hands the disposable meal trays to the LPN assigned to the client. B The social worker encourages the client to verbalize about stressors at home. C A member of the housekeeping staff thoroughly cleans and disinfects the hallways near the client's room. D. The health care provider orders vital signs, including temperature, every 8 hours.

A.The dietary worker hands the disposable meal trays to the LPN assigned to the client. Rationale: The dietary worker giving the meal tray to the LPN limits the number of health care personnel entering the room, thus reducing the risk for infection. Verbalizing stressors does not reduce the risk for infection. Cleaning of bathrooms, not hallways, at least once daily by housekeeping staff reduces risk for infection. Vital signs, including temperature, should be taken every 4 hours to detect potential infection, but this does not reduce the risk for infection.

In comparing and contrasting the function of the pancreas and the cluster of cells within the pancreas, the primary function of these cells within the pancreas is to:

Act as endocrine.

34.16 Pathogenicity is different than virulence in that pathogenicity can: a. Cause a disease when pathogens are present. b. Lead to the ability of organisms to cause infection. c. Disrupt cell lining. d. Kill pathogens.

Answer B Lead to the ability of organisms to cause infection. Rationale: A. Virulence can cause a disease to be present, even in small numbers. B. Pathogenicity can lead to an organism's causing an infection. C. Cell lining is not disrupted in this process. D. Bactericidal ability leads to killing pathogens.

A client is taking chloroquine (Aralen). Before beginning therapy baseline data on the client, the nurse will need to: a. Have the client sign a consent for taking this medicine. b. Have an EKG done. c. Stop all other medications for 24 hours. d. Have the client admitted to an ICU for the first 24 hours of therapy.

Answer: b. Have an EKG done. Objective: Describe the nurse's role in the pharmacologic management of fungal, protozoal, and helminth infections. Rationale: The client needs to be assessed for preexisting cardiovascular disease, and an EKG should be done.

The nurse is administering amphotericin B (Albelcet). This medication: a. Should not be given with food. b. Should be given with plenty of fluids. c. Should be alternated with capsule and oral solution to increase effectiveness. d. Should be given with an antacid to decrease gastric upset.

Answer: b. Should be given with plenty of fluids. Objective: Describe the nurse's role in the pharmacologic management of fungal, protozoal, and helminth infections. Rationale: Food and fluids in the stomach at the time of administration greatly increases the bioavailability of the medicine.

The elderly client has hypothyroidism. Which assessment finding would the nurse report to the physician immediately?

Atrial fibrillation

An older adult patient with chronic obstructive pulmonary disease (COPD) develops bronchitis. The patient has a temperature of 39.5°C. The nurse will expect the provider to: a. obtain a sputum culture and wait for the results before prescribing an antibiotic. b. order empiric antibiotics while waiting for sputum culture results. c. treat symptomatically, because antibiotics are usually ineffective against bronchitis. d. treat the patient with more than one antibiotic without obtaining cultures.

B

Harry is a diabetic patient who is experiencing a reaction of alternating periods of nocturnal hypoglycemia and hyperglycemia. The patient might be manifesting which of the following? A Uncontrolled diabetes B Somogyi phenomenon C Brittle diabetes D Diabetes insipidus

B Question 18 Explanation: Somogyi phenomenon manifests itself with nocturnal hypoglycemia, followed by a marked increase in glucose and increase in ketones.

The client arrives in the emergency room after sustaining a chemical eye injury from a splash of battery acid. The initial nursing action is to: A. Begin visual acuity testing B. Irrigate the eye with sterile normal saline C. Swab the eye with antibiotic ointment D. Cover the eye with a pressure patch

B. Irrigate the eye with sterile normal saline Emergency care following a chemical burn to the eye includes irrigating the eye immediately with sterile normal saline or ocular irrigating solution. In the emergency department. the irrigation should be maintained for at least 10 minutes. Following this emergency treatment. visual acuity is assessed.

The nurse is reviewing the physician's orders for a client with Meniere's disease. Which diet will most likely be prescribed? A. Low-cholesterol diet B. Low-sodium diet C. Low-carbohydrate diet D. Low-fat diet

B. Low-sodium diet Dietary changes such as salt and fluid restrictions that reduce the amount of endolymphatic fluid sometimes are prescribe

A recent campaign, initiated by the Centers for Disease Control (CDC), to delay the emergence of antibiotic resistance in hospitals, has what as one of its objectives? a. Allowing patients to stop antibiotics when symptoms subside b. Allowing prescribers to develop their own prescribing guidelines c. Increased adherence to prescribed antibiotics d. Increased use of antibiotics among parents of young children

C

A nurse is assessing a client who has Addison's disease. Which of the following skin manifestations should the nurse expect to find? A. Purple striae on the chest and abdomen B. Butterfly rash across the bridge of the nose C. Bronze pigmentation of skin D. Jaundice of the face and sclera

C. Bronze pigmentation of skin Darkening of skin can be found in both exposed and unexposed parts of the body dues to a hormone deficiency caused by damage to the outer layer of the adrenal gland (adrenal cortex)

If seizures were to develop with antibiotic therapy, which of the following symptoms would most likely NOT be observed? A. Jerking muscular movements B. Difficulty breathing and biting the tongue C. Elevated blood pressure D. Loss of bladder control

C. Elevated blood pressure

Bacteriocidal drugs are those that: A. Have a high potency B. Have high effectiveness C. Kill the infectious agent D. Slow the growth of the infectious agent

C. Kill the infectious agent

The client has just begun highly active antiretroviral therapy (HAART) for the treatment of acquired immune deficiency syndrome (AIDS). Which teaching point is a priority for this client? Select one: A. Know which medications target which phases of the human immunodeficiency virus (HIV) replication cycle. B. Medications must be taken for 3 years after viral load is not measurable. C. Taking medications as scheduled is vital to successful treatment. D. The goal of highly active antiretroviral therapy (HAART) is to reduce plasma human immunodeficiency virus (HIV) ribonucleic acid (RNA) to the lowest possible level.

C. Taking medications as scheduled is vital to successful treatment.

A client has diabetes insipidus and receives desmopressin (DDAVP). The nurse completes med education and evaluates that learning has occurred when the client makes which statement? A. This medication is a potent vasodilator; my blood pressure can fall. B. This medication promotes diuresis in my body; my blood pressure can fall C. This medication increases water reabsorption in my kidneys. D. This medication supresses hormone secretion from my posterior pituitary gland.

C. This medication increases water reabsorption in my kidneys.

A client with diabetes mellitus type 1 is found unresponsive in the clinical setting. Which nursing action is a priority? Select one: A. Assess the client's vital signs. B. Call the physician STAT. C. Treat the client for hypoglycemia. D. Call a code.

C. Treat the client for hypoglycemia.

The mechanism of action of anterior pituitary agents, such as vasopressin (Pitressin), is to:

Cause renal tubules to increase permeability to water.

Cephalosporin

Cefepime (Maxipime), Cephalexin (Keflex)

Fluoroquinolone or miscellaneous

Ciprofloxacin (Cipro), Vancomycin (Vancocin)

A client has been taking thyroid hormone replacement for several years. On todays clinic visit, the nurse remarks that the client has lost 20 pounds. The client states, I was getting fat, so I started taking an extra thyroid pill every day or so. How should the nurse respond? Select all that apply. 1. Well, it certainly is working for you. 2. You should follow the dosage instructions for your thyroid medication exactly as written. 3. I'll ask your physician to increase the number of refills on your prescription. 4. That could be dangerous to your health. 5. I may ask the doctor to write a prescription for me.

Correct Answer: 2,4 Rationale 1: Thyroid medications should not be used in this manner. Rationale 2: Thyroid medications are of various strengths and should be taken as prescribed. Rationale 3: The nurse should not provide additional medication to this client. Rationale 4: Taking excess thyroid medication is dangerous. Rationale 5: The nurse should not encourage the misuse of prescription medications.

Which of the following is a symptom of Cushings syndrome? 1. Hypoglycemia 2. Vomiting 3. Moon face 4. Diarrhea

Correct Answer: 3 Rationale 1: Hypoglycemia is a symptom of Addisons disease. Rationale 2: Vomiting is a sign of Addisons disease. Rationale 3: A moon face is caused from long-term glucocorticoids. Rationale 4: Diarrhea is a symptom of Addisons disease.

The nurse is caring for a patient on a medical-surgical unit who has a fever of unknown origin. The prescriber has ordered a broad-spectrum antibiotic. Which intervention is the priority? a. Administering the antibiotic immediately b. Administering antipyretics as soon as possible c. Delaying administration of the antibiotic until the culture results are available d. Obtaining all cultures before the antibiotic is administered

D

Nurse Andy has finished teaching a client with diabetes mellitus how to administer insulin. He evaluates the learning has occurred when the client makes which statement? A "I should check my blood sugar immediately prior to the administration." B "I should provide direct pressure over the site following the injection." C "I should use the abdominal area only for insulin injections." D "I should only use calibrated insulin syringe for the injections.

D Question 10 Explanation: To ensure the correct insulin dose, a calibrated insulin syringe must be used. Insulin injections should be rotated to the arm and thigh, not just the abdominal area. There is no need to apply direct pressure over the site following an insulin injection. There is no need to check blood glucose immediately prior to the injection.

Gary has diabetes type 2. Nurse Martha has taught him about the illness and evaluates learning has occurred when the client makes which statement? A "My cells have increased their receptors, but there is enough insulin." B "My peripheral cells have increased sensitivity to insulin." C "My beta cells cannot produce enough insulin for my cells." D"My cells cannot use the insulin my pancreas makes."

D Question 15 Explanation: With type 2 diabetes mellitus, the pancreas produces insulin, but the cells cannot use it. Peripheral cells have a decreased, not an increased, sensitivity to insulin. The beta cells continue to produce insulin with type 2 diabetes. There is a decrease, not an increase, in receptor sites with type 2 diabetes.

A client in the allergy clinic develops all of these clinical manifestations after receiving an intradermal injection of an allergen. Which symptom requires the most immediate action by the nurse? A. Anxiety B. Urticaria C. Pruritus D. Stridor

D. Stridor Rationale: Stridor indicates airway involvement and warrants immediate intervention, such as use of oxygen and administration of epinephrine. Maintaining the client's airway is the highest priority. Anxiety, urticaria, and pruritus may be symptoms of a reaction, but are not the nurse's highest priority when the client is in respiratory distress.

Which of the following terms is NOT associated with the drug levodopa (Dopar)? A. Anticholinergic B. Anti-Parkinson's drug C. Dopamine D. Sympathomimetic

D. Sympathomimetic

An antibiotic responsible for causing "red-man syndrome" as an adverse effect is: A. Cefotaxime (Claforan) B. Tetracycline HCI (Sumycin, others) C. Erythromycin (E-mycin, Erythrocin) D. Vancomycin (Vancocin)

D. Vancomycin (Vancocin)

Factors contributing to acquired resistance include: A. Errors during replication of bacterial DNA B. Overuse of antibiotics C. Not taking antibiotic therapy for the prescribed length of time D. a, b, and c

D. a, b, and c

Following surgery, a client is placed on cefotaxime (Claforan). The assessment for possible adverse effects should include observing for Select one: A. headache. B. constipation. C. tachycardia. D. diarrhea.

D. diarrhea

Which antifungal drug can be given intravenously to treat severe yeast infections as well as a one-time oral dose to treat vaginal yeast infections? Nystatin (Mycostatin) Caspofungin (Cancidas) Fluconazole (Diflucan) Voriconazole (Vfend)

Fluconazole (Diflucan) Fluconazole is an antifungal drug that does not cause the major adverse effects of amphotericin when given intravenously. It is also very effective against vaginal yeast infections, and a single dose is often sufficient to treat vaginal infections.

When administering intravenous (IV) phenytoin (Dilantin), which action should the nurse perform? Flush the line with normal saline before and after administration to prevent precipitation. Administer through peripheral IV sites only. Monitor blood pressure and pulse oximetry. Obtain an infusion pump for the medication.

Flush the line with normal saline before and after administration to prevent precipitation. Phenytoin is very irritating to veins and incompatible with all fluids except normal saline. Flushing with normal saline before and after minimizes precipitation. You do not need an infusion pump when administering via IV push, and administration via central lines is preferred.

The client has type 1 diabetes mellitus and receives insulin. Which laboratory test will the nurse assess?

Insulin causes potassium to move into the cell and may cause hypokalemia.

Which instructions should the nurse discuss to reduce the gastrointestinal (GI) adverse effects of orlistat (Xenical)? Advise to take vitamin C supplement. Take the medication with an antacid. Limit dietary intake of fat. Increase fluid and fiber in the diet.

Limit dietary intake of fat. Orlistat is an anorexiant that works by blocking the absorption of fat from the GI tract. Restricting dietary intake of fat reduces the GI adverse effects associated with increased fat content in stool (flatulence, oily spotting, and fecal incontinence).

The nurse understands which of the following drugs falls under the classification of biguanides?

Metformin HCI (Glucophage)

A client, newly diagnosed with type 1 diabetes, says, "I have heard this is a bad disease. What complications could I have?" How should the nurse respond?

Problems with arteries can occur that may cause such problems as heart disease, stroke, kidney disease, or blindness."

The nurse would question a prescription for voriconazole (Vfend) if the client was taking which medication? Prednisone (Deltasone) Quinidine Clindamycin (Cleocin) Captopril (Capoten)

Quinidine The nurse would question a prescription for quinidine because both voriconazole and quinidine are metabolized by the cytochrome P-450 enzyme system. The drugs will compete for the limited number of enzymes, and one of the drugs will end up accumulating.

While completing discharge teaching for a patient prescribed an antiepileptic drug, the nurse instructs the patient of which potential complication if the medication is stopped abruptly? Rebound seizure activity Orthostatic hypotension Acute withdrawal syndrome Confusion and delirium

Rebound seizure activity Abrupt withdrawal of antiepileptic drugs can cause rebound seizure activity.

Of the types of insulin available for pharmacotherapy of diabetes mellitus (DM), the most effective form of insulin is:

Recombinant DNA.

A nurse working with patients who are diagnosed with ADHD is aware that such patients often take CNS stimulant drugs. These medications are potent with a high potential for abuse and dependence. Based on this potential, how are these medications classified? Schedule IV Schedule II Schedule I Schedule III

Schedule II CNS stimulants are the first-line drugs of choice for both ADHD and narcolepsy. They are potent drugs with a strong potential for tolerance and psychological dependence and are therefore classified as Schedule II drugs under the Controlled Substances Act.

What is the primary function of the islets of Langerhans in the pancreas?

Secretion of glucagon and insulin

Which antiparkinson drug causes an increase in the levels of dopaminergic stimulation in the central nervous system (CNS) and therefore allows a decreased dose of other medications? Carbidopa- levodopa (Sinemet) Selegiline (Eldepryl) Diphenhydramine (Benadryl) Tolcapone (Tasmar)

Selegiline (Eldepryl) Selegiline is a selective monoamine oxidase (MAO) B inhibitor that has been shown to cause an increase in the levels of dopaminergic stimulation in the CNS and thus allow the dose of levodopa to be decreased.

The client has hyperthyroidism. The nurse teaches the client to avoid which food selections?

Soy sauce

The nurse teaches a class for the public about diabetes mellitus. Which individual does the nurse assess as being at highest risk for developing diabetes?

The 42-year-old client who is 50 pounds overweight Obesity increases the likelihood of developing diabetes mellitus due to overstimulation of the endocrine system.

The mechanism of action of regular insulin is to

The action of regular insulin is to promote entry of glucose into the cells, thereby lowering glucose.

The client has diabetes type 1 and receives insulin for glycemic control. The client tells the nurse that she likes to have a glass of wine with dinner. What will the best plan by the nurse for client education include?

The alcohol could predispose you to hypoglycemia.

The client has diabetes type 1 and receives insulin for glycemic control. The client tells the nurse that she likes to have a glass of wine with dinner. What will the best plan of the nurse for client education include?

The alcohol could predispose you to hypoglycemia.

The client receives aminogluthethimide (Cytadren) therapy. What evaluation criteria will the nurse report to the physician?

The client has been on the therapy for four months.

The client injects his insulin as prescribed, but then gets busy and forgets to eat. What will the best assessment of the nurse reveal?

The client will have moist skin.

The client with diabetes mellitus type 1 is found unresponsive in the clinical setting. Which nursing action is a priority?

Treat the client for hypoglycemia.

Fluoroquinolones are effective against Pseudomonas, a microorganism often responsible for urinary tract and skin infections?

True

A patient with Parkinson's disease discusses with the nurse experiencing insomnia recently. The patient asks the nurse, "Can I take my old prescription of benzodiazepine that is prescribed to treat my insomnia?" What does the nurse know about the use of benzodiazepines in patients taking levodopa (Larodopa)? Use of benzodiazepines decreases the therapeutic effect of the levodopa and may result in an increase in the symptoms of Parkinson's disease. Use of benzodiazepines increases the therapeutic effect of the levodopa and may result in a decrease in the symptoms of Parkinson's disease. Use of benzodiazepines increases the therapeutic effect of the levodopa and may result in an increase in the symptoms of Parkinson's disease. Use of benzodiazepines decreases the therapeutic effect of the levodopa and may result in a decrease in the symptoms of Parkinson's disease.

Use of benzodiazepines decreases the therapeutic effect of the levodopa and may result in an increase in the symptoms of Parkinson's disease. Benzodiazepines interact with levodopa to cause reduced levodopa effects and an increase in the symptoms of Parkinson's disease.

A patient who has been taking antiparkinson medications for years begins to have increased symptoms on a constant basis. In documenting these symptoms, what term will the nurse use? On-off phenomenon Wearing-off phenomenon Dystonia Chorea

Wearing-off phenomenon The wearing-off phenomenon occurs when anti-Parkinson's disease medications begin to lose their effectiveness, despite maximal dosing, as the disease progresses. The symptoms return and continue on a constant basis despite treatment and medications.

In the client taking thyroid agents, the nurse will assess for which adverse effect?

Weight loss

The physician orders insulin lispro (Humalog) 10 units for the client. When will the nurse administer this medication?

When the client is eating

After a nebulizer treatment with the beta agonist albuterol, the patient complains of feeling a little "shaky". with slight tremors of the hands. The patient's heart rate is 98 beats/min, increased from the pretreatment rate of 88 BPM. The nurse knows this reaction is an a. expected adverse effect of the medication b. allergic reaction to the medication c. indication that he has received an overdose of the medication. d. idiosyncratic reaction to the medication

a

The client has type 1 diabetes mellitus and is receiving insulin intravenously. Which laboratory test should the nurse know is most important to assess? a. Potassium b. Serum amylase c. AST (aspartate aminotransferase) d. Sodium

a

The nurse is administering an intravenous dose of iron dextran. For which potential adverse effect is it most important for the nurse to monitor at this time? a. anaphylaxis b. gastrointestinal distress c. black, tarry stools d. bradycardia

a

The client receives metformin (Glucophage). What will the best plan by the nurse include with regard to patient education with this drug? (Select all that apply.) a. It decreases sugar production in the liver. b. It inhibits absorption of fats. c. It stimulates the pancreas to produce more insulin. d. It reduces insulin resistance.

a, d

The nurse is planning care for a client prescribed once-daily IV gentamicin (Garamycin) therapy. When should the nurse schedule a trough drug level to be drawn? a. 12 hours after completing the antibiotic infusion b. 30 minutes after beginning the antibiotic infusion c. 18 hours after completing the antibiotic infusion d. 60 minutes after beginning the antibiotic infusion

a. 12 hours after completing the antibiotic infusion Trough serum drug levels should be drawn at least 8 to 12 hours after the medication is infused.

A client with diabetes mellitus is taking oral agents, and is scheduled for a diagnostic test that requires him to be NPO (nothing by mouth) and to have contrast dye. What is the best plan by the nurse with regard to giving the client his oral medications? a. Notify the diagnostic department and request orders. b. Notify the physician and request orders. c. Administer the oral agents immediately after the test. d. Administer the oral agents with a sip of water before the test.

b

A client with diabetes mellitus type 1, being treated with insulin, is found unresponsive in the clinical setting. Which nursing action is a priority? a. Call a code. b. Treat the client for hypoglycemia c. Start CPR d. Give the patient some orange juice

b

A nurse administering niacin would implement which action to help to reduce adverse effects? a. give the medication with grapefruit juice. b. administer a small dose of aspirin or an NSAID 30 minutes before the niacin dose. c. Administer the medication on an empty stomach. d. have the patient increase dietary fiber intake.

b

A nurse is caring for a client that has been diagnosed with hypothyroidism. The nurse expects to find the following medication, which is used to treat hypothyroidism, on the patient's medication list: a. Propylthiouracil b. Levothyroxine c. Levophed d. Propranolol

b

A patient has been receiving an aminophylline (xanthine derivative) infusion for 24 hours. The nurse will assess for which adverse effect when assessing the patient during the infusion? a. CNS depression b. sinus tachycardia c. increased appetite d. temporary urinary retention

b

A patient is currently taking a statin. The nurse considers that the patient may have a higher risk for developing rhabdomyolysis when also taking which product? a. NSAIDs b. a fibric acid derivative c. orange juice d. fat-soluble vitamins

b

A women who is planning to become pregnant asks the nurse when she should start to take folic acid supplements. What is the nurse's best response? a. "There is no evidence to support the use of folic acid during pregnancy." b. "You should start taking it at least 1 month before you become pregnant and continue it throughout early pregnancy." c. "You need to start it as soon as you discover you are pregnant." d. "You should only take it during the last trimester, and not any earlier."

b

After the patient takes a dose of an inhaled corticosteroid, such as fluticasone (flovent), what is the most important action the patient needs to do next? a. hold the breath fro 60 seconds b. rinse out the mouth with water c. follow the corticosteroid with a bronchodilator inhaler if ordered d. Repeat the dose in 15 minutes if the patient feels short of breath.

b

The nurse is giving an antihistamine and will observe the patient for which side effects? (select all that apply) a. hypertension b. dizziness c. "hangover" effect d. drowsiness e. tachycardia f. dry mouth

b, c, d, f

The nurse is educating a client on dietary sources of folic acid. The nurse should teach the client that which food source has the highest amount of folic acid? a. apples b. dried beans c. steak d. swiss cheese

b. Dried beans Dried beans, green vegetables, and oranges are some of the common folate-containing foods.

The nurse is discussing use of antitussive medications with a client. What common adverse effect does the nurse include in the client teaching? a. Diarrhea and abdominal cramping b. Drowsiness and dizziness c. Flushing and decreased heart rate d. Tremors and palpitations

b. Drowsiness and dizziness Antitussive medications suppress cough through its action on the central nervous system, thus causing drowsiness and dizziness.

A patient is being assess before a newly ordered antilipemic medication is given. Which condition would be a potential contraindication? a. diabetes insipidus b. pulmonary fibrosis c. liver cirrhosis d. myocardial infarction

c

A patient who is taking a xanthine derivative for chronic bronchitis asks the nurse, "I miss my morning coffee. I can't wait to go home and have some." What is the nurse's response? a. "I know how you feel. I'd miss my coffee too." b. "I can get some coffee for you. I'll be right back." c. "It's important not to take coffee or other caffeinated products with this medicine as it may cause an increased heart rate as well as other problems." d. "You've been on this medicine for a few days. I can call the prescriber to ask whether you can have some coffee."

c

A client is making an appointment for allergy testing. The nurse instructs the client not take what class of medications at least 4 days before allergy testing? a. Antitussives b. Decongestants c. Antihistamines d. Bronchodilators

c. Antihistamines An allergist will usually recommend discontinuation of antihistamine drug therapy at least 4 days before allergy testing.

A nurse is providing education to a client taking two different bronchodilator medications. The nurse identifies which characteristic as the advantage of salmeterol (Serevent) over other beta2 agonists such as albuterol (Proventil)? a. Shorter onset of action b. Extended time of action c. Longer duration of action d. Quicker peak action

d. Quicker peak action Salmeterol has a longer duration of action, requiring the client to use it only twice a day instead of three or four times a day with albuterol.

The client has hypothyroidism and is treated with levothyroxine (synthroid). The nurse plans to do medication education. What will the best plan by the nurse include? a. monitor daily weights b. assess for decreased appetite c. assess weekly serum blood levels d. assess for altered sleep patterns

d. assess for altered sleep patterns

dexamethasone

long acting corticosteroid

starlix(nateglinide)

metglitnide

A client with DM is taking oral agents, and is scheduled for a diagnostic test that requires him to be NPO and to have contrast dye. What is the best plan by the nurse with regard to giving the client his oral medications?

notify the client's physician and request orders.

diabeta(glyburide)

sulfonylureas, second generation

49.22 The primary treatment for middle ear infection is: a. Topical steroids. b. Topical antibiotics. c. Systemic antibiotics. d. Systemic steroids.

Answer: C Systemic antibiotics. Rationale: A. Topical steroids are usually used for external ear infections.

36. The purpose of general pharmacotherapy for human immune virus (HIV)/AIDS is to: A. Cure the disease. B. Relieve symptoms for a longer period of time. C. Kill the virus. D. Decrease viral load.

36. Answer: B. Relieve symptoms for a longer period of time. Rationale: A. Anti-retroviral drugs cannot cure the disease B. The purpose is to remain symptom-free for longer. C. Anti-retroviral drugs will not kill the virus. D. Viral load is not decreased.

A client has been prescribed exenatide Byetta. What medication education should the nurse provide?

"You should take this medication twice each day." You may develop diarrhea while taking this drug." "This drug will help you secrete more insulin."

Before administering an LTRA medication, the nurse would assess the client for allergies to which substance? (Select all that apply.) a. cellulose b. latex c. Chlorhexidine d. providone e. lactose

a, d, e Allergies to povidone, lactose, titanium dioxide, or cellulose derivatives are important to note because these are inactive ingredients in LTRAs.

The client has type 1 diabetes mellitus and receives insulin. Which laboratory test will the nurse assess? 1. Potassium 2. Serum amylase 3. AST (aspartate aminotransferase) 4. Sodium

1 Rationale 1: Insulin causes potassium to move into the cell and may cause hypokalemia. Rationale 2: There is no need to monitor the serum amylase level. Rationale 3: There is no need to monitor the AST (aspartate aminotransferase) level. Rationale 4: There is no need to monitor the sodium level.

The physician writes orders for the client with diabetes mellitus. Which order would the nurse validate with the physician? 1. Lantus insulin 20U BID 2. Administering regular insulin 30 minutes prior to meals 3. 5 units of Humalog/10 units NPH daily 4. Metformin (Glucophage) 1000 mg per day in divided doses

1 Rationale 1: Lantus insulin is usually prescribed in once-a-day dosing so an order for BID dosing should be validated with the physician. Rationale 2: Regular insulin is administered 30 minutes before meals. Rationale 3: Humalog and NPH insulin can be mixed. Rationale 4: Metformin (Glucophage) is often prescribed in divided doses of 1000 mg per day.

A client has been prescribed oseltamivir (Tamiflu) after complaining of influenza like symptoms. What information should the nurse provide for this client? Select all that apply. 1. This is an expensive medication 2. Get this prescription filled and begin taking the medication immediately 3. This medication is given by inhalation 4. This medication will be helpful if you have influenza or a cold 5. This medication will keep you from getting the flu

1 and 2

A patient has recovered from hepatitis B. Which discharge information should the nurse provide? Select all that apply. 1. You are now immune to getting hepatitis B again. 2. Chronic hepatitis B may take years to develop 3. You should get hepatitis B vaccines annually 4. You should get one dose of interferon to boost your immunity to hepatitis B 5. You should get hepatitis immunoglobulin annually

1 and 2

The client receives amphotericin B (Fungizone) for histoplasmosis. What does the best assessment by the nurse include? 1. Serum creatinine 2. Serum sodium 3. Blood urea nitrogen 4. Serum amylase 5. Serum glucose

1 and 3

A nurse is preparing to administer prescribed medication to a client exposed to hepatitis A. Which information should the nurse provide to this client? Select all that apply. 1. This injection is meant to keep you from contracting hepatitis A. 2. I am giving you HAV vaccine. 3. You will likely need another injection or booster in 6 months. 4. I am giving you hepatitis A immunoglobulin (HAIg). 5. You will not need to worry about contracting hepatitis A for the next 5 to 8 years.

1 and 4

A patient with a severe systemic fungal infection will be treated with amphotericin B (Fungizone). What nursing interventions are required? Select all that apply. 1. Review BUN level 2. Give corticosteriods 1 hour after infusion is completed 3. Monitor for gastrointestinal bleeding 4. Administer the medication by IV push 5. Give acetaminophen before giving the amphotericin B (Fungizone)

1 and 5

An antifungal topical cream has been prescribed to treat a patient's tinea pedis. What information should the nurse provide? Select all that apply. 1. You should take precautions against spreading you disease. 2. Expect to use this medication for an extended period of time. 3. This drug is absorbed into your system and may make you nauseated 4. Don't drink coffee while you are using this drug 5. You may notice burning when you apply this medication

1, 2, and 5

A patient is prescribed chloroquine phosphate for the prevention of malaria. What information should the nurse provide regarding the time frame for administration? Select all that apply. 1. Start taking this medication 2 weeks before you go on your trip. 2. Always take an antacid when you take your dose of medication 3. Take the medication for 6 weeks after your trip is completed. 4. Take the medicine regularly during your trip 5. Take one dose now, one dose daily during your trip, and once dose once you return home.

1, 3, and 4

An infant is prescribed nystatin (Mycostatin) for treatment of infant oral candidiasis or thrush. Which information should the nurse provide to the infants caregiver in regard to this medication? Select all that apply. 1. Use a cotton swab to place medication in several parts of the infants mouth. 2. Give the medication right before a feeding 3. The breastfeeding mother may also require treatment 4. The infant should not breastfeed until this infection is gone

1, 3, and 4

A nurse has provided education regarding type 2 diabetes to a newly diagnosed client. Which statements would the nurse interpret as indicating need for additional education? Select all that apply. 1. Well, at least the medications I will be on will help me lose weight. 2. I can take an oral medication and will never have to inject myself. 3. I can increase my bodys ability to use the insulin I make by exercising regularly. 4. I have several lifestyle changes to make. 5. I dont run the risk of blindness and kidney disease like type 1 diabetics.

1,2,5 Rationale 1: Some of the medications used for type 2 diabetes cause weight gain. Rationale 2: As type 2 diabetes progresses, the cells that produce insulin may fail, causing the need for insulin. Some medications especially designed to treat type 2 diabetes are injected. Rationale 3: The activity of insulin receptors can be increased by physical exercise. Rationale 4: Lifestyle changes can help the type 2 diabetic avoid complications. Rationale 5: If type 2 diabetes is poorly managed these complications can occur.

A client, newly diagnosed with type 1 diabetes, says, I have heard this is a bad disease. What complications could I have? How should the nurse respond? Select all that apply. 1. Problems with arteries can occur that may cause such problems as heart disease, stroke, kidney disease, or blindness. 2. Lets not talk about that now, but rather focus on keeping you healthy. 3. You could have nerve problems that lead to numbness or tingling in your feet or hands. 4. One of the most serious complications is diabetic ketoacidosis. 5. You may experience inability to think and difficulty with memory.

1,3,4 Rationale 1: Arterial damage can lead to the problems listed. Rationale 2: The client is interested in this topic today and the topic should be addressed. Rationale 3: Neuropathy may occur, causing numbness, tingling, or loss of sensation in the limbs. Rationale 4: DKA is one of the most serious complications of type 1 diabetes. Rationale 5: While these symptoms may occur related to CVA, they are not primary complications of type 1 diabetes.

A pregnant women who is HIV positive is taking zidovudine. She is very concerned about her infant and says, "My doctor said they won't even test my baby until she is 2 weeks old. I wish they would do it when she is born. "What responses by the nurse are indicated? Select all that apply. 1. You can expect your baby to be tested at 1 month of age 2. The most sensitive test is one done at 1 week of age 3. The tests are much more accurate when your baby gets to be 2 weeks old 4. Since you are taking zidovudine a test at birth will not be accurate 5. Testing will continue until your baby is 1 year old

1,3,4, and 5

A client has been prescribed levothyroxine (Levothroid). What medication information should the nurse provide? Select all that apply. 1. "It may take a few weeks for you to see the full benefits from this drug." 2. "Be sure to keep all of your follow-up appointments." 3. "Take this medication at whatever time you eat your evening meal." 4. "Do not start a fiber laxative without first discussing it with your health care team." 5. "Take your calcium supplement at least 4 hours after taking this drug."

1. "It may take a few weeks for you to see the full benefits from this drug." 2. "Be sure to keep all of your follow-up appointments." 4. "Do not start a fiber laxative without first discussing it with your health care team." 5. "Take your calcium supplement at least 4 hours after taking this drug."

A client has been prescribed propylthiouracil (PTU). What medication education should the nurse provide? Select all that apply. 1. "It may take several days or weeks for you to see effects of this drug." 2. "Take this drug with meals." 3. "Do not become pregnant while taking this medication." 4. "You may experience a rash while taking this drug." 5. "You may continue to breastfeed while taking this drug."

1. "It may take several days or weeks for you to see effects of this drug." 2. "Take this drug with meals." 3. "Do not become pregnant while taking this medication." 4. "You may experience a rash while taking this drug."

The nursing instructor teaches the student nurses about the endocrine system. The nursing instructor evaluates that learning has occurred when the student nurses make which statements? Select all that apply. 1. "The hypothalamus secretes releasing hormones." 2. "Hormones released by the endocrine system influence every organ in the body." 3. "The hypothalamus is considered the master gland." 4. "The pituitary gland secretes TSH (thyroid stimulating hormone)." 5. "The endocrine system is a major controller of homeostasis."

1. "The hypothalamus secretes releasing hormones." 2. "Hormones released by the endocrine system influence every organ in the body." 4. "The pituitary gland secretes TSH (thyroid stimulating hormone)." 5. "The endocrine system is a major controller of homeostasis."

1. A patient who seeks health care for vague symptoms of fatigue and headaches has HIV testing and is found to have a positive enzyme immunoassay (EIA) for HIV antibodies. In discussing the test results with the patient, the nurse informs the patient that: A. The enzyme immunoassay test will need to be repeated to verify the results. B. A viral culture will be done to determine the progress of the disease. C. It will probably be 10 or more years before the patient develops AIDS. D. The Western blot test will need to be done to determine whether AIDS has developed.

1. A. Rationale: After an initial positive EIA test, the EIA is repeated before more specific testing such as the Western blot is done. Viral cultures are not part of HIV testing. Because the nurse does not know how recently the patient was infected, it is not appropriate to predict the time frame for AIDS development. The Western blot tests for HIV antibodies, not for AIDS.

The nurse has finished teaching a client with diabetes mellitus how to administer insulin. The nurse evaluates that learning has occurred when the client makes which statement? 1. "I should only use a calibrated syringe for the injections" 2. "I should check my blood sugar immediately prior to the administration." 3. I should use the abdominal area only for insulin injections." 4. I should provide direct pressure over the site following injection"

1."I should only use a calibrated insulin syringe for the injections." To ensure the correct insulin dose, a calibrated insulin syringe must be used.

37. The drug most likely to be ordered for the client with herpes simplex virus is which of the following? A. Methonidazole (Flagyl) B. Nystatin (Fungizone) C. Zidovudine (Retrovir) D. Acyclovir (Zovirax)

37. Answer: D. Acyclovir (Zovirax) Rationale: A. Methonidazole is an antifungal medication. B. Nystatin is an antifungal medication. C. Zidovudine is an antiviral mainly used in the treatment of HIV. D. The drug acyclovir (Zovirax) is an antiviral drug used for herpes simplex virus and influenza.

A client, newly diagnosed with type 1 diabetes, says, "I have heard this is a bad disease. What complications could I have?" How should the nurse respond? Select all that apply: 1. Problems with arteries can occur that may cause such problems as heart disease, stroke, kidney disease, or blindness 2. Let's not talk about that now, but rather focus on keeping you healthy 3. You could have nerve problems that lead to numbness or tingling in your feet or hands 4. One of the most serious complications is diabetic ketoacidosis 5. You may experience inability to think and difficulty with memory

1.Problems with arteries can occur that may cause such problems as heart disease, stroke, kidney disease, or blindness." 3.You could have nerve problems that lead to numbness or tingling in your feet or hands." 4."One of the most serious complications is diabetic ketoacidosis."

A nurse has provided education regarding type 2 diabetes to a newly diagnosed client. Which statements would the nurse interpret as indicating need for additional education? Select all that apply: 1. Well, at least the medications I will be on will help me lose weight 2. I can take an oral medication and will never have to inject myself 3. I can increase my body's ability to use the insulin make by exercising regularly 4. I have several lifestyle changes to make 5. I don't run the risk of blindness and kidney disease like type 1 diabetics

1.Well, at least the medications I will be on will help me lose weight." 2."I can take an oral medication and will never have to inject myself." 5."I don't run the risk of blindness and kidney disease like type 1 diabetics."

11. Which of these patients will the nurse working in an HIV testing and treatment clinic anticipate teaching about ART? A. A patient who is HIV negative but has unprotected sex with multiple partners B. A patient who has been HIV positive for 5 years and has cytomegalovirus (CMV) retinitis C. A patient who was infected with HIV 15 years ago and has a CD4 count of 740/µl D. An HIV-positive patient with a CD4 count of 120/µl who drinks a fifth of whiskey daily

11. B. Rationale: CMV retinitis is an AIDS-defining illness and indicates that the patient is appropriate for ART even though the HIV infection period is relatively short. An HIV-negative patient would not be offered ART. A patient with a CD4+ count in the normal range would not require ART. A patient who drinks alcohol heavily would be unlikely to be able to manage the complex drug regimen and would not be appropriate for ART despite the low CD4+ count.

12. When teaching a patient with HIV infection about ART, the nurse explains that these drugs: A. Work in various ways to decrease viral replication in the blood. B. Boost the ability of the immune system to destroy the virus. C. Destroy intracellular virus as well as lowering the viral load. D. Increase the number of CD4+ cells available to fight the HIV.

12. A. Rationale: The three groups of antiretroviral drugs work in different ways to decrease the ability of the virus to replicate. The drugs do not work by boosting the ability of the immune system or CD4 cells to fight the virus. The viral load detected in the blood is decreased with effective therapy, but intracellular virus is still present.

13. When assessing an individual who has been diagnosed with early chronic HIV infection and has a normal CD4+ count, the nurse will: A. Ask about problems with diarrhea. B. Examine the oral mucosa for lesions. C. Check neurologic orientation. D. Palpate the regional lymph nodes.

13. D. Rationale: Persistent generalized lymphadenopathy is common in the early stage of chronic infection. Diarrhea, oral lesions, and gait abnormalities would occur in the later stages of HIV infection.

14. While teaching community groups about AIDS, the nurse informs people that the most common method of transmission of the HIV virus currently is: A. Perinatal transmission to the fetus. B. Sharing equipment to inject illegal drugs. C. Transfusions with HIV-contaminated blood. D. Sexual contact with an infected partner.

14. D. Rationale: Sexual contact with an infected partner is currently the most common mode of transmission, although HIV is also spread through perinatal transmission, through sharing drug injection equipment, and through transfusions with HIV-infected blood.

15. A 24-year-old woman who uses injectable illegal drugs asks the nurse about preventing AIDS. The nurse informs the patient that the best way to reduce the risk of HIV infection from drug use is to: A. Participate in a needle-exchange program. B. Clean drug injection equipment before use. C. Ask those who share equipment to be tested for HIV. D. Avoid sexual intercourse when using injectable drugs.

15. A. Rationale: Participation in needle-exchange programs has been shown to control the rate of HIV infection. Cleaning drug equipment before use also reduces risk, but it might not be consistently practiced by individuals in withdrawal. HIV antibodies do not appear for several weeks to months after exposure, so testing drug uses would not be very effective in reducing risk for HIV exposure. It is difficult to make appropriate decisions about sexual activity when under the influence of drugs.

16. At the health promotion level of care for HIV infection, which question is most appropriate for the nurse to ask? A. "Are you having any symptoms such as severe weight loss or confusion?" B. "Are you experiencing any side effects from the antiretroviral medications? C. "Do you need any assistance to obtain antiretroviral drugs or other treatments?" D. "Do you use any injectable drugs or have sexual activity with multiple partners?"

16. D. Rationale: At the health-promotion level, the nurse screens for behaviors that might increase the risk for HIV infection and implements interventions to prevent infection (or, in the case of an already infected patient, implement interventions to prevent progression of the disease to AIDS). The other questions would be appropriate at the acute intervention level, when the patient already has significant immune compromise.

17. A patient with HIV infection has developed Mycobacterium avium complex infection. An appropriate outcome for the patient is that the patient will: A. Be free from injury. B. Maintain intact perineal skin. C. Have adequate oxygenation. D. Receive immunizations.

17. B. Rationale: The major manifestation of M. avium infection is loose, watery stools, which would increase the risk for perineal skin breakdown. The other outcomes would be appropriate for other complications (pneumonia, dementia, influenza, etc) associated with HIV infection.

18. A patient who has been treated for HIV infection for 7 years has developed fat redistribution to the trunk, with wasting of the arms, legs, and face. The nurse will anticipate teaching the patient about: A. Treatment with antifungal agents. B. A change in antiretroviral therapy. C. Foods that are higher in protein. D. The benefits of daily exercise.

18. B. Rationale: A frequent first intervention for metabolic disorders is a change in ART. Treatment with antifungal agents would not be appropriate because there is no indication of fungal infection. Changes in diet or exercise have not proven helpful for this problem.

19. The nurse is preparing to give the following medications to an HIV-positive patient who is hospitalized with PCP. Which is most important to administer at the right time? A. Nystatin (Mycostatin) tablet for vaginal candidiasis B. Aerosolized pentamadine (NebuPent) for PCP infection C. Oral acyclovir ((Zovirax to treat systemic herpes simplex D. Oral saquinavir (Inverase) to suppress HIV infection

19. D. Rationale: It is important that antiretrovirals be taken at the prescribed time every day to avoid developing drug-resistant HIV. The other medications should also be given as close as possible to the correct time, but they are not as essential to receive at the same time every day.

The client has diabetes type 1 and receives insulin for glycemic control. The client tells the nurse that she likes to have a glass of wine with dinner. What will the best plan by the nurse for client education include? 1. The alcohol could cause pancreatic disease and decrease insulin production. 2. The alcohol could predispose you to hypoglycemia. 3. The alcohol could cause serious liver disease. 4. The alcohol could predispose you to hyperglycemia.

2 Rationale 1: Alcohol can cause pancreatic disease, but the clients pancreas is not producing any insulin currently. Rationale 2: Alcohol can potentiate hypoglycemic effects in the client. Rationale 3: Alcohol can cause liver disease, but the more immediate concern is hypoglycemia. Rationale 4: Alcohol can potentiate hypoglycemic, not hyperglycemic, effects in the client.

A client with diabetes mellitus type 1 is found unresponsive in the clinical setting. Which nursing action is a priority? 1. Call a code. 2. Treat the client for hypoglycemia. 3. Call the physician STAT. 4. Assess the clients vital signs.

2 Rationale 1: Assessment for ABCs should precede calling a code; there is no information that the client is not breathing. Rationale 2: When a client with diabetes mellitus type 1 is found unresponsive, the nurse should focus on and treat for hypoglycemia, as this is more likely than hyperglycemia. Rationale 3: .This is an emergency situation where the nurse must act before calling the physician. Rationale 4: Vital signs should be taken after the client is treated for hypoglycemia.

Insulin is released when 1. blood glucose stays the same. 2. blood glucose increases. 3. blood glucose decreases. 4. glucagon increases.

2 Rationale 1: Insulin would not be released. Rationale 2: Insulin is released when blood glucose increases. Rationale 3: Glucagon is released when glucose decreases. Rationale 4: Glucagon increases when insulin is not needed.

The mechanism of action of regular insulin is to 1. stimulate the pancreas to produce insulin. 2. promote entry of glucose into the cells. 3. promote the entry of glucose into the bloodstream. 4. stimulate the pancreas to secrete more insulin.

2 Rationale 1: Oral hypoglycemic drugs, such as glipizide, stimulate the pancreas to produce insulin. Rationale 2: The action of regular insulin is to promote entry of glucose into the cells, thereby lowering glucose. Rationale 3: Insulin would not promote glucose into the bloodstream. Rationale 4: Oral hypoglycemic drugs, such as glipizide, stimulate the pancreas to secrete insulin.

The nurse teaches a class for the public about diabetes mellitus. Which individual does the nurse assess as being at highest risk for developing diabetes? 1. The 38-year-old client who smokes one pack of cigarettes per day 2. The 42-year-old client who is 50 pounds overweight 3. The 50-year-old client who does not get any physical exercise 4. The 56-year-old client who drinks three glasses of wine each evening

2 Rationale 1: Smoking is a serious health concern, but is not a specific risk factor for diabetes. Rationale 2: Obesity increases the likelihood of developing diabetes mellitus due to overstimulation of the endocrine system. Rationale 3: Exercise is important, but a lack of exercise is not as big of a risk factor as obesity. Rationale 4: Consuming alcohol is associated with liver disease, but is not as high a risk factor for diabetes as obesity.

The physician orders insulin lispro (Humalog), 10 units for the client. When will the nurse administer this medication? 1. Thirty minutes before meals 2. Five minutes before a meal 3. When the meal trays arrive on the floor 4. Fifteen minutes after meals

2 Rationale 1: The onset of action for insulin lispro (Humalog) is 10 to 15 minutes so it must be given when the client is eating, not 30 minutes before meals, to prevent hypoglycemia. Rationale 2: The onset of action for insulin lispro (Humalog) is 10 to 15 minutes so it must be given 5 to 10 minutes before the client eats. Rationale 3: The onset of action for insulin lispro (Humalog) is 10 to 15 minutes so it must be given just before the client eats, not when meal trays arrive on the floor, to prevent hypoglycemia. Rationale 4: The onset of action for insulin lispro (Humalog) is 10 to 15 minutes so it must be given when the client is eating, not 15 minutes after meals, to prevent hypoglycemia.

39. The nurse is preparing to discharge a client on an antiretroviral agent. Which of the following should the nurse include? A. Practice good handwashing. B. Take your blood pressure daily. C. Take your pulse daily. D. Weigh yourself daily

39. Answer: A. Practice good handwashing. Rationale: A. Handwashing is a technique the client and family members can use to control infection. B. Taking a blood pressure daily temperature is not necessary. C. Taking the pulse is not necessary. D. Daily weight is not necessary.

the nurse is teaching the client about newly prescribed medication. Which statement made by the client would indicated the need for further medication education? 1. "the liquid form of the drug will be absorbed faster than the tablets." 2. "If i take more, I'll have a better response" 3. "taking this drug with food will decrease how much gets into my system." 4. "I can consult my health care provider if i experience unexpected adverse effects"

2. "If i take more, i'll have a better response. Rationale: Although taking a larger dose of a medication usually results in a greater therapeutic response, the response also depends on the drugs plasma concentration and could reach a toxic level. Answers 1, 3, and 4 are all true statements

A client has been taking thyroid hormone replacement for several years. On today's clinic visit, the nurse remarks that the client has lost 20 pounds. The client states, "I was getting fat, so I started taking an extra thyroid pill every day or so." How should the nurse respond? Select all that apply. 1. "Well, it certainly is working for you." 2. "You should follow the dosage instructions for your thyroid medication exactly as written." 3. "I'll ask your physician to increase the number of refills on your prescription." 4. "That could be dangerous to your health." 5. "I may ask the doctor to write a prescription for me."

2. "You should follow the dosage instructions for your thyroid medication exactly as written." 4. "That could be dangerous to your health."

A client has been prescribed exenatide (Byetta). What medication education should the nurse provide? Select all that apply: 1. Drink an 8 oz glass of water with this pill 2. You should take this medication twice each day 3. You may experience dryness of the mouth while taking this drug 4. You may develop diarrhea while taking this drug 5. This drug will help you secrete more insulin

2."You should take this medication twice each day." 4.You may develop diarrhea while taking this drug." 5."This drug will help you secrete more insulin."

A nurse is caring for a client who had a thyroidectomy to treat hyperthyroidism caused by an adenoma. Which of the following findings should the nurse report to the provider? (select all that apply) A. Tachycardia and hypertension B. Respiratory rate 16/min C. Negative Chvostek's sign D. Laryngeal stridor and hoarseness E. Positive Trousseau's sign

A, D, E

The mother of a 4-year-old boy states, "I can't believe my son has type 1 diabetes. We eat well and I was so careful during the pregnancy. What could have caused this?" How should the nurse respond? Select all that apply: 1. There must've been something you were exposed to during your pregnancy that caused it. 2. Are there others in your family that have type 1 diabetes? 3. He must have been allowed to eat too much sugar. 4. We are not certain what causes type 1 diabetes 5. It is thought to be a combination of factors

2.Are there others in your family that have type 1 diabetes?" 4.We are not certain what causes type 1 diabetes." 5."It is thought to be a combination of factors."

The client has been diagnosed with diabetes mellitus type 1. He asks the nurse what this means. What is the best response by the nurse? Select all that apply: 1. The exocrine function of your pancreas is to secrete insulin and it is not working. 2. Without insulin you will develop ketoacidosis (DKA)." 3. "The endocrine function of your pancreas is to secrete insulin, but it isn't working." 4. Your alpha cells should be able to secrete insulin, but it isn't working. 5. "It means your pancreas cannot secrete insulin."

2.Without insulin you will develop ketoacidosis (DKA)." 3"The endocrine function of your pancreas is to secrete insulin, but it isn't working." 5"It means your pancreas cannot secrete insulin."

20. To evaluate the effectiveness of ART, the nurse will schedule the patient for: A. Viral load testing. B. Enzyme immunoassay. C. Rapid HIV antibody testing. D. Immunofluorescence assay.

20. A. Rationale: The effectiveness of ART is measured by the decrease in the amount of virus detectable in the blood. The other tests are used to detect for HIV antibodies, which remain positive even with effective ART.

21. When designing a program to decrease the incidence of HIV infection in the community, the nurse will prioritize education about: A. How to prevent transmission between sexual partners. B. Methods to prevent perinatal HIV transmission. C. Ways to sterilize needles used by injectable drug users. D. Means to prevent transmission through blood transfusions.

21. A. Rationale: Sexual transmission is the most common way that HIV is transmitted. The nurse should also provide education about perinatal transmission, needle sterilization, and blood transfusion, but the rate of HIV infection associated with these situations is lower.

22. The nurse explains to the patient newly diagnosed with HIV that prophylactic measures that should be taken as early as possible during the course of the infection include which the following (Select all that apply.)? A. Hepatitis A vaccine B. Hepatitis B vaccine C. Pneumococcal vaccine D. Influenza virus vaccine E. Trimethoprim-sulfamethoxazole F. Varicella zoster immune globulin

22. A, B, C, D. Rationale: Prevention of other infections is an important intervention in patients who are HIV positive, and these vaccines are recommended as soon as the HIV infection is diagnosed. Antibiotics and immune globulin are used to prevent and treat infections that occur later in the course of the disease, when the CD4 count has dropped or when infection has occurred.

23. The nurse plans to teach the client with acquired immune deficiency syndrome (AIDS) about bacterial infections. Which statement describes the best plan of the nurse? Select all that apply. A. "Pathogenicity means the bacteria can cause an infection." B. "Pathogens are divided into two classes, bacteria and viruses." C. "If just a few bacteria make you sick, this is virulence." D. "Actually, most bacteria will not harm us." E. "Most bacteria have developed antibiotic resistance."

23. Answer: A, C, D. Rationale: The ability of an organism to cause infection is called pathogenicity. A highly virulent microbe is one that can produce disease when present in minute numbers. Only a few dozen pathogens commonly cause disease in humans; most are harmless. Human pathogens include viruses, bacteria, fungi, unicellular organisms, and multicellular animals. Antibiotic resistance is a problem; however only a few, not most, bacteria have not developed it.

24. The nurse educates clients with acquired immune deficiency syndrome (AIDS) about the nature of viruses. The nurse evaluates that learning has occurred when the clients make which response(s). Select all that apply. A. "Viruses are nonliving particles." B. "Viruses are intracellular parasites." C. "A virion is a mature virus." D. "The structure of viruses is complex." E. "Viruses can infect plants as well as animals."

24. Answer: A, B, C, E. Rationale: Viruses are nonliving agents that infect bacteria, plants, and animals. Viruses must use intracellular machinery to replicate, so are called intracellular parasites. A mature infective particle is called a virion. Viruses can infect plants as well as animals. The structure of viruses is quite primitive compared to the simplest cell.

25. The client asks the nurse why there aren't better drugs for human immunodeficiency virus (HIV) infection when so much money is spent on research. What is the best response by the nurse? A. "Developing new drugs is difficult because people think acquired immune deficiency syndrome (AIDS) is a "gay" disease." B. "Developing new drugs is difficult because the virus mutates so readily." C. "Developing new drugs is difficult because we still do not understand the virus." D. "Developing new drugs is difficult because we still do not have enough money."

25. Answer: B "Developing new drugs is difficult because the virus mutates so readily." Rationale: Antiviral pharmacotherapy can be extremely challenging because of the rapid mutation rate of viruses, which can quickly render drugs ineffective. Most people recognize human immunodeficiency virus (HIV) infection as a heterosexual, not homosexual, disease. After more then 20 years of research, the virus is mostly understood. More money is spent on human immunodeficiency virus (HIV) infection research than just about any other illness.

26. The client receives zidovudine (Retrovir) for treatment of human immunodeficiency virus (HIV) infection. Which assessment data indicates an adverse reaction to zidovudine (Retrovir)? A. Decreased white blood count (WBC) B. Enlarged lymph nodes C. Fever D. Edema

26. Answer: A. Decreased white blood count (WBC). Rationale: Bone marrow suppression is a common adverse effect of zidovudine (Retrovir), and decreased white blood cells (WBCs) are indicative of bone marrow suppression. Enlarged lymph nodes are not an adverse effect of zidovudine (Retrovir). Fever is the result of infection, not the use of zidovudine (Retrovir). Edema is not an adverse effect of zidovudine (Retrovir).

27. The nurse plans to teach a client with human immunodeficiency virus (HIV) infection about zidovudine (Retrovir). What will the best plan of the nurse include? Select all that apply. A. Zidovudine (Retrovir) will need to be stopped if bone marrow depression occurs. B. Zidovudine (Retrovir) therapy frequently results in the development of anemia. C. Zidovudine (Retrovir) prevents spread of the virus through sexual contact. D. Zidovudine (Retrovir) will slow the disease, but not cure it. E. Zidovudine (Retrovir) was the first drug developed to treat acquired immune deficiency syndrome (AIDS)

27. Answer: A, B, D, E. Rationale: Stopping zidovudine (Retrovir) if bone marrow depression occurs allows the bone marrow time to recover. Treatment with zidovudine (Retrovir) frequently results in anemia. The drug only slows the disease; it will not cure it. Zidovudine (Retrovir) was the first drug developed to treat AIDS. Zidovudine (Retrovir) will slow the progression of the disease; it will not prevent its transmission.

28. The nurse provides care for clients with acquired immune deficiency syndrome (AIDS). Which laboratory test is the best indicator of effective treatment with antiviral medications? A. T4 lymphocyte count B. CD4 count C. Viral load D. Absolute neutrophil count

28. Answer: C. Viral load. Rationale: The viral load is an actual count of viral presence, and is the best test. The viral load, not a T4 lymphocyte count, is the best test. The viral load, not a CD4 count, is the best test. The viral load, not an absolute neutrophil count, is the best test.

29. The nurse teaches clients with acquired immune deficiency syndrome (AIDS) about the importance of taking their medications as prescribed. What does the nurse recognize as the primary factor for medication noncompliance in the acquired immune deficiency syndrome (AIDS) population? A. The necessity of having to take multiple medications throughout the day B. The difficulty with availability of medications to treat acquired immune deficiency syndrome (AIDS). C. The unpleasant side effects that are associated with the medications D. A lack of understanding for the reason to take the medications

29. Answer: A. The necessity of having to take multiple medications throughout the day Rationale: Multiple medications are required throughout the day. The higher the number of medications taken daily equates with a higher noncompliance rate. Medications are generally available for clients with acquired immune deficiency syndrome (AIDS) today. Most clients with acquired immune deficiency syndrome (AIDS) recognize the importance of taking medications in spite of side effects. A lack of understanding could be a factor, but it is the schedule of taking multiple drugs that leads to noncompliance.

The client has a fungal infection of the toenails and receives oral terbinafine (Lamisil). The client asks the nurse how a pill will heal his nail infection. What is the best response by the nurse? 1. The medication works by destroying circulating fungi in your blood 2. The medication works by destroying toxins excreted by the fungi in your nails 3. The medication accumulates in your nail beds and is there for many months 4. The medication should be combined with topical agent to increase effectiveness

3

The client has just begun highly active antiretroviral therapy (HAART) for the treatment of acquired immune deficiency syndrome (AIDS). Which teaching oin is a priority for this client? 1. Know which medications target which phases of the human immunodeficiency virus (HIV) replication cycle. 2. Medications must be taken for 3 years after viral load is no measurable 3. Taking medications as scheduled is vital to successful treatment 4. The goal of highly active antiretroviral therapy (HAART) is to reduce plasma human immunodeficiency virus (HIV) ribonucleic acid (RNA) to the lowest possible level

3

The client is being treated for pinworms with mebendazole (Vermox). The nurse has completed medication education and evaluates that learning has occurred when the client makes which statement? 1. I need to have three negative stool cultures before I am cured. 2. I must avoid aspirin while I am on this medication. 3. I should chew this tablet. 4. I need high-fiber foods to help with passage of the worms.

3

Which sign or symptom is most typical of an untreated client with type 1 diabetes? 1. Increased energy 2. Weight gain 3. Fatigue 4. Decreased hunger

3 Rationale 1: Clients with type 1 DM do not experience increased energy; a typical sign/symptom is fatigue. Rationale 2: Clients with type 1 DM typically experience weight loss as opposed to weight gain. Rationale 3: Fatigue is a typical sign/symptom of type 1 DM due to sustained hyperglycemia. Rationale 4: Clients with type 1 DM typically experience polyphagiaincreased hungeras opposed to decreased hunger.

What is the primary function of the islets of Langerhans in the pancreas? 1. Secretion of enzymes 2. Acting as exocrine 3. Secretion of glucagon and insulin 4. Absorption of insulin

3 Rationale 1: Secretion of enzymes relates to the exocrine function, which is to release enzymes for chemical digestion of nutrients. Rationale 2: Other cells of the pancreas are responsible for exocrine function. Rationale 3: The cluster of cells within the pancreas (islets of Langerhans) is responsible for the endocrine function, which is to release insulin and glucagon. Rationale 4: These are secretory cells, not absorptive cells.

The client has type 1 diabetes and receives insulin. He asks the nurse why he cant just take pills instead. What is the best response by the nurse? 1. I know it is tough, but you will get used to the shots soon. 2. Have you talked to your doctor about taking pills instead? 3. Insulin cant be in a pill because it is destroyed in stomach acid. 4. Insulin must be injected because it needs to work quickly.

3 Rationale 1: Telling the client he will get used to the shots does not answer his question and is condescending. Rationale 2: The nurse should answer the clients question, not refer him back to the physician. Rationale 3: Insulin must be injected because it is destroyed in stomach acid if taken orally. Rationale 4: Insulin must be injected because it is destroyed in stomach acid if taken orally; the onset of action is not the issue here.

The physician orders intravenous (IV) insulin for the client with a blood sugar of 563. The nurse administers insulin lispro (Humalog) intravenously (IV). What does the best evaluation by the nurse reveal? 1. The nurse used the correct insulin. 2. The nurse should have contacted the physician. 3. The nurse should have used regular insulin (Humulin R). 4. The nurse could have given the insulin subcutaneously.

3 Rationale 1: The nurse did not use the correct insulin as it was not regular insulin. Rationale 2: There was no need to contact the physician; regular insulin is the only insulin that can be given intravenously (IV). The physician should be contacted now. Rationale 3: Regular insulin is the only insulin that can be given intravenously (IV). Rationale 4: The nurse cannot give the insulin subcutaneously when it is ordered to be given intravenously (IV).

Which of the following is a symptom of Cushing's syndrome? 1. Hypoglycemia 2. Vomiting 3. Moon face 4. Diarrhea

3. Moon face

The primary goal of pharmacotherapy in hyperthyroidism (Graves' disease) is to 1. increase metabolism. 2. increase synthesis of corticosteroid. 3. lower the activity of the thyroid hormone. 4. increase the activity of the thyroid hormone.

3. lower the activity of the thyroid hormone.

The client injects his insulin as prescribed, but then gets busy and forgets to eat. What is the nurse's most likely assessment finding? 1. The client will be very thirsty 2. The client will need to urinate 3. The client will have moist skin 4. The client will complain of nausea

3. the client will have moist skin

The client is receiving multiple medications, including one drug specifically used to stimulate gastric peristalsis. The nurse knows that this drug could have which influence on additional oral medications? 1. Increased absorption 2. Reduced excretion 3. Decreased absorption 4. enhanced distribution

3.Decreased absorption Rationale: Peristalsis is the wavelike muscular contraction that pushes food into the stomach and helps to mix the stomach contents. An increase in this activity would decrease the time that drugs would remain in the GI system, and decrease absorption. Excretion of the drugs occur mostly in the kidneys, lungs, and glands. Peristalsis would not reduce excretions.A delay in peristalsis would prolong absorption time, and preistalsis is not involved in the distribution of drugs to their target sites

35. The client receives delavirdine (Rescriptor) as treatment for acquired immune deficiency syndrome (AIDS). The nurse assesses the client for which serious adverse effect? A. Bone marrow suppression B. Cardiac arrest C. Seizures D. Rash

35. Answer: D. Rash. Rationale: A serious adverse effect of delavirdine (Rescriptor) is Stevens-Johnson rash, which can be life threatening. Stevens-Johnson rash, not bone marrow suppression, is an adverse effect of delavirdine (Rescriptor). Stevens-Johnson rash, not cardiac arrest, is an adverse effect of delavirdine (Rescriptor). Stevens-Johnson rash, not seizures, is an adverse effect of delavirdine (Rescriptor).

32. The nurse does health teaching with a client who has acquired immune deficiency syndrome (AIDS) and who has been started on antiviral medications. The nurse recognizes teaching has been effective when the client makes which statement? A. "I will still need to take precautions to avoid spreading the virus to others." B. "I will need to be on a high-calorie diet to enhance the effect of the medication." C. "I will need to limit my travel to avoid people with other infections." D. "I will not be able to continue working at my high-stress job anymore."

32. Answer: A. "I will still need to take precautions to avoid spreading the virus to others." Rationale: Precautions are still necessary to avoid spreading the virus to others. A well-balanced diet is the best diet for a client with acquired immune deficiency syndrome (AIDS). There is no need to limit travel at this point. A high-stress job is not ideal, but at this point the client doesn't need to stop working.

33. The client has acquired immune deficiency syndrome (AIDS) and has just learned she is pregnant. She tearfully asks the nurse if her baby will die of acquired immune deficiency syndrome (AIDS). What is the best outcome for this client? A. The client will take penciclovir (Denavir) as prescribed. B. The client will take indinavir (Crixivan) as prescribed. C. The client will take zidovudine (Retrovir) as prescribed. D. The client will take tipranavir (Aptivus) as prescribed.

33. Answer: C. The client will take zidovudine (Retrovir) as prescribed. Rationale: A regime of oral zidovudine (Retrovir) will decrease the possibility of mother-to-baby transmission by 70%. Zidovudine (Retrovir), not penciclovir (Denavir), will decrease the possibility of mother-to-baby transmission by 70%. Zidovudine (Retrovir), not indinavir (Crixivan), will decrease the possibility of mother-to-baby transmission by 70%. Zidovudine (Retrovir), not tipranavir (Aptivus), will decrease the possibility of mother-to-baby transmission by 70%.

34. The nurse accidentally sticks herself with a needle after starting an intravenous (IV) line on a client with acquired immune deficiency syndrome (AIDS). The nurse tells the supervisor about the accident. What is the best initial response by the supervisor to decrease anxiety in the nurse? A. "Fortunately, the chances of you contracting human immunodeficiency virus (HIV) after the stick are very small." B. "If you are started on medications soon, it will decrease the severity of the disease." C. "Workers compensation will cover the cost of your illness and medications." D. "Did you use the hospital protocols for starting intravenous (IV) lines on a client with acquired immune deficiency syndrome (AIDS)?"

34. Answer: A. "Fortunately, the chances of you contracting human immunodeficiency virus (HIV) after the stick are very small." Rationale: There have been only about 56 cases of client-to-healthcare worker transmission; the risk is very small. The risk of transmission of human immunodeficiency virus (HIV) from client-to-healthcare worker is very small, and telling the nurse to start on medication, will not reduce anxiety. If the nurse did contract human immunodeficiency virus (HIV), workers compensation would cover the cost, but the risk of human immunodeficiency virus (HIV) transmission is small and this response will not reduce anxiety. Asking the nurse if she used protocols is very non-therapeutic at this point, and will most likely increase anxiety.

The client has diabetes mellitus type 2. The nurse has taught the client about the illness and evaluates that learning has occurred when the client makes which statement? 1. My beta cells just cannot produce enough insulin for my cells. 2. My peripheral cells have increased sensitivity to insulin. 3. My cells have increased their receptors, but there is not enough insulin. 4. My cells cannot use the insulin my pancreas makes.

4 Rationale 1: The beta cells continue to produce insulin with type 2 diabetes. Rationale 2: Peripheral cells have a decreased, not an increased, sensitivity to insulin. Rationale 3: There is a decrease, not an increase, in receptor sites with type 2 diabetes. Rationale 4: With type 2 diabetes mellitus, the pancreas produces insulin, but the cells cannot use it.

The nurse plans to administer intranasal desmopressin (DDAVP) to the client. What will the best plan by the nurse include? 1. Instruct the client to blow his nose following administration. 2. Be sure to have fresh water at the bedside. 3. Withhold other medications so absorption of desmopressin (DDAVP) will not be affected. 4. Direct the spray high into the nasal cavity.

4. Direct the spray high into the nasal cavity.

The Nurse is caring for a client with hepatitis and resulting in hepatic impairment. The nurse would expect the duration of action for most medications to: 1. Decrease 2. Improve 3. Be unaffected 4. Increase

4. Increase Rationale: the length of time a drug concentration remains in the theraeutic range is its duration of action. Clients with hepatic impairment do not effectively metabolize drugs, which increases the duration of action. Options 1,2,3 are incorrect. In clients with hepatic disease, the duration of action most likely will increase since drug metabolism is impaired. Although the duration of action is extended, the effects of the drug are not improved.

9. A patient has recently tested positive for HIV and asks the nurse about drug therapy for HIV infection. The nurse informs the patient that: A. Drug therapy for HIV is indicated only for patients whose CD4+ cell counts indicate that AIDS has developed. B. Medication therapy is delayed as long as possible to prevent development of viral resistance to the drugs. C. Treatment is individualized based on CD4+ counts, the amount of virus in the blood, and the patient's wishes. D. ART is typically started soon after HIV diagnosis to prevent progression of the disease.

9. C. Rationale: ART is typically considered when the CD4+ count drops below normal levels or the viral load is high in patients who are appropriate for ART and desire ART. ART is used to prevent the progression to AIDS and is used in patients who have AIDS. ART is not delayed as long as possible but can be started when the CD4+ counts are relatively high in some patients. ART is not started soon after HIV diagnosis; rather, it is started when CD4+ count, viral load, or patient symptoms indicate that it will be beneficial.

Gregory is a 52-year-old man identified as high-risk for diabetes mellitus. Which laboratory test should a nurse anticipate a physician would order for him? (Select all that apply.) A Fasting Plasma Glucose (FPG) B Two-hour Oral Glucose Tolerance Test (OGTT) C Glycosylated hemoglobin (HbA1C) D Finger stick glucose three times daily

A, B Question 23 Explanation: When an older person is identified as high-risk for diabetes, appropriate testing would include FPG and OGTT. An FPG greater than 140 mg/dL usually indicates diabetes. The OGTT is to determine how the body responds to the ingestion of carbohydrates in a meal. HbA1C evaluates long-term glucose control. A finger stick glucose three times daily spot-checks blood glucose levels.

Which nursing interventions are appropriate for a client recovering from surgery for retinal detachment? Select all that apply. A. Monitor for hemorrhage. B. Administer eye medications. C. Maintain the eye patch or shield. D. Assist with activities of daily living. E. Encourage coughing and deep breathing. F. Educate regarding symptoms of retinal detachment

A, B, C, D, F An eye patch or shield is applied to protect the eye and prevent any further detachment. Educating the client regarding symptoms is necessary because the client is at risk for subsequent retinal detachment. Positioning. activity restrictions. and eye patches hinder the client in the performance of activities of daily living. and the client needs the nurse's assistance with these activities. Eye medications are prescribed postoperatively. and hemorrhage is also a risk post surgery. Coughing is not encouraged because this can increase intraocular pressure and harm the client.

According to the National Diabetes Statistics Report, diabetes remains as one of the leading causes of death in the United States since 2010. Which of the following factors are risks for the development of diabetes mellitus? Select all that apply. A Age over 45 years B Overweight with a waist/hip ratio >1 C Having a consistent HDL level above 40 mg/dl D Maintaining a sedentary lifestyle

A, B, D Question 24 Explanation: Aging results in reduced ability of beta cells to respond with insulin effectively. Overweight with waist/hip ratio increase is part of the metabolic syndrome of DM II. There is an increase in atherosclerosis with DM due to the metabolic syndrome and sedentary lifestyle. Question 25 WRONG

The nurse is preparing a teaching plan for a client who is undergoing cataract extraction with intraocular implant. Which home care measures will the nurse include in the plan? Select all that apply. A. To avoid activities that require bending over B. To contact the surgeon if eye scratchiness occurs C. To place an eye shield on the surgical eye at bedtime D. That episodes of sudden severe pain in the eye is expected E. To contact the surgeon if a decrease in visual acuity occurs F. To take acetaminophen (Tylenol) for minor eye discomfort

A, C, E, F After eye surgery. some scratchiness and mild eye discomfort may occur in the operative eye and is usually relieved by mild analgesics. If the eye pain becomes severe. the client should notify the surgeon because this may indicate hemorrhage. infection. or increased intraocular pressure. The nurse would also instruct the client to notify the surgeon of purulent drainage. increased redness. or any decrease in visual acuity. The client is instructed to place an eye shield over the operative eye at bedtime to protect the eye from injury during sleep and to avoid activities that increase intraocular pressure such as bending over.

Dr. Shrunk orders intravenous (IV) insulin for Rita, a client with a blood sugar of 563. Nurse AJ administers insulin lispro (Humalog) intravenously (IV). What does the best evaluation of the nurse reveal? Select all that apply. A The nurse could have given the insulin subcutaneously. B The nurse should have contacted the physician. C The nurse should have used regular insulin (Humulin R). D The nurse used the correct insulin.

A, D: Question 2 Explanation: Regular insulin is the only insulin that can be given intravenously (IV). The nurse did not use correct insulin as it was not regular insulin. Contact the provider to clarify the order, regular insulin is the only insulin that can be given intravenously (IV). The nurse cannot give the insulin subcutaneously when it is ordered to be given intravenously (IV).

The client with glaucoma asks the nurse is complete vision will return. The most appropriate response is: A. "Although some vision as been lost and cannot be restored. further loss may be prevented by adhering to the treatment plan." B. "Your vision will return as soon as the medications begin to work." C. "Your vision will never return to normal." D. "Your vision loss is temporary and will return in about 3-4 weeks."

A. "Although some vision as been lost and cannot be restored. further loss may be prevented by adhering to the treatment plan." Vision loss to glaucoma is irreparable. The client should be reassured that although some vision has been lost and cannot be restored. further loss may be prevented by adhering to the treatment plan. Option C does not provide reassurance to the client.

When preparing a client for allergy testing, the nurse provides the client with which instruction? A. "Antihistamines should be discontinued 2 weeks before the test to avoid suppressing the test response." B. "It is okay to use your fluticasone propionate (Flonase) nasal spray before testing." C. "Aspirin in a low dose may be taken before testing." D. "You can take antihistamine nasal sprays before testing."

A. "Antihistamines should be discontinued 2 weeks before the test to avoid suppressing the test response." Rationale: Systemic glucocorticoids and antihistamines are discontinued 2 weeks before the test to avoid suppressing the test response. Nasal sprays to reduce mucous membrane swelling are permitted, except for sprays that contain an antihistamine. Allergists recommend that aspirin be withheld before testing

Which dietary change does the nurse suggest for the client who has diarrhea associated with HIV disease? A. "Avoid fatty foods." B. "Increase your intake of fiber." C. "Take an antacid 30 minutes before each meal." D. "Restrict your intake of fluids to 1 liter per day."

A. "Avoid fatty foods." Rationale: Fatty foods tend to reduce intestinal absorption and make diarrhea worse. Although fiber can prevent some types of diarrhea, it has no benefit for the chronic diarrhea associated with AIDS.

In discharging a client diagnosed with acquired immune deficiency syndrome (AIDS), which statement by the nurse uses a nonjudgmental approach in discussing sexual practices and behaviors? A. "Have you had sex with men or women or both?" B. "I hope you use condoms to protect your partners." C. "You must tell me all of your partners' names, so I can let them know about possibly having AIDS." D. "You must tell me if you have a history of any sexually transmitted diseases because the public health department needs to know."

A. "Have you had sex with men or women or both?" Rationale: The straightforward approach of asking the client about having sex with men or women is nonjudgmental and most appropriate. "I hope you use..." is a judgmental statement. Naming partners is voluntary; also, assuming that more than one partner exists is judgmental and presumptuous. Asking for information in the name of the public health department is not straightforward, and the tone of this entire statement is judgmental.

The nurse presents a seminar on human immune deficiency virus (HIV) testing to a group of seniors and their caregivers in an assisted-living facility. Which responses fit the recommendations of the Centers for Disease Control and Prevention regarding HIV testing? (Select all that apply.) A. "I am 78 years old, and I was treated and cured of syphilis many years ago." B. "In 1986, I received a transfusion of platelets." C. "Seven years ago, I was released from a penitentiary." D. "I used to smoke marijuana 30 years ago, but I have not done any drugs since that time." E. "At 68, I am going to get married for the fourth time.

A. "I am 78 years old, and I was treated and cured of syphilis many years ago." C. "Seven years ago, I was released from a penitentiary. E. "At 68, I am going to get married for the fourth time. Rationale: People who have had a sexually transmitted disease should be tested. People who are in or have been in correctional institutions such as jails or prisons and people who are planning to get married should be tested for HIV. HIV testing is recommended for clients who received a blood transfusion between 1978 and 1985. People who have used injectable drugs (not marijuana) should be tested.

Acarbose (Precose), an alpha-glucosidase inhibitor, is prescribed for a female client with type 2 diabetes mellitus. During discharge planning, nurse Pauleen would be aware of the client's need for additional teaching when the client states: A. "If I have hypoglycemia, I should eat some sugar, not dextrose." B. "The drug makes my pancreas release more insulin." C. "I should never take insulin while I'm taking this drug." D. "It's best if I take the drug with the first bite of a meal."

A. "If I have hypoglycemia, I should eat some sugar, not dextrose." Acarbose delays glucose absorption, so the client should take an oral form of dextrose rather than a product containing table sugar when treating hypoglycemia. The alpha-glucosidase inhibitors work by delaying the carbohydrate digestion and glucose absorption. It's safe to be on a regimen that includes insulin and an alpha-glucosidase inhibitor. The client should take the drug at the start of a meal, not 30 minutes to an hour before.

A nurse is providing teaching to a client who has type 2 diabetes mellitus about pathophysiology of the disease. Which of the following statements by the client indicates an understanding of the teaching? A. "My cells are resistant to the effects of insulin" B. "My body breaks down sugars too efficiently" C. "My pancreas does not produce insulin" D. "My body produces antibodies against pancreatic beta cells"

A. "My cells are resistant to the effects of insulin" the client who has type 2 DM will have resistance to insulin and decrease in the secretion of insulin by the pancreatic beta cells.

The nurse is preparing a client for discharge on postoperative day 1 after a modified radical mastectomy. Which instruction is most important for the nurse to include in this client's discharge plan? A. "Please report any increased redness, swelling, warmth, or pain to your health care provider." B. "Do not allow anyone to take your blood pressure or draw blood on the side where you had your breast removed." C. "A referral has been made to the American Cancer Society's Reach to Recovery program, and a volunteer will call you next week." D. "Avoid the prone and hunchback positions, and ask your health care provider for any other needed activity restrictions."

A. "Please report any increased redness, swelling, warmth, or pain to your health care provider." Rationale: Instruction on increased signs and symptoms of inflammation could reveal signs of potential infection and is most important. Although information about having blood pressure taken or having blood drawn should be included, it is not the most important instruction for postoperative day 1 discharge. Referrals are important in helping with coping but are not the most important consideration when the client is being sent home on postoperative day 1. Positioning is important but is not the priority here.

A client has diabetes insipidus and receives desmopressin (DDAVP). The nurse completes medication education and evaluates that learning has occurred when the client makes which statement? Select one: A. "This medication increases water reabsorption in my kidneys." B. "This medication is a potent vasodilator; my blood pressure can fall." C. "This medication promotes diuresis in my body; my blood pressure can fall." D. "This medication suppresses hormone secretion from my posterior pituitary gland."

A. "This medication increases water reabsorption in my kidneys."

A client is receiving highly active antiretroviral therapy (HAART). Which statement by the client indicates a need for further teaching by the nurse? A. "With this treatment, I probably cannot spread this virus to others." B. "This treatment does not kill the virus." C. "This medication prevents the virus from replicating in my body." D. "Research has shown the effectiveness of this therapy if I do not forget to take any doses."

A. "With this treatment, I probably cannot spread this virus to others." Rationale: HAART reduces viral load and improves CD4+ T-cell counts, but the client must still protect others from contact with his or her body fluids. HAART inhibits viral replication; it does not kill the virus. Remembering to take all doses of HAART is very important for preventing drug resistance.

Jemma, who weighs 210 lb (95 kg) and has been diagnosed with hyperglycemia tells the nurse that her husband sleeps in another room because her snoring keeps him awake. The nurse notices that she has large hands and a hoarse voice. Which of the following would the nurse suspect as a possible cause of the client's hyperglycemia? A. Acromegaly B. Type 1 diabetes mellitus C. Hypothyroidism D. Deficient growth hormone

A. Acromegaly Acromegaly, which is caused by a pituitary tumor that releases excessive growth hormone, is associated with hyperglycemia, hypertension, diaphoresis, peripheral neuropathy, and joint pain. Enlarged hands and feet are related to lateral bone growth, which is seen in adults with this disorder. The accompanying soft tissue swelling causes hoarseness and often sleep apnea. Type 1 diabetes is usually seen in children, and newly diagnosed persons are usually very ill and thin. Hypothyroidism isn't associated with hyperglycemia, nor is growth hormone deficiency.

Nurse Louie is developing a teaching plan for a male client diagnosed with diabetes insipidus. The nurse should include information about which hormone lacking in clients with diabetes insipidus? A. Antidiuretic hormone (ADH). B. Thyroid-stimulating hormone (TSH). C. Follicle-stimulating hormone (FSH). D. Luteinizing hormone (LH)

A. Antidiuretic hormone (ADH). ADH is the hormone clients with diabetes insipidus lack. The client's TSH, FSH, and LH levels won't be affected.

Which local anesthetic drug might interfere with the antibacterial activity of some sulfonamide drugs? A. Benzocaine (Americaine) B. Tetracaine (Pontocaine) C. Bupivacaine (Marcaine) D. Lidocaine (Xylocaine)

A. Benzocaine (Americaine)

If a male client experienced a cerebrovascular accident (CVA) that damaged the hypothalamus, the nurse would anticipate that the client has problems with: A. Body temperature control. B. Balance and equilibrium. C. Visual acuity. D. Thinking and reasoning.

A. Body temperature control. The body's thermostat is located in the hypothalamus; therefore, injury to that area can cause problems of body temperature control. Balance and equilibrium problems are related to cerebellar damage. Visual acuity problems would occur following occipital or optic nerve injury. Thinking and reasoning problems are the result of injury to the cerebrum.

A nurse caring for a client who is postoperative following a parathyroidectomy to treat hyperpararthyroidism. Which of the following laboratory values should the nurse expect to decrease as a therapeutic effect of the procedure? A. Calcium B. Sodium C. Potassium D. Phosphorus

A. Calcium Parathyroid hormone regulates calcium, phosphorus, and magnesium balance within the client's blood and bone by maintaining a balance between the mineral levels in the blood and bone. Hyperparathyroidism is associated with hypercalcemia; therefore, a decrease in the calcium level indicates an improvement in the client's condition.

Following a unilateral adrenalectomy, nurse Betty would assess for hyperkalemia shown by which of the following? A. Muscle weakness B. Tremors C. Diaphoresis D. Constipation

A. Muscle weakness Muscle weakness, bradycardia, nausea, diarrhea, and paresthesia of the hands, feet, tongue, and face are findings associated with hyperkalemia, which is transient and occurs from transient hypoaldosteronism when the adenoma is removed. Tremors, diaphoresis, and constipation aren't seen in hyperkalemia.

During the early postoperative period. the client who had a cataract extraction complains of nausea and severe eye pain over the operative site. The initial nursing action is to: A. Call the physician B. Administer the ordered main medication and antiemetic C. Reassure the client that this is normal. D. Turn the client on his or her operative side

A. Call the physician Severe pain or pain accompanied by nausea is an indicator of increased intraocular pressure and should be reported to the physician immediately. The other options are inappropriate.

Which interventions does the home health nurse teach to family members to reduce confusion in a client diagnosed with acquired immune deficiency syndrome (AIDS)-related dementia? (Select all that apply.) A. Change the decorations in the home according to the season. B. Put the bed close to the window. C. Write out detailed instructions, and have the client read them over before performing a task. D.Ask the client what time he or she prefers to shower or bathe. E. Mark off the days of the calendar, leaving open the current date.

A. Change the decorations in the home according to the season. B. Put the bed close to the window. D.Ask the client what time he or she prefers to shower or bathe. E. Mark off the days of the calendar, leaving open the current date. Rationale: Changing decorations according to the season and using a calendar to mark off the days will help to keep the client oriented. Keeping the bed close to the window may help keep the client oriented. The client should be included in planning the daily schedule. Directions should be short and uncomplicated.

A nurse is caring for a client who is postoperative following a bilateral adrenalectomy. The nurse should expect to administer glucocorticoids following the procedure to enhance which of the following therapeutic effects? A. Compensate for decrease in cortisol levels B. inhibit glucose metabolism C. Act as a diuretic to maintain urine output. D. Decrease susceptibility to infection

A. Compensate for decrease in cortisol levels The client who has an adrenalectomy requires glucocorticoids before, during and after surgery to prevent an adrenal crisis caused by a sudden drop in cortisol levels. One of the hormones produced by the adrenal glands is cortisol, a glucocorticoid. Loss of glucocorticoid secretion leads to a state of altered metabolism and an inability to deal with stressors which, if untreated is fatal.

The nurse is caring for a client with a diagnosis of detached retina. Which assessment sign would indicate that bleeding has occurred as a result of the retinal detachment? A. Complaints of a burst of black spots or floaters B. A sudden sharp pain in the eye C. Total loss of vision D. A reddened conjunctiva

A. Complaints of a burst of black spots or floaters Complaints of a sudden burst of black spots or floaters indicate that bleeding has occurred as a result of the detachment.

A client with a history of asthma is admitted to the clinic for allergy testing. During skin testing, the client develops shortness of breath and stridor and becomes hypotensive. What is the most appropriate drug for the nurse to give in this situation? A. Epinephrine (Adrenalin) B.Fexofenadine (Allegra) C. Cromolyn sodium (Nasalcrom) D. Zileuton (Zyflo)

A. Epinephrine (Adrenalin) Rationale: The client is experiencing an anaphylactic reaction, and epinephrine is a first-line sympathomimetic used to treat anaphylaxis. Fexofenadine (Allegra) is a nonsedating antihistamine and is not a first-line drug to treat anaphylaxis. Cromolyn sodium (Nasalcrom) is a mast cell-stabilizing drug; it is used to prevent symptoms of allergic rhinitis, but is not useful during an acute episode. Zileuton (Zyflo) is a leukotriene antagonist; it is also used to prevent symptoms of allergic rhinitis, but is likewise not useful during an acute episode.

A client recently diagnosed with human immune deficiency virus (HIV) is being treated for candidiasis. Which medication does the nurse anticipate the health care provider will prescribe for this client? A. Fluconazole (Diflucan) B.Trimethoprim/sulfamethoxazole (Bactrim) C. Rifampin (Rifadin) D. Acyclovir (Zovira

A. Fluconazole (Diflucan) Rationale: Fluconazole (Diflucan) is indicated for opportunistic candidiasis infection related to HIV. Trimethoprim/sulfamethoxazole (Bactrim) is indicated for bacterial infections such as urinary tract infection. Rifampin (Rifadin) is used for treatment of tuberculosis. Acyclovir (Zovirax) is an antiviral agent.

The client with a brain attack (stroke) has residual dysphagia. When a diet order is initiated, the nurse avoids doing which of the following? A. Giving the client thin liquids B. Thickening liquids to the consistency of oatmeal C. Placing food on the unaffected side of the mouth D. Allowing plenty of time for chewing and swallowing

A. Giving the client thin liquids Before the client with dysphagia is started on a diet, the gag and swallow reflexes must have returned. The client is assisted with meals as needed and is given ample time to chew and swallow. Food is placed on the unaffected side of the mouth. Liquids are thickened to avoid aspiration

A nurse is checking laboratory values to determine if a client who has diabetes is adhering to the treatment plan. Which of the following tests should the nurse use to make this determination? A. Glycosylated hemoglobin levels B. Urine sugar and acetone C. Glucose tolerance test D. Fasting serum glucose

A. Glycosylated hemoglobin levels Checking glycosylated hemoglobin levels, or HbA1c is an accurate method to determine if the client is routinely compliant. Glycosylated hemoglobin refers to hemoglobin that is connected to glucose. Since the life span of an RBC is 4 months, this value will not be affected by recent changes in the client's diet or medication.

Nurse Wayne is aware that a positive Chvostek's sign indicate? A. Hypocalcemia B. Hyponatremia C. Hypokalemia D. Hypermagnesemia

A. Hypocalcemia Chvostek's sign is elicited by tapping the client's face lightly over the facial nerve, just below the temple. If the client's facial muscles twitch, it indicates hypocalcemia. Hyponatremia is indicated by weight loss, abdominal cramping, muscle weakness, headache, and postural hypotension. Hypokalemia causes paralytic ileus and muscle weakness. Clients with hypermagnesemia exhibit a loss of deep tendon reflexes, coma, or cardiac arrest.

The physician writes orders for the client with diabetes mellitus. Which order would the nurse validate with the physician? Select one: A. Lantus insulin 20U BID B. Administering regular insulin 30 minutes prior to meals C. 5 units of Humalog/10 units NPH daily D. Metformin (Glucophage) 1000 mg per day in divided doses

A. Lantus insulin 20U BID

Which of the following laboratory results would alert the healthcare provider that a patient who has diabetes is experiencing diabetic nephropathy? Please choose from one of the following options. Microalbuminuria Hemoglobin A1c 6% Decreased BUN Ketonuria

A. Microalbuminuria Rationale: 1 / 3 Renal changes in diabetics are the result of hyperglycemia induced changes at the microvascular level. Hint #22 / 3 Glomerular changes include endothelial dysfunction and membrane permeability. Hint #33 / 3 Nephropathy results in the loss of proteins in the urine.

The nurse is caring for a client following enucleation. The nurse notes the presence of bright red blood drainage on the dressing. Which nursing action is appropriate? A. Notify the physician B. Continue to monitor the drainage C. Document the finding D. Mark the drainage on the dressing and monitor for any increase in bleeding

A. Notify the physician If the nurse notes the presence of bright red drainage on the dressing. it must be reported to the physician because this indicated hemorrhage

Which antibiotic class is one of the oldest, safest, and effective although widespread resistance has developed to the drugs? A. Penicillins B. Tetracyclines C. Macrolides D. Aminoglycosides

A. Penicillins

After undergoing a subtotal thyroidectomy, a female client develops hypothyroidism. Dr. Smith prescribes levothyroxine (Levothroid), 25 mcg P.O. daily. For which condition is levothyroxine the preferred agent? A. Primary hypothyroidism B. Graves' disease C. Thyrotoxicosis D. Euthyroidism

A. Primary hypothyroidism Levothyroxine is the preferred agent to treat primary hypothyroidism and cretinism, although it also may be used to treat secondary hypothyroidism. It is contraindicated in Graves' disease and thyrotoxicosis because these conditions are forms of hyperthyroidism. Euthyroidism, a term used to describe normal thyroid function, wouldn't require any thyroid preparation.

A client recently admitted to the hospital is to receive an antibiotic intravenously for the first time for a urinary tract infection. Before checking the five rights prior to administration, what is the nurse's first action? A. Review the clinical records and ask the client about any known allergies. B. Check with the pharmacy for any known allergies for this client. C. Check the client's identification band for any allergies. D.Ask the nurse who previously cared for the client about any known allergies.

A. Review the clinical records and ask the client about any known allergies. Rationale: The clinical record should have all known hypersensitivities listed for the client. The client should also be asked directly about any known allergies. The pharmacy is not responsible for obtaining information on all of the client's known allergies. Checking the client's identification band for allergies is part of the "five rights" process at the bedside before the medication is given. Asking the previous nurse is not an appropriate safety measure before medication administration.

Nurse Troy is aware that the most appropriate for a client with Addison's disease? A. Risk for infection B. Excessive fluid volume C. Urinary retention D. Hypothermia

A. Risk for infection Addison's disease decreases the production of all adrenal hormones, compromising the body's normal stress response and increasing the risk of infection. Other appropriate nursing diagnoses for a client with Addison's disease include Deficient fluid volume and Hyperthermia. Urinary retention isn't appropriate because Addison's disease causes polyuria.

A nurse is providing teaching to a client who has type 1 diabetes mellitus about hypoglycemia. Which of the following manifestations should the nurse include in the teaching? A. Shakiness B. Urinary frequency C. Dry mucous membranes D. Excess thirst

A. Shakiness

A patient diagnosed with type 2 diabetes mellitus is admitted to the medical unit with pneumonia. The patient's oral antidiabetic medication has been discontinued and the patient is now receiving insulin for glucose control. Which of the following statements best explains the rationale for this change in medication? Please choose from one of the following options. Stress-related states such as infections increase risk of hyperglycemia Infection has compromised beta cell function so the patient will need insulin from now on Insulin administration will help prevent hypoglycemia during the illness Acute illnesses like pneumonia will cause increased insulin resistance

A. Stress-related states such as infections increase risk of hyperglycemia Rationale: 1 / 3 Infections such as pneumonia stimulate a stress response in the body. Hint #22 / 3 Stress hormones include glucocorticoids and epinephrine. Hint #33 / 3 Glucocorticoids and epinephrine increase blood glucose levels. Type 2 diabetics may temporarily require insulin during acute illnesses and hospitalizations, but they often return to their normal medication regimen after they recover.

The client receives treatment with radioactive iodine (Iodine-131) therapy. What will the best evaluation by the nurse reveal? Select one: A. The client will most likely require thyroid replacement therapy. B. The client does not have to distance herself from others. C. The client will only temporarily accomplish the euthyroid state. D. The client could safely become pregnant while receiving this treatment.

A. The client will most likely require thyroid replacement therapy.

Which statement accurately explains otitis media? A. The inflammatory response is triggered by the invasion of foreign proteins. B. Phagocytosis by macrophages and neutrophils destroys and eliminates foreign invaders. C. It is caused by a left shift or increase in immature neutrophils. D. Many immune system cells released into the blood have specific effects.

A. The inflammatory response is triggered by the invasion of foreign proteins. Rationale: Inflammation is the process that occurs in response to invasion by organisms. In otitis media, it is bacteria. Macrophages and neutrophils are involved in inflammation; otitis media is an inflammation caused by infection. Immature neutrophil forms should not be in the blood; the change in form is caused by infection, such as sepsis. Immune system cells take action when encountering a non-self or foreign protein to neutralize, destroy, or eliminate a foreign invader, but this does not cause inflammation.

A female client has a serum calcium level of 7.2 mg/dl. During the physical examination, nurse Noah expects to assess: A. Trousseau's sign. B. Homans' sign. C. Hegar's sign D. Goodell's sign

A. Trousseau's sign. This client's serum calcium level indicates hypocalcemia, an electrolyte imbalance that causes Trousseau's sign (carpopedal spasm induced by inflating the blood pressure cuff above systolic pressure). Homans' sign (pain on dorsiflexion of the foot) indicates deep vein thrombosis. Hegar's sign (softening of the uterine isthmus) and Goodell's sign (cervical softening) are probable signs of pregnancy.

34.2 The student nurse asks the nursing instructor for help with her microbiology class. The student is studying bacteria. What does the best plan of the nursing instructor include? Select all that apply. 1. Spherical-shaped bacteria are called cocci. 2. Gram-staining is one way to identify bacteria. 3. E-coli are gram-negative bacteria. 4. Bacteria are either aerobic or anaerobic. 5. Bacteria are multicellular organisms.

Answer: 1, 2, 3 Rationale: Spherical-shaped bacteria are called cocci. Gram-staining is one way to identify bacteria. E-coli are gram-negative bacteria. Some organisms have the ability to change their metabolism and survive in either aerobic or anaerobic conditions. Bacteria are single cell organisms.

When caring for a male client with diabetes insipidus, nurse Juliet expects to administer: A. Vasopressin (Pitressin Synthetic). B. Furosemide (Lasix). C. Regular insulin. D. 10% dextrose

A. Vasopressin (Pitressin Synthetic). Because diabetes insipidus results from decreased antidiuretic hormone (vasopressin) production, the nurse should expect to administer synthetic vasopressin for hormone replacement therapy. Furosemide, a diuretic, is contraindicated because a client with diabetes insipidus experiences polyuria. Insulin and dextrose are used to treat diabetes mellitus and its complications, not diabetes insipidus.

A male client with a history of hypertension is diagnosed with primary hyperaldosteronism. This diagnosis indicates that the client's hypertension is caused by excessive hormone secretion from which of the following glands? A. Adrenal cortex B. Pancreas C. Adrenal medulla D. Parathyroid

A. adrenal cortex Excessive secretion of aldosterone in the adrenal cortex is responsible for the client's hypertension. This hormone acts on the renal tubule, where it promotes reabsorption of sodium and excretion of potassium and hydrogen ions. The pancreas mainly secretes hormones involved in fuel metabolism. The adrenal medulla secretes the catecholamines — epinephrine and norepinephrine. The parathyroids secrete parathyroid hormone.

The nurse is caring for a client receiving gentamicin IV. The nurse would observe for adverse effects of Select one: A. ototoxicity. B. increased urinary output. C. diarrhea. D. bleeding.

A. ototoxicity.

49.10 The client has glaucoma. Which assessment finding indicates to the nurse that a client's medical regimen could have contributed to onset of glaucoma? 1. Occasional use of antihistamines for allergies 2. Regular use of an anticholinergic drug for urinary incontinence 3. Taking a beta-blocker to treat hypertension 4. Taking glucocorticoids to treat arthritis

Answer: 2 Regular use of an anticholinergic drug for urinary incontinence Rationale: Anticholinergic drugs can contribute to glaucoma. Occasional use of antihistamines for allergies should not be a factor. Hypertension can be a factor, but beta-blockers may be used to treat glaucoma. Glucocorticoids can contribute to the development of cataracts, but not glaucoma.

In a 29-year-old female client who is being successfully treated for Cushing's syndrome, nurse Lyzette would expect a decline in: A. Serum glucose level. B. Hair loss. C. Bone mineralization. D. Menstrual flow

A. serum glucose level Hyperglycemia, which develops from glucocorticoid excess, is a manifestation of Cushing's syndrome. With successful treatment of the disorder, serum glucose levels decline. Hirsutism is common in Cushing's syndrome; therefore, with successful treatment, abnormal hair growth also declines. Osteoporosis occurs in Cushing's syndrome; therefore, with successful treatment, bone mineralization increases. Amenorrhea develops in Cushing's syndrome. With successful treatment, the client experiences a return of menstrual flow, not a decline in it.

The nurse is assessing a client with suspected serum sickness. Which symptoms are consistent with serum sickness? (Select all that apply.) A.Arthralgia Correct B.Blurred vision C. Lymphadenopathy D. Malaise E. Ptosis

A.Arthralgia C. Lymphadenopathy D. Malaise Rationale: Serum sickness is a group of symptoms that occurs after receiving serum or certain drugs; symptoms include arthralgia (achy joints), lymphadenopathy (enlarged lymph nodes), fever, rash, malaise, and possibly polyarthritis and nephritis. Blurred vision and ptosis are not symptoms of serum sickness.

Which factors are possible transmission routes for human immune deficiency virus (HIV)? (Select all that apply.) A.Breast-feeding B.Anal intercourse C. Mosquito bites D.Toileting facilities E. Oral sex

A.Breast-feeding B.Anal intercourse E. Oral sex Rationale: HIV can be transmitted via breast milk from an infected mother to the child. Anal intercourse not only allows seminal fluid to make contact with the mucous membranes of the rectum, but it also tears the mucous membranes, making infection more likely. Oral sexual contact exposes the mucous membranes to infected semen or vaginal secretions. HIV is not spread by mosquito bites or by other insects. HIV is not transmitted by casual contact, and sharing toilet facilities does not allow transmission of HIV.

A patient has been diagnosed with tuberculosis and is prescribed Rifater (pyrazinamide with isoniazid and rifampin). While the patient is on this medication, what teaching is essential? (Select all that apply.) 1. "It is critical to continue therapy for at least 6 to 12 months." 2. "Two or more drugs are used to prevent tuberculosis bacterial resistance." 3. "These drugs may also be used to prevent tuberculosis." 4. "No special precautions are required." 5. "After 1 month of treatment, the medication will be discontinued

Ans: 1,2,3 Rationale: In order to effectively treat the TB bacterium, it is critical that the medicine be taken for 6 to 12 months and possibly as long as 24 months. Antitubercular drugs such as pyrazinamide, isoniazid (INH), and rifampin are also used for prevention and treatment of patients who convert from a negative TB test to a positive, although single drug use is most often prescribed in that situation. Multiple drug therapy is necessary because the Mycobacteria grow slowly, and resistance is common. Using multiple drugs in different combinations during the long treatment period lowers the potential for resistance and increases the chances for successful therapy. Options 4 and 5 are incorrect. Precautions to avoid adverse effects are required, and the drugs will be required much longer than 1 month.

A patient is started on efavirenz (Sustiva) for HIV. What should the nurse teach the patient about this drug? 1. Efavirenz (Sustiva) will cure the disease over time. 2. Efavirenz (Sustiva) will not cure the disease but may signifcantly extend the life expectancy. 3. Efavirenz (Sustiva) will be used prior to vaccines. 4. Efavirenz (Sustiva) will prevent the transmission of the disease.

Ans: 2 Rationale: Drug therapy with efavirenz (Sus-tiva) and other HAART drugs has not produced a cure but has resulted in a significant number of therapeutic successes with increased life span. Options 1, 3, and 4 are incorrect. There is currently no vaccine for HIV although research is ongoing. The drug does not cure the disease. Evidence has shown that HIV treatment significantly decreases viral loads and thus decreases the risk of transmission, but this has not yet been proven in all cases of infection.

A patient will be discharged after surgery with a prescription for penicillin. When planning at-home instructions, what will the nurse include? 1. Penicillin's can be taken while breast-feeding. 2. The entire prescription must be finished. 3. All penicillin's can be taken without regard to eating. 4. Some possible side effects include abdominal pain and constipation

Ans: 2 Rationale: Many people will discontinue medication after improvement is noted. All antibiotic regimens must be completed to prevent recurrence of infection unless allergy or significant adverse effects occur that warrant discontinuing or changing the drug used. Options 1, 3, and 4 are incorrect. Some penicillins (e.g., amoxicillin) should be taken with meals, whereas all others should be taken 1 hour before or 2 hours after meals. Penicillins should be used with caution during breast-feeding. Penicillins, along with other antibiotics, tend to cause diarrhea and not constipation.

A 32-year-old female has been started on amoxicillin (Amoxil, Trimox) for a severe UTI. Before sending her home with this prescription, the nurse will provide which instruction? 1. Teach her to wear sunscreen. 2. Ask her about oral contraceptive use and recommend an alternative method for the duration of the ampicillin course. 3. Assess for hearing loss. 4. Recommend taking the pill with some antacid to prevent gastrointestinal upset.

Ans: 2 Rationale: Penicillin antibiotics such as amoxicillin (Amoxil, Trimox) may significantly decrease the effectiveness of oral contraceptives and another method of birth control should be suggested during the time the drug is taken. Options 1, 3, and 4 are incorrect. Sun burning and hearing loss are not adverse effects commonly associated with penicillin.

The nurse is teaching a community health class to a group of young adults who have recently immigrated to the United States about preventing hepatitis B. What is the most effective method of preventing a hepatitis B infection? 1. Peginterferon alfa-2a (Pegasys) 2. Hepatitis B vaccine (Engerix-B) 3. Adefovir dipivoxil (Hepsera) 4. Entecavir (Baraclude)

Ans: 2 Rationale: The best method of preventing hepatitis B (HBV) infections is to complete a series of the HBV vaccination. Three doses of the vaccine provide up to 90% of patients with protection following exposure to the virus. Options 1, 3, and 4 are incorrect. Treatment of acute HBV infection is symptomatic because no specific therapy is available. Interferons such as peginterferon alfa-2a (Pegasys) or antiviral drugs such as adefovir dipivoxil (Hepsera) or entecavir (Baraclude) only treat the disease by stopping viral replication to reduce the length of the disease process or by boosting the body's defenses.

A patient with HIV has been taking lopinavir with ritonavir (Kaletra) for the past 8 years and has noticed a redistribution of body fat in the arms, legs, and abdomen (lipodystrophy). The nurse will evaluate this patient for what other additional adverse effects associated with this drug? (Select all that apply.) 1. Renal failure 2. Hyperglycemia 3. Pancreatitis 4. Bone marrow suppression 5. Hepatic failure

Ans: 2,3,5 Rationale: Hyperglycemia, pancreatitis, and hepatic failure are adverse effects associated with lopinavir with ritonavir (Kaletra). Options 1 and 4 are incorrect. Renal failure and bone marrow suppression are not adverse effects associated with this drug.

Which of the following findings would suggest that myelosuppression is occurring in a patient who is taking zidovudine (Retrovir)? 1. Increase in serum blood urea nitrogen (BUN) levels 2. Increase in white blood cell (WBC) count 3. Decrease in platelet count 4. Decrease in blood pressure

Ans: 3 Rationale: Myelosuppression is the declining ability of the bone marrow to produce blood cells. A de- crease in platelet count may indicate myelosuppression is occurring. Options 1, 2, and 4 are incorrect. An increase in BUN or a decrease in blood pressure does not indicate myelosuppression. A decrease, rather than increase, in WBC count would be expected if myelosuppression is occurring.

What important information should be included in the patient's education regarding taking ciprofloxacin (Cipro)? 1. The drug can cause discoloration of the teeth. 2. Fluid intake should be decreased to prevent urine retention. 3. Any heel or lower leg pain should be reported immediately. 4. The drug should be taken with an antacid to reduce gastric effects.

Ans: 3 Fluoroquinolones such as ciprofloxacin (Cipro) have been associated with an increased risk of tendinitis and tendon rupture. Any heel or lower leg pain should be reported immediately for evaluation. Options 1, 2, and 4 are incorrect. Ciprofloxacin will not cause discoloration of the teeth, and fluids should be encouraged during use of the drug. Taking antacids concurrently with ciprofloxacin may significantly impair absorption of the drug.

Superinfections are an adverse effect common to all antibiotic therapy. Which of the following best describes a superinfection? 1. An initial infection so overwhelming that it requires multiple antimicrobial drugs to treat successfully 2. Bacterial resistance that creates infections that are difficult to treat and are often resistant to multiple drugs 3. Infections requiring high-dose antimicrobial therapy with increased chance of organ toxicity 4. The overgrowth of normal body flora or of opportunistic organisms such as viruses and yeast no longer held in check by normal, beneficial flora

Ans: 4 Rationale: When normal host flora are de- creased or killed by antibacterial therapy, opportunistic organisms such as viral and fungal infections may occur. Options 1, 2, and 3 are incorrect. Bacterial resistance and organ toxicity may be adverse drug effects of anti- bacterial therapy but do not describe superinfections. The use of multiple antibiotics for severe infections is a therapeutic use of the drugs.

The nurse is administering an anti-infective that is considered to be a bacteriocidal. Before beginning therapy, the nurse should assess the client for: a. Any neurologic abnormalities. b. Cardiac history. c. Hypersensitivity. d. Respiratory conditions.

Answer: C. Hypersensitivity. Objective: Describe the nurse's role in the pharmacologic management of bacterial infections. Rationale: The role of the nurse in drug therapy with penicillin involves careful history taking to assess for previous reactions to penicillin.

A patient has received a prescription for zanamivir (Relenza) for flulike symptoms. The patient states, "I think I'll hold off on starting this. I don't feel that bad yet." What is the nurse's best response? 1. "The drug has a stable shelf life so you can save it for later infections." 2. "It can be saved for later but you will also require an antibiotic to treat your symptoms if you wait." 3. "It can be started within two weeks after the onset of symptoms." 4. "To be effective, it must be started within 48 hours after the onset of symptoms."

Ans: 4 Rationale: Zanamivir (Relenza) must be started within 48 hours after the onset of symptoms to be effective. Options 1, 2, and 3, are incorrect. Immunity begins approximately 2 weeks after influenza immunization. Waiting longer than 48 hours before taking the drug will not shorten the infection period, and the drug should not be saved for later.

A patient has been prescribed tetracycline. When providing information regarding this drug, the nurse should include what information about tetracycline? 1. It is classifed as a narrow-spectrum antibiotic with minimal adverse effects. 2. It is used to treat a wide variety of disease processes. 3. It has been identifed to be safe during pregnancy. 4. It is contraindicated in children younger than 8 years.

Ans: 4 Tetracycline has the ability to cause permanent mottling and discoloration of teeth and there- fore is not advised for children younger than 8 years of age. Options 1, 2, and 3 are incorrect. Tetracyclines have one of the broadest spectrums of the antibiotics, and all antibiotics have significant adverse effects. Tetracycline is contraindicated in pregnancy.

34.11 The client receives an injection of penicillin G benzathine (Bicillin LA) in the outpatient clinic. What is a priority nursing action by the nurse prior to administering this injection? 1. Inform the client she will need to wait for 30 minutes before leaving the clinic. 2. Ask the client if she has ever had an allergic reaction to penicillin. 3. Have the client lie down and assess vital signs before she leaves. 4. Tell the client she will need to have someone drive her home.

Answer: 1 Rationale: It is important for the client to wait for 30 minutes before leaving the clinic because the client may not know whether or not she is allergic to penicillin. Asking about a penicillin allergy is important, but the client could still have an allergic reaction if she has not received previous doses of penicillin. There is no reason for the client to lie down and the nurse does not need to assess vital signs unless an allergy is suspected. There is no reason for the client to have someone drive her home.

35.9 The client receives metronidizole (Flagyl) for treatment of a vaginal yeast infection. What does the best medication education by the nurse include? 1. Do not drink alcohol with this medication. 2. Drink at least 2,000 mL of fluid with this medication. 3. Do not take this medication with milk or milk products. 4. Eat at least two cups of yogurt daily while on this medication.

Answer: 1 Rationale: The interaction of metronidizole (Flagyl) and alcohol causes an Antabuse-like reaction, i.e., severe nausea, vomiting, and abdominal pain. Pushing fluids is not necessary. There is no interaction with this medication and milk or milk products. There is no need to consume yogurt while on this medication.

36.14 The nurse accidentally sticks herself with a needle after starting an intravenous (IV) line on a client with acquired immune deficiency syndrome (AIDS). The nurse tells the supervisor about the accident. What is the best initial response by the supervisor to decrease anxiety in the nurse? 1. "Fortunately, the chances of you contracting human immunodeficiency virus (HIV) after the stick are very small." 2. "If you are started on medications soon, it will decrease the severity of the disease." 3. "Workers compensation will cover the cost of your illness and medications." 4. "Did you use the hospital protocols for starting intravenous (IV) lines on a client with acquired immune deficiency syndrome (AIDS)?"

Answer: 1 "Fortunately, the chances of you contracting human immunodeficiency virus (HIV) after the stick are very small." Rationale: There have been only about 56 cases of client-to-healthcare worker transmission; the risk is very small. The risk of transmission of human immunodeficiency virus (HIV) from client-to-healthcare worker is very small, and telling the nurse to start on medication, will not reduce anxiety. If the nurse did contract human immunodeficiency virus (HIV), workers compensation would cover the cost, but the risk of human immunodeficiency virus (HIV) transmission is small and this response will not reduce anxiety. Asking the nurse if she used protocols is very non-therapeutic at this point, and will most likely increase anxiety.

49.6 The client receives two different eye drop medications. The nurse has completed medication education and evaluates that learning has occurred when the client makes which statement? 1. "I will give two drops of one medication, wait five minutes, and then give two drops of the other medication." 2. "I will give two drops of one medication, wait one minute, and then give two drops of the other medication." 3. "I will give two drops of one medication and then two drops of the other medication." 4. "I will wait for five minutes between each set of drops for each of the medications."

Answer: 1 "I will give two drops of one medication, wait five minutes, and then give two drops of the other medication." Rationale: The correct procedure is for the client to administer one medication, wait five minutes, and then administer the other medication. The correct procedure is for the client to administer one medication, wait five minutes, not one minute, and then administer the other medication. The correct procedure is for the client to administer one medication; the client must then wait five minutes before administering the second medication - the second medication cannot be given immediately after the first medication. The correct procedure is for the client to administer all drops of one medication, wait five minutes, and then administer all drops of the second medication.

36.12 The nurse does health teaching with a client who has acquired immune deficiency syndrome (AIDS) and who has been started on antiviral medications. The nurse recognizes teaching has been effective when the client makes which statement? 1. "I will still need to take precautions to avoid spreading the virus to others." 2. "I will need to be on a high-calorie diet to enhance the effect of the medication." 3. "I will need to limit my travel to avoid people with other infections." 4. "I will not be able to continue working at my high-stress job anymore."

Answer: 1 "I will still need to take precautions to avoid spreading the virus to others." Rationale: Precautions are still necessary to avoid spreading the virus to others. A well-balanced diet is the best diet for a client with acquired immune deficiency syndrome (AIDS). There is no need to limit travel at this point. A high-stress job is not ideal, but at this point the client doesn't need to stop working.

35.13 The client has a fungal infection of the toenails and receives oral terbinafine (Lamisil). The client asks the nurse how a pill will heal his nail infection. What is the best response by the nurse? 1. "The medication accumulates in your nail beds and is there for many months." 2. "The medication should be combined with a topical agent to increase effectiveness." 3. "The medication works by destroying circulating fungi in your blood." 4. "The medication works by destroying toxins excreted by the fungi in your nails."

Answer: 1 "The medication accumulates in your nail beds and is there for many months." Rationale: Terbinafine (Lamisil) is an oral agent that has the advantage of accumulating in nail beds, allowing it to remain active many months after therapy is discontinued. Terbinafine (Lamisil) does not need to be combined with a topical agent. Terbinafine (Lamisil) does not destroy circulating fungi in the blood. Terbinafine (Lamisil) does not destroy toxins excreted by fungi.

35.2 The client has had malaria for many years. He asks the nurse why medications might not cure his illness when there are several drugs available. What is the best response by the nurse? 1. "When cysts occur late in the disease, the parasite is too resistant for medications to be effective." 2. "When erythrocytes rupture, the parasites are too numerous for medications to be effective." 3. "Once the parasite starts multiplying in your liver, medications are usually ineffective." 4. "Late in the illness, the immune system is too overwhelmed for medications to be effective."

Answer: 1 "When cysts occur late in the disease, the parasite is too resistant for medications to be effective." Rationale: When cysts occur inside the host, the parasite is often resistant to pharmacotherapy. Erythrocytes rupture early in the course of the illness, and medications are more effective at this stage The parasites begin multiplying in the liver in the earliest stage of the illness when medications are most effective. Late in the illness, medications are often ineffective because the parasites are in cysts, not because the immune system is overwhelmed.

36.3 The client receives zidovudine (Retrovir) for treatment of human immunodeficiency virus (HIV) infection. Which assessment data indicates an adverse reaction to zidovudine (Retrovir)? 1. Decreased white blood count (WBC) 2. Enlarged lymph nodes 3. Fever 4. Edema

Answer: 1 Decreased white blood count (WBC) Rationale: Bone marrow suppression is a common adverse effect of zidovudine (Retrovir), and decreased white blood cells (WBCs) are indicative of bone marrow suppression. Enlarged lymph nodes are not an adverse effect of zidovudine (Retrovir). Fever is the result of infection, not the use of zidovudine (Retrovir). Edema is not an adverse effect of zidovudine (Retrovir).

34.7 The client comes to the emergency department with a fever of 104° F. What will the best plan of the nurse include? 1. Plan to obtain blood cultures 2. Plan to obtain a complete blood count (CBC) test 3. Plan to obtain a sterile urine specimen 4. Plan to obtain liver and renal function tests

Answer: 1 Plan to obtain blood cultures Rationale: A high fever is usually indicative of a systemic infection. Blood cultures are the best way of identifying the causative organism. Blood cultures, not a complete blood count (CBC), are the best way of identifying the causative organism. Blood cultures, not sterile urine samples, are the best way of identifying the causative organism. Blood cultures, not liver and renal function tests, are the best way of identifying the causative organism.

35.3 The nurse plans to provide health promotion to a group of business travelers who are going to travel to Africa. What will the best teaching plan of the nurse include? 1. Plan to take antimalarial drugs prophylactically. 2. Plan to take antibiotics prophylactically. 3. Plan to take protease inhibitors prophylactically. 4. Plan to receive immunizations for malaria.

Answer: 1 Plan to take antimalarial drugs prophylactically. Rationale: Taking antimalarial drugs prophylactically is recommended when traveling to countries where malaria is found. Antibiotics are not necessary for travel into many countries. Protease inhibitors for viral infections are not recommended. To date, there are no effective immunizations against malaria.

49.5 The client tells the nurse that he experiences frequent eye irritation even after using over-the-counter (OTC) medications. What is the best recommendation by the nurse? 1. See your eye doctor for further evaluation. 2. Use normal saline rinses instead of over-the-counter (OTC) preparations. 3. Increase your fluid intake; you are probably dehydrated. 4. This sounds like an allergic response; try an antihistamine.

Answer: 1 See your eye doctor for further evaluation. Rationale: The client should follow-up with his physician if over-the-counter (OTC) remedies are ineffective for eye irritation. The client should follow-up with his physician; not use normal saline rinses. The client should follow-up with his physician; not increase fluid intake. The client should follow-up with his physician; not try an antihistamine.

36.9 The nurse teaches clients with acquired immune deficiency syndrome (AIDS) about the importance of taking their medications as prescribed. What does the nurse recognize as the primary factor for medication noncompliance in the acquired immune deficiency syndrome (AIDS) population? 1. The necessity of having to take multiple medications throughout the day 2. The difficulty with availability of medications to treat acquired immune deficiency syndrome (AIDS). 3. The unpleasant side effects that are associated with the medications 4. A lack of understanding for the reason to take the medications

Answer: 1 The necessity of having to take multiple medications throughout the day Rationale: Multiple medications are required throughout the day. The higher the number of medications taken daily equates with a higher noncompliance rate. Medications are generally available for clients with acquired immune deficiency syndrome (AIDS) today. Most clients with acquired immune deficiency syndrome (AIDS) recognize the importance of taking medications in spite of side effects. A lack of understanding could be a factor, but it is the schedule of taking multiple drugs that leads to noncompliance.

35.12 The client is noncompliant with taking medications to prevent malaria prior to an overseas business trip. What will the best assessment of the nurse reveal? 1. The side effects of vomiting and diarrhea were too uncomfortable. 2. The medication dosing was too frequent for the client to tolerate. 3. The cost of the medications was too high for the client's budget. 4. The taste of the various tablets was really too offensive to the client.

Answer: 1 The side effects of vomiting and diarrhea were too uncomfortable. Rationale: Side effects, such as vomiting and diarrhea, can be severe after taking the medication. Usually the dosing is once weekly; it is not too frequent for a client to tolerate. The cost of the medication is not a factor, generic forms are available. The medication is not coated, but the taste is not a significant factor.

36.1 The nurse educates clients with acquired immune deficiency syndrome (AIDS) about the nature of viruses. The nurse evaluates that learning has occurred when the clients make which response(s). Select all that apply. 1. "Viruses are nonliving particles." 2. "Viruses are intracellular parasites." 3. "A virion is a mature virus." 4. "The structure of viruses is complex." 5. "Viruses can infect plants as well as animals."

Answer: 1, 2, 3, 5 Rationale: Viruses are nonliving agents that infect bacteria, plants, and animals. Viruses must use intracellular machinery to replicate, so are called intracellular parasites. A mature infective particle is called a virion. Viruses can infect plants as well as animals. The structure of viruses is quite primitive compared to the simplest cell.

35.1 The nursing instructor teaches the student nurses about fungal infections. The nursing instructor evaluates learning has occurred when the student nurses make which statement(s)? Select all that apply. 1. "Systemic infections require oral medications that have serious adverse effects." 2. "Systemic infections are much more common than superficial infections." 3. "Newer medications can be used for superficial as well as systemic infections." 4. "Superficial infections are considered more benign than systemic infections. 5. "Superficial infections are more difficult to treat than systemic infections."

Answer: 1, 3, 4 Rationale: Systemic infections often require aggressive oral or parenteral medications that produce more adverse effects than the topical agents. Some of the newer antifungal agents may be used for either superficial or systemic infections. Superficial infections are relatively benign; systemic infections can be life threatening. Systemic infections are less, not more common than superficial infections. Superficial infections are less difficult, not more difficult, to treat than systemic infections.

34.1 The nurse plans to teach the client with acquired immune deficiency syndrome (AIDS) about bacterial infec¬tions. Which statement describes the best plan of the nurse? Select all that apply. 1. "Pathogenicity means the bacteria can cause an infection." 2. "Pathogens are divided into two classes, bacteria and viruses." 3. "If just a few bacteria make you sick, this is virulence." 4. "Actually, most bacteria will not harm us." 5. "Most bacteria have developed antibiotic resistance."

Answer: 1, 3, 4 Rationale: The ability of an organism to cause infection is called pathogenicity. A highly virulent microbe is one that can produce disease when present in minute numbers. Only a few dozen pathogens commonly cause disease in humans; most are harmless. Human pathogens include viruses, bacteria, fungi, unicellular organisms, and multicellular animals. Antibiotic resistance is a problem; however only a few, not most, bacteria have not developed it.

36.2 The client asks the nurse why there aren't better drugs for human immunodeficiency virus (HIV) infection when so much money is spent on research. What is the best response by the nurse? 1. "Developing new drugs is difficult because people think acquired immune deficiency syndrome (AIDS) is a "gay" disease." 2. "Developing new drugs is difficult because the virus mutates so readily." 3. "Developing new drugs is difficult because we still do not understand the virus." 4. "Developing new drugs is difficult because we still do not have enough money."

Answer: 2 "Developing new drugs is difficult because the virus mutates so readily." Rationale: Antiviral pharmacotherapy can be extremely challenging because of the rapid mutation rate of viruses, which can quickly render drugs ineffective. Most people recognize human immunodeficiency virus (HIV) infection as a heterosexual, not homosexual, disease. After more then 20 years of research, the virus is mostly understood. More money is spent on human immunodeficiency virus (HIV) infection research than just about any other illness.

35.6 The client has scalp ringworm and is being treated with econazole (Spectazole). What are the best discharge instructions by the nurse? 1. "Wash your head at least three times a day for 2 weeks." 2. "Do not share any towels with family members." 3. "Avoid meat and high protein foods with this medication." 4. "Comb the medication into your hair and cover with a towel."

Answer: 2 "Do not share any towels with family members." Rationale: Ringworm can be spread by contact with articles such as towels that are used by the affected person. Washing the head three times a day is excessive and unnecessary while taking econazole (Spectazole). There is no reason to restrict protein. The medication is left on the head for 15 minutes, and then washed out.

35.5 The nurse does medication education for the client with histoplasmosis who receives ketoconazole (Nizoral). The nurse evaluates learning has occurred when the client makes which statement? 1. "I should take this medication with milk or antacids to decrease GI upset." 2. "I could develop nausea, vomiting, and abdominal pain with this medication." 3. "I cannot take this medication longer than 10 days." 4. "I could develop resistance to this medication if I take if too often."

Answer: 2 "I could develop nausea, vomiting, and abdominal pain with this medication." Rationale: Nausea, vomiting and abdominal pain are common side effects of ketoconazole (Nizoral). Taking ketoconazole (Nizoral) with milk or antacids will decrease its absorption. Ketoconazole (Nizoral) is often used for longer than 10 days. Organisms, not people, develop resistance; this is not a likely occurrence anyway.

49.13 The client receives timolol (Timoptic) eye drops for glaucoma. The nurse has completed medication education and evaluates learning has occurred when the client makes which statement? 1. "I will restrict my caffeine to one cup per day." 2. "I will refrain from using over-the-counter (OTC) cold remedies." 3. "I will discontinue the drops if my eye looks red." 4. "I will store my drops in the refrigerator to keep them fresh."

Answer: 2 "I will refrain from using over-the-counter (OTC) cold remedies." Rationale: Many over-the-counter (OTC) cold remedies contain sympathetic nervous system (SNS) drugs that can interact with timolol (Timoptic). There is no need to reduce caffeine when a client uses timolol (Timoptic) eye drops. Timolol (Timoptic) must be continued to prevent loss of vision, the client should not discontinue them. Refrigeration is not necessary for timolol (Timoptic) eye drops.

34.12 The client comes to the emergency department complaining of a sore throat. He has white patches on his tonsils, and he has swollen cervical lymph nodes. What will the best plan of the nurse include? 1. Plan to obtain blood cultures 2. Plan to administer a broad-spectrum antibiotic 3. Plan to obtain a throat culture 4. Plan to administer a narrow-spectrum antibiotic

Answer: 3 Rationale: A throat culture is necessary to identify the causative organism and initiate the best antibiotic treatment. Blood cultures are not necessary at this point because the infection is in the throat; it is not systemic. A broad-spectrum antibiotic is commonly ordered, but a throat culture should be obtained first. Initial therapy with a narrow-spectrum antibiotic is too specific without knowing the causative organism.

34.13 The client is prescribed amoxicillin (Amoxil) for 10 days to treat strep throat. After 5 days, the client tells the nurse he plans to stop the medication because he feels better. What is the best response by the nurse? 1. "You should get another throat culture to see if the infection is gone." 2. "If you stop the medicine early, this could result in damage to your heart." 3. "If you stop the medicine early, this could result in resistance to the antibiotic." 4. "You should get another throat culture if your symptoms return."

Answer: 2 "If you stop the medicine early, this could result in damage to your heart." Rationale: If all the medication is not taken, remaining organisms could become resistant, and strep infections can damage heart valves. Another throat culture is inappropriate; the client must finish the medication. Stopping the medicine early can result in resistance to the antibiotic, but the client may not care about this unless he can see how it directly affects him. Another throat culture is inappropriate; the client must finish the medication.

34.10 The client receives multiple drugs for treatment of tuberculosis. The nurse teaches the client the rationale for multiple drug treatment, and evaluates learning as effective when the client makes which statement? 1. "Treatment for tuberculosis is complex, and multiple drugs must be continued for as long as I am contagious." 2. "Multiple drugs are necessary because the bacteria are likely to develop resistance to just one drug." 3. "Multiple drug treatment is necessary for me to be able to develop immunity to tuberculosis." 4. "Current research indicates that the most effective way to treat tuberculosis is with multiple drugs."

Answer: 2 "Multiple drugs are necessary because the bacteria are likely to develop resistance to just one drug." Rationale: Tuberculosis bacilli are likely to develop resistance to one drug, so multiple drugs must be used. Treatment must be continued long after the client is no longer contagious. Clients cannot develop immunity to bacterial infections. Current research does support multiple drug treatment, but this does not explain the rationale for this to the client.

34.6 The nurse works in infection control and teaches a class to staff nurses about the ways that resistance to antibiotics can occur. The nurse evaluates that learning has occurred when the nurses make which statement? 1. "Resistance to antibiotics can occur by the prophylactic use of them for pre-operative clients." 2. "Resistance to antibiotics can occur by the common use of them for nosocomial infections." 3. "Resistance to antibiotics can occur when physicians prescribe too many of them for elderly clients." 4. "Resistance to antibiotics can occur when physicians prescribe too many of them for children with ear infections."

Answer: 2 "Resistance to antibiotics can occur by the common use of them for nosocomial infections." Rationale: The organisms that cause nosocomial infections have most likely been treated with antibiotics, and are the most likely organisms to develop resistance to antibiotics. The prophylactic use of antibiotics does not promote antibiotic resistance. The use of antibiotics by physicians with elderly clients does not promote antibiotic resistance. The use of antibiotics by physicians for children with ear infections does not promote antibiotic resistance.

49.15 The client receives eye drops as treatment for glaucoma. The client calls the clinic one day and tells the nurse that his eye color is changing. What is the best response by the nurse? 1. "This is unusual; please come in for an evaluation." 2. "This is an expected side effect of the medication." 3. "Are you sure that your eyes have changed color?" 4. "What do you mean that your eyes have changed color?"

Answer: 2 "This is an expected side effect of the medication." Rationale: The iris may change color with the medication; this is expected. The client has said his eye color is changing; it is condescending to ask him if he is sure. Asking the client what he means by saying his eyes have changed color does not make sense; he means his eyes have changed color. It is not unusual for the eyes to change color, this is an expected reaction.

34.3 The client tells the nurse that the doctor told him his antibiotic did not kill his infection, but just slowed its growth. The client is anxious. What is the best response by the nurse to decrease the client's anxiety? 1. "This is okay because your infection is not really that serious." 2. "This is okay because your body will help kill the infection too." 3. "This is okay because your doctor is an infectious disease specialist." 4. "This is okay because your blood work is being monitored daily."

Answer: 2 "This is okay because your body will help kill the infection too." Rationale: Some drugs do not kill the bacteria, but instead slow their growth and depend on the body's natural defenses to dispose of the microorganisms. These drugs which slow the growth of bacteria are called bacteriostatic. Telling a client with an infection that the infection is not serious will increase anxiety because to the client, all infections are serious. Telling the client that the doctor is a specialist does not answer the question and will increase anxiety. Telling the client that blood work is being monitored does not answer the question and will increase anxiety.

36.11 The client with acquired immune deficiency syndrome (AIDS) asks the nurse why he must take so many medications. What is the best response by the nurse? 1. "To provide you with the most effective treatment for your illness." 2. "To decrease the possibility of the virus developing resistance to the medications." 3. "Because the earlier we start multiple medications the better for you." 4. "Research has shown single medications to be ineffective."

Answer: 2 "To decrease the possibility of the virus developing resistance to the medications." Rationale: Decreasing the possibility of resistance is the key; single drugs can be effective, but viral resistance is a problem. Multiple medications will provide the best treatment, but this is not as good an answer as avoiding drug resistance. It is debatable if multiple drugs should be used early in the course of the disease. Single medications can be effective, but drug resistance is more likely the reason.

35.7 The client receives nystatin (Nilstat) for a fungal infection in the mouth. The nurse plans to do medication education prior to discharge. What will the best plan of the nurse include? 1. Take the oral suspension with a straw to prevent tooth discoloration. 2. Dissolve the oral tablet in your mouth and then swallow it. 3. Crush the oral tablet, mix it with orange juice, and then swallow it. 4. Swallow the oral tablet whole without chewing or crushing it.

Answer: 2 Dissolve the oral tablet in your mouth and then swallow it. Rationale: Dissolving the tablet in the mouth allows contact of the medication with the organisms causing the infection. The medication does not cause tooth discoloration. The medication must remain in the mouth as long as possible, so it should not be crushed and mixed with liquids. The medication must remain in the mouth as long as possible, so it should not be swallowed whole.

49.4 The client tells the nurse that when he uses his eye drops they sting his eyes. What is the best response by the nurse? 1. "You should wash your eyes immediately with saline." 2. "Your eye drops may have expired; check the date." 3. "This is a normal and expected effect of the drops." 4. "Hold the next dose and contact your physician."

Answer: 3 "This is a normal and expected effect of the drops." Rationale: It is normal for eye drops to cause stinging when used. Washing the eye with saline will dilute the eye drops; this is not recommended. The drops should cause stinging, it means they have not expired. The physician does not need to be contacted; stinging is a normal effect of the eye drops.

49.7 The client receives medications for glaucoma. She asks the nurse what over-the-counter (OTC) medication she can take for her allergic rhinitis. What is the best response by the nurse? 1. "You can take an antihistamine, but avoid decongestants." 2. "You can take a decongestant, but avoid antihistamines." 3. "You will need to avoid antihistamines as well as decongestants." 4. "You should use nasal sprays instead of oral preparations."

Answer: 3 "You will need to avoid antihistamines as well as decongestants." Rationale: Antihistamines as well as decongestants can interact with glaucoma treatment, the client must avoid both. The client must avoid antihistamines as well as decongestants. The client must avoid antihistamines as well as decongestants. Nasal sprays will also interact with the systemic effects of the eye drops for glaucoma, so they must be avoided.

34.4 The nurse works on an infectious disease unit. What is the best understanding of the nurse about the major role of the nurse on this unit? 1. Monitoring liver and renal function tests. 2. Ensuring that all antibiotic medications are given on time. 3. Educating the client about the illness and antibiotic therapy. 4. Ensuring that everyone entering the clients' rooms wash their hands.

Answer: 3 Educating the client about the illness and antibiotic therapy. Rationale: Nurses have many roles on an infectious disease unit. The role of the nurse that will best optimize client outcomes includes education about the illness and education about precisely following the antibiotic protocol. Monitoring liver and renal function tests is important, but if the client is not well educated, treatment may not be successful. Giving antibiotics on time is very important, but the client is likely to continue antibiotics at home and must be educated about the protocol. Ensuring that anyone entering a client's room washes their hands is the best way to prevent nosocomial infections, but does not mean the individual client's treatment will be successful.

36.4 The client receives acyclovir (Zovirax) for treatment of genital herpes. What is a priority assessment by the nurse? 1. Respiratory distress 2. Thrombocytopenia 3. Increased serum creatinine 4. Auditory and visual hallucinations

Answer: 3 Increased serum creatinine Rationale: Acyclovir (Zovirax) is nephrotoxic, so serum creatinine should be monitored. Kidney failure, not respiratory distress, is an adverse effect of acyclovir (Zovirax). Kidney failure, not bone marrow suppression, is an adverse effect of acyclovir (Zovirax). Kidney failure, not auditory and visual hallucinations, is an adverse effect of acyclovir (Zovirax).

34.8 The client receives gentamicin (Garamycin) intravenous (IV) in the clinical setting. What is a priority nursing action? 1. Place the client on isolation precautions 2. Draw daily blood chemistries 3. Monitor the client for hearing loss 4. Increase the fluids for the client during therapy

Answer: 3 Monitor the client for hearing loss Rationale: Aminoglycocides are ototoxic drugs, and the client should be monitored for hearing loss. Isolation is determined by the causative organism, not the drug used for treatment. Serum levels of the drug are indicated, but not blood chemistries. Increasing fluids during therapy is not indicated.

49.14 The client wears contact lenses and has been prescribed eye drops. What will the best education by the nurse include with regard to contact lenses? 1. Instill the drops with the contacts in as long as they are the soft kind of contacts. 2. Eyeglasses must be worn for as long as the client must have the eye drops. 3. Remove lenses before instilling eye drops; do not reinsert lenses for 15 minutes. 4. Instill the drops with the contacts in as long as they are the hard kind of contacts.

Answer: 3 Remove lenses before instilling eye drops; do not reinsert lenses for 15 minutes. Rationale: The client should remove lenses before instilling eye drops and not re-insert them for 15 minutes. Drops should not be instilled while the client is wearing contact lenses, regardless of the kind of lens. Eyeglasses do not need to be worn for as long as the client must have eye drops. Drops should not be instilled while the client is wearing contact lenses, regardless of the kind of lens.

36.10 The client has just begun highly active antiretroviral therapy (HAART) therapy for the treatment of acquired immune deficiency syndrome (AIDS). Which teaching point is a priority for this client? 1. The goal of highly active antiretroviral therapy (HAART) therapy is to reduce plasma human immunodeficiency virus (HIV) ribonucleic acid (RNA) to the lowest possible level. 2. Knowing which medications target which phases of the human immunodeficiency virus (HIV) replication cycle. 3. Taking medications as scheduled is vital to successful treatment. 4. Medications must be taken for three years after viral load is not measurable.

Answer: 3 Taking medications as scheduled is vital to successful treatment. Rationale: Taking medicines, as scheduled, is vital to maintain adequate treatment and prevent resistance to the medication. The goal of highly active antiretroviral therapy (HAART) is to reduce plasma human immunodeficiency virus (HIV), but this is not as important as medication compliance. The client may be interested in knowing which medications target which phases of the human immunodeficiency virus (HIV) ribonucleic acid (RNA) reproduction cycle, but this is not as important as medication compliance. Medications must be continued for the lifetime of the client.

36.13 The client has acquired immune deficiency syndrome (AIDS) and has just learned she is pregnant. She tearfully asks the nurse if her baby will die of acquired immune deficiency syndrome (AIDS). What is the best outcome for this client? 1. The client will take penciclovir (Denavir) as prescribed. 2. The client will take indinavir (Crixivan) as prescribed. 3. The client will take zidovudine (Retrovir) as prescribed. 4. The client will take tipranavir (Aptivus) as prescribed.

Answer: 3 The client will take zidovudine (Retrovir) as prescribed. Rationale: A regime of oral zidovudine (Retrovir) will decrease the possibility of mother-to-baby transmission by 70%. Zidovudine (Retrovir), not penciclovir (Denavir), will decrease the possibility of mother-to-baby transmission by 70%. Zidovudine (Retrovir), not indinavir (Crixivan), will decrease the possibility of mother-to-baby transmission by 70%. Zidovudine (Retrovir), not tipranavir (Aptivus), will decrease the possibility of mother-to-baby transmission by 70%.

49.3 The client is scheduled for an eye exam. Prior to the exam the physician will put a cycloplegic drug, atropine sulfate (Isopto Atropine) eye drops in the client's eyes. What will the nurse teach the client about these eye drops? 1. The drops will dilate the pupil so the physician can better visualize the retina during examination. 2. The drugs will dilate the pupil and lubricate the eye to provide additional comfort during the examination. 3. The drops dilate the pupil and paralyze the ciliary muscle to prevent the lens from moving during examination. 4. The drops will paralyze the ciliary muscle so the eye will not move during the examination.

Answer: 3 The drops dilate the pupil and paralyze the ciliary muscle to prevent the lens from moving during examination. Rationale: Cycloplegic eye drops dilate the pupil and paralyze the ciliary muscle. Cycloplegic eye drops not only dilate the pupil, but also paralyze the ciliary muscle which is necessary for visualization of the retina. Cycloplegic eye drops dilate the pupil and paralyze the ciliary muscle; they do not lubricate the eye. Cycloplegic eye drops will paralyze the ciliary muscle, but they also dilate the pupil, which is necessary for the exam.

49.8 The client has excess cerumen in his ears. What will the best plan of the nurse include as to the safe removal of the cerumen? 1. Instill 2% acetic acid in each ear. 2. Use a sterile Q-tip to remove cerumen. 3. Use warm water and a bulb syringe. 4. Take hot showers to facilitate drainage.

Answer: 3 Use warm water and a bulb syringe. Rationale: Warm water and a bulb syringe is the best method for cerumen removal. Acetic acid is not the best method for cerumen removal. A Q-tip is dangerous as it can pack the cerumen into the ear canal. Hot showers will not facilitate drainage of cerumen.

36.8 The nurse provides care for clients with acquired immune deficiency syndrome (AIDS). Which laboratory test is the best indicator of effective treatment with antiviral medications? 1. T4 lymphocyte count 2. CD4 count 3. Viral load 4. Absolute neutrophil count

Answer: 3 Viral load Rationale: The viral load is an actual count of viral presence, and is the best test. The viral load, not a T4 lymphocyte count, is the best test. The viral load, not a CD4 count, is the best test. The viral load, not an absolute neutrophil count, is the best test.

34.14 The physician orders penicillin for a female client who has a sinus infection. What is a priority question to ask the client prior to administering the medication? 1. "Are you pregnant?" 2. "Are you breast-feeding?" 3. "Do you plan to become pregnant?" 4. "Are you taking birth control pills?"

Answer: 4 "Are you taking birth control pills?" Rationale: Penicillin can cause birth control pills to lose their effectiveness. Penicillin is a Pregnancy Category B drug, and is safe to take if a client is pregnant. Penicillin is a Pregnancy Category B drug, and is safe to take if a client is breast-feeding. Penicillin is a Pregnancy Category B drug, and is safe to take if a client plans to become pregnant.

35.15 The client receives terbinafine (Lamisil) as treatment for a fungal infection of the toenails. What is an important assessment question for the nurse to ask? 1. "How do you clean and clip your toenails?" 2. "Do you have diabetes mellitus?" 3. "Are you HIV-positive?" 4. "What analgesic do you take for headaches?"

Answer: 4 "What analgesic do you take for headaches?" Rationale: Clients must be instructed not to use acetaminophen (Tylenol) while receiving terbinafine (Lamisil), because terbinafine (Lamisil) is hepatotoxic and so is acetaminophen (Tylenol) in large enough quantities. How the client cleans and clips his toenails is not significant. Terbinafine (Lamisil) does not affect glucose levels. Terbinafine (Lamisil) is not contraindicated for a client who is HIV-positive.

34.5 The client receives multiple antibiotics to treat a serious infection. What will the priority assessment of the client by the nurse include? 1. Assessing blood cultures for the presence of bacteria 2. Assessing renal and liver function tests 3. Assessing whether or not the client has adequate food and fluid intake 4. Assessing for diarrhea, and difficult or painful urination

Answer: 4 Assessing for diarrhea, and difficult or painful urination Rationale: A superinfection occurs when microorganisms normally present in the body, host flora, are destroyed by antibiotic therapy. A superinfection can be lethal and should be suspected if a new infection appears while the client is receiving antibiotics. Signs of superinfection commonly include diarrhea, difficult or painful urination, and abnormal vaginal discharges. Assessing blood cultures is important, but not as important as assessing for superinfections. Assessing renal and liver function tests is very important, but not as important as assessing for superinfections. Assessing food and fluid intake is very important, but not as important as assessing for superinfections.

36.7 The physician orders amantadine (Symmetrel) for a client with influenza. Which part of the client's health history would the nurse report to the physician prior to administering amantadine (Symmetrel)? 1. Allergy to shellfish 2. Diabetes mellitus 3. Chronic obstructive pulmonary disease (COPD). 4. Epilepsy

Answer: 4 Epilepsy Rationale: Amantadine (Symmetrel) is associated with seizures, and should not be administered to a client with epilepsy. An allergy to shellfish is not a contraindication to the use of amantadine (Symmetrel). Diabetes mellitus is not a contraindication to the use of amantadine (Symmetrel). Chronic obstructive pulmonary disease (COPD) is an indication for amantadine (Symmetrel) because influenza could be fatal in this client.

34.9 The client has MRSA and receives vancomycin (Vancocin) intravenously (IV). The nurse assesses an upper body rash and decreased urine output. What is the nurse's priority action? 1. Obtain a stat X-ray, and notify the physician. 2. Obtain a sterile urine specimen, and notify the physician. 3. Administer an antihistamine, and notify the physician. 4. Hold the next dose of vancomycin (Vancocin), and notify the physician.

Answer: 4 Hold the next dose of vancomycin (Vancocin), and notify the physician. Rationale: Upper body rash and decreased urine output are most likely symptoms of vancomycin (Vancocin) toxicity, so the medication should be held and the physician notified. There is no reason to obtain a chest x-ray. The client's symptoms are most likely not due to a urinary tract infection, so a sterile urine specimen is not indicated. An antihistamine would help if the problem was an allergy, but an allergy would not cause a decrease in urine output.

35.18 Treatment for malaria involves: a. Single-drug therapy. b. Dietary restrictions. c. Multi-drug therapy. d. Topical drug therapy.

Answer: C Multi-drug therapy. Rationale: A. The treatment of choice is multi-drug therapy. B. Dietary restrictions are not treatment. C. Multi-drug therapy is the treatment of choice due to the life cycle of the protozoan parasite. D. Topical therapy would be ineffective due to the infestation of the parasite in the red blood cells.

49.11 The client is to receive eye drops for glaucoma. What is the correct method of administration? 1. Place the drop in the center of the eye. 2. Place the drop so it falls on the white part of the eye. 3. Turn the head to the side so that the drop flows to the outer corner. 4. Place the drop in the conjunctival sac below the eye.

Answer: 4 Place the drop in the conjunctival sac below the eye Rationale: The correct method for administering eye drops is to place the drop in the conjunctival sac below the eye. The eye drop should be placed in the conjunctival sac below the eye, not in the center of the eye. The eye drop should be placed in the conjunctival sac below the eye, not one the sclera. The head should not be turned to the side when administering eye drops.

36.5 The client receives acyclovir (Zovirax) as treatment for herpes simplex type 1 virus (genital herpes). What is the best, expected outcome for this client? 1. The client will prevent a reoccurrence of infection in the affected area. 2. The client will report a decrease in the number of lesions in the affected area. 3. The client will identify the names of sexual contacts in the past month. 4. The client will report decreased pain using the approved pain scale.

Answer: 4 The client will report decreased pain using the approved pain scale. Rationale: Pain is a major problem associated with this infection; the best outcome is decreased pain. It is not possible to prevent a reoccurrence of herpes simplex type 1 virus. Decreased lesions are not an outcome of treatment with acyclovir (Zovirax). It is not necessary to provide the names of sexual contacts, and this is not related to acyclovir (Zovirax).

A client has been on an antibiotic for two weeks for treatment of an ulcer caused by Helicobacter pylori. The client asks the nurse why a superinfection is caused by this medication. The nurse responds: a. "This is a secondary infection due to "Candida." b. "The infection has developed immunity to the current drug." c. "The infection has become severe." d. "The infection has a restricted group of microorganisms."

Answer: A "This is a secondary infection due to "Candida." Objective: Explain the importance of culture and sensitivity testing to anti-infective chemotherapy. Rationale: A superinfection is a side effect of antibiotic therapy. The antibiotic destroys the body's normal flora, resulting in another infection.

36.19 Which of the following laboratory tests best evaluates HIV disease? a. CD4 count b. Complete blood count c. Platelets d. Liver function studies

Answer: A CD4 count Rationale: A. CD4 count best determines the progress of the disease. B. Complete blood count can be used, but does not assess the progress. C. Platelets are not indicated. D. Liver function studies can be used, but do not assess the progress.

34.23 Following surgery, a client is placed on cefotaxine (Claforan). The assessment for possible adverse effects should include observing for: a. Diarrhea. b. Tachycardia. c. Constipation. d. Headache.

Answer: A Diarrhea. Rationale: A. Diarrhea is a frequent adverse effect of cephalosporins. B. Tachycardia is not an adverse effect. C. Diarrhea, not constipation, is a common problem. D. Headache is not an adverse effect.

34.18 Discharge planning for the client prescribed tetracycline will include which of the following? a. Do not take the medication with milk. b. Decrease the amount of vitamins. c. Take the medication with antacids. d. Take the medication with iron supplements.

Answer: A Do not take the medication with milk. Rationale: A. Tetracycline effectiveness can be decreased by using milk products. B. It is not necessary to decrease vitamins. C. Antacids can decrease the effectiveness of tetracycline. D. Iron can decrease the effectiveness of tetracycline.

49.18 Which of the following is an adverse effect of prostaglandins, such as latanoprost (Xaltan)? a. Eye pain b. Hypotension c. Tachycardia d. Nausea

Answer: A Eye pain Rationale: A. Eye pain, burring, and dry eyes are adverse effects of prostaglandins. B. Hypotension is an adverse effect of beta-adrenergic blockers. C. Tachycardia is an adverse effect of carbonic anhydrase inhibitors. D. Nausea is not an adverse effect.

49.20 The client is preparing for an eye examination. Which of the following classification of drugs will be used? a. Mydriatics b. Beta-adrenergic blockers c. Vasoconstrictors d. Prostaglandins

Answer: A Mydriatics Rationale: A. Mydriatics dilate the pupil for better visualization. B. Beta-adrenergic blockers are used for glaucoma. C. Vasoconstrictors are used for irritation and dryness of the eye. D. Prostaglandins are used for open-angle glaucoma.

36.20 The nurse is caring for a client receiving mesylate (Fortovase). The nurse would observe for adverse effects of: a. Nausea. b. Dizziness. c. Increased urinary output. d. Constipation.

Answer: A Nausea. Rationale: A. Nausea is a common adverse effect of mesylate (Fortovase), a protease inhibitor, and is used in combination with other drugs. B. Dizziness is not an adverse effect that could occur while receiving mesylate (Fortovase). C. Increased urinary output is not an adverse effect. D. Diarrhea, not constipation, is a common adverse effect with mesylate (Fortovase).

36.22 The nurse is preparing to discharge a client on an antiretroviral agent. Which of the following should the nurse include? a. Practice good handwashing. b. Take your blood pressure daily. c. Take your pulse daily. d. Weigh yourself daily.

Answer: A Practice good handwashing. Rationale: A. Handwashing is a technique the client and family members can use to control infection. B. Taking a blood pressure daily temperature is not necessary. C. Taking the pulse is not necessary. D. Daily weight is not necessary.

A nursing intervention for administering sulfamethoxazole-trimethoprim (Bactrim) to a client is to: a. Have the client drink a full glass of water with the medicine. b. Have the client drink a glass of milk. c. Have the client take the medicine with solid foods. d. Have the client take the medicine on an empty stomach.

Answer: A. Have the client drink a full glass of water with the medicine. Objective: Use the nursing process to care for clients who are receiving drug therapy for bacterial infections. Rationale: An adverse effect of this drug is the formation of crystals in the urine.

36.21 The primary purpose of agents for herpesviruses is to: a. Inhibit HIV protease. b. Bind directly to reverse transcriptase DNA. c. Prevent viral DNA synthesis. d. Create a defective DNA strand.

Answer: C Prevent viral DNA synthesis. Rationale: A. Protease inhibitors for HIV inhibit HIV protease. B. Non-nucleoside reverse transcriptase inhibitors for HIV bind directly to reverse transcriptase DNA. C. Herpesvirals prevent viral DNA synthesis. D. Nucleoside and nucleotide reverse transcriptase inhibitors create a defective DNA strand.

A nurse is preparing to administer a broad-spectrum antibiotic medication to a client. An important nursing intervention prior to administration regarding anti-infectives is: a. Obtaining the culture report before starting any medication. b. Performing a culture within 24 hours after starting the medication. c. Performing the culture for evidence before administering the first dose of the anti-infective. d. Administering medicine, and omitting performing cultures.

Answer: A. Performing the culture for evidence before administering the first dose of the anti-infective. Objective: Describe the nurse's role in the pharmacologic management of bacterial infections. Rationale: The nurse should obtain specimens for culture and sensitivity before the start of antibiotic therapy. Laboratory tests should be performed before beginning the anti-infective therapy, but the organism does not have to be identified before starting medication.

35.16 Superficial fungal infections differ from systemic fungal infections in that superficial fungal infections: a. Are less common. b. Affect hair and skin. c. Affect internal organs. d. Can be fatal.

Answer: B Affect hair and skin. Rationale: A. Systemic fungal infections are less common than are superficial ones. B. Superficial fungal infections affect hair, skin, nails, and mucous membranes. C. Systemic fungal infections affect internal organs. D. Systemic fungal infections can be fatal.

49.16 The nurse is counseling a client with glaucoma. The nurse explains that, if left untreated, the condition can lead to: a. Diabetes mellitus. b. Blindness. c. Nearsightedness. d. Myopia.

Answer: B Blindness. Rationale: A. Diabetes mellitus is a risk factor leading to glaucoma. B. Glaucoma can lead to optic nerve damage and blindness. C. Nearsightedness is a risk factor for glaucoma. D. Myopia is the same as nearsightedness, which is a risk factor for glaucoma

34.20 Which of the following laboratory tests will be performed to determine whether a specific bacterium is resistant to a specific drug? a. Complete blood count b. Culture and sensitivity test c. Urinalysis d. Blood urea nitrogen

Answer: B Culture and sensitivity test Rationale: A. Complete blood count would not determine the specific drug for the specific organism. B. Culture and sensitivity is the examination for a specific organism, and can determine the correct medication. C. Urinalysis would not determine the specific drug for the specific organism. D. Blood urea nitrogen would not determine the specific drug for the specific organism.

49.17 Which of the following best describes open-angle glaucoma? a. It is unilateral with a rapid onset. b. It is bilateral with a slow onset. c. It is less common than closed-angle glaucoma. d. It is accompanied by eye pain.

Answer: B It is bilateral with a slow onset. Rationale: A. Closed-angle glaucoma is unilateral with rapid onset. B. Open-angle glaucoma is bilateral, and has a slow onset over several years. C. Open-angle glaucoma is more common. D. Open-angle glaucoma, in contrast to closed angle glaucoma, has no symptoms.

34.19 The nurse is caring for a client receiving gentamicin IV. The nurse would observe for adverse effects of: a. Diarrhea. b. Ototoxicity. c. Increased urinary output. d. Nausea.

Answer: B Ototoxicity. Rationale: A. Diarrhea is not a common adverse effect of gentamicin. B. Ototoxicity is an adverse effect that could occur while receiving gentamicin. This could become permanent with continued use. C. Increased urinary output is not an adverse effect. D. Nausea is not a common adverse effect.

49.19 The primary mechanism of action of beta-adrenergic blockers in the use of open-angle glaucoma is to: a. Increase the outflow of aqueous humor. b. Reduce production of aqueous humor. c. Dilate the pupil to increase outflow. d. Constrict the pupil.

Answer: B Reduce production of aqueous humor. Rationale: A. Prostaglandins increase the outflow of aqueous humor. B. Beta-adrenergic blockers decrease the production of aqueous humor. C. Carbonic anhydrase inhibitors dilate the pupil to increase outflow. D. Cholinergic agonists constrict the pupil to allow for more room for outflow.

36.17 The purpose of general pharmacotherapy for human immune virus (HIV)/AIDS is to: a. Cure the disease. b. Relieve symptoms for a longer period of time. c. Kill the virus. d. Decrease viral load.

Answer: B Relieve symptoms for a longer period of time. Rationale: A. Anti-retroviral drugs cannot cure the disease B. The purpose is to remain symptom-free for longer. C. Anti-retroviral drugs will not kill the virus. D. Viral load is not decreased.

34.17 What is the action of bactericidal drugs? a. They will slow the slow growth of the bacteria. b. They will kill the bacteria. c. They have a high potency. d. They disrupt normal cell function.

Answer: B They will kill the bacteria. Rationale: A. Bacteriostatic drugs slow the growth of bacteria. B. Bactericidal drugs kill the bacteria. C. Potency is related to the properties of resistance. D. They do not disrupt normal cell function.

A client has been discharged with a prescription for penicillin. Discharge instructions include that: (Select all that apply.) a. Penicillin can be taken while breastfeeding. b. The entire prescription must be finished. c. All penicillin can be taken without regards to eating. d. Some possible side effects include abdominal pain and diarrhea.

Answer: B. The entire prescription must be finished. Objective: Use the nursing process to care for clients who are receiving drug therapy for bacterial infections. Rationale: Many people will discontinue medication after improvement is noted. All antibiotics must be completed in their entirety to prevent reoccurrence of infection.

35.17 Which client is at the greatest risk of acquiring a serious fungal infection? a. A teenager with no health problems b. A client with diabetes mellitus c. A client who is immunosuppressed d. A client with a history of heart problems

Answer: C A client who is immunosuppressed Rationale: A. A teenager with no health problems is most likely resistant to a serious fungal infection. B. A client with diabetes mellitus can acquire an infection, but most likely not a serious one. C. Clients who are immunosuppressed, such as HIV or organ transplant clients, are very non-resistant to serious infections. D. A client with heart problems most likely is resistant to a serious fungal infection.

49.21 The drug most likely to be used to soften or eliminate cerumen is: a. An antifungal agent. b. A steroid. c. A wax softener. d. A local analgesic.

Answer: C A wax softener. Rationale: A. An antifungal would not be used for cerumen. B. Steroids would not be used for cerumen, but for inflammation. C. Wax softeners are used to soften or eliminate cerumen. D. Local analgesics would be used for otitis external.

36.18. The drug most likely to be ordered for the client with herpes simplex virus is which of the following? a. Methonidazole (Flagyl) b. Nystatin (Fungizone) c. Zidovudine (Retrovir) d. Acyclovir (Zovirax)

Answer: D Acyclovir (Zovirax) Rationale: A. Methonidazole is an antifungal medication. B. Nystatin is an antifungal medication. C. Zidovudine is an antiviral mainly used in the treatment of HIV. D. The drug acyclovir (Zovirax) is an antiviral drug used for herpes simplex virus and influenza.

35.19 The nurse is preparing to administer an antifungal medication\to an adult client. The nurse will administer the medication: a. With water. b. With milk. c. With other medications. d. Alone, and will allow the client to swish and swallow the medication.

Answer: D Alone, and will allow the client to swish and swallow the medication. Rationale: A. The client should wait at least 10 minutes after antifungal treatment to put anything else in the mouth. B. The client should wait at least 10 minutes after antifungal treatment to put anything else in the mouth. C. Other medications should be given before or after the nystatin. D. Medication should be swished and swallowed and swished and spit without anything else in the mouth.

34.21 The drug that would most likely be used in the treatment of tuberculosis is: a. Erythromycin (E-mycin). b. Gentamicin (Garamycin). c. Vancomycin (Vancocin). d. Isoniazid (INH).

Answer: D Isoniazid (INH). Rationale: A. Erythromycin is most effective against gram-positive bacteria. B. Gentamicin is used for bactericidal reasons. C. Vancomycin used for bactericidal reasons. D. Isoniazid (INH) Is the drug of choice for anti-tuberculosis therapy.

34.22 Treatment of tuberculosis usually involves: a. The use of a single drug. b. Keeping the client hospitalized. c. Surgical removal of tubercular lesions. d. The use of two or more drugs at the same time.

Answer: D The use of two or more drugs at the same time. Rationale: A. Multi-drug therapy is usually the treatment, so use of a single drug is not usual. B. It is not necessary to keep the client in the hospital. C. Surgery is not the treatment. D. Multi-drug therapy for 6-12 months is the usual

Before taking chloroquine (Aralen), the client should be instructed that: a. This is the only cure known for malaria. b. His blood pressure should be monitored for 60 minutes after taking the medicine. c. Daily weights will need to be taken every morning. d. A CBC will need to be performed.

Answer: D. A CBC will need to be performed. Objective: Use the nursing process to care for clients receiving drug therapy for fungal, protozoal, and helminth infections. Rationale: This medicine can precipitate anemia in clients with 6GPD deficiency, and a CBC test should be performed.

The client asks the nurse why the physician didn't prescribe the same antibiotic that he always takes for an infection. The best response by the nurse would be: a. "It doesn't matter which antibiotic is taken." b. "Try this medicine, and if you're not better in 10 days, return to the office." c. "You don't want to take the same antibiotic all the time." d. "Bacteria can become resistant to some antibiotics."

Answer: D. "Bacteria can become resistant to some antibiotics." Objective: Using a specific example, explain how resistance can develop to an anti-infective drug. Rationale: Antibiotics promote the development of drug-resistant bacterial strains by killing populations of bacteria that are sensitive to the drug. The only bacteria remaining are those cells that possess mutations that made them insensitive to the effects of the antibiotic.

A client has been diagnosed with methicillin-resistant Staphylococcus aureus (MRSA) infection. While his medicine is being administered, he asks questions regarding his treatment with vancomycin. The nurse's appropriate response would be: a. "After starting the medicine regimen, you are considered to be no longer contagious." b. "The majority of antibacterial drugs are used exclusively for MRSA." c. "Drug therapy for MRSA does not differ from treating other infections." d. "It is necessary to continue IV therapy infusion for at least two hours for each dose and watch for nephrotoxicity."

Answer: D. "It is necessary to continue IV therapy infusion for at least two hours for each dose and watch for nephrotoxicity." Objective: Use the Nursing Process to care for clients who are receiving drug therapy for bacterial infections. Rationale: Vancomycin is administered orally and IV. A reaction that can occur with rapid IV administration is known as red man syndrome, and it includes hypotension with flushing and a red rash on the face and upper body.

Important information to include in the client's education regarding taking aminoglycosides is that: a. The drug can cause discoloration of teeth. b. Fluid intake should be decreased to prevent retention. c. This drug primarily is given orally, because it is absorbed in the GI tract. d. A serious side effect is hearing loss.

Answer: D. A serious side effect is hearing loss. Objective: Use the nursing process to care for clients who are receiving drug therapy for bacterial infections. Rationale: This anti-infective is noted for toxic effects on kidneys and vestibular apparatus. Clients should be monitored for ototoxicity and nephrotoxicity during and after therapy. Hearing loss can occur after therapy.

A client has been prescribed tetracycline. When providing information regarding this drug, the nurse would be correct in stating that tetracycline: a. Is classified as a narrow-spectrum antibiotic. b. Is used to treat a wide variety of disease processes. c. Has been identified to be safe during pregnancy. d. Is contraindicated in children under 8 years of age.

Answer: D. Is contraindicated in children under 8 years of age. Objective: For each of the drug classes listed in Drugs at a Glance, list representative drug examples, and explain their mechanisms of action, primary actions, and important adverse effects. Rationale: This drug has the ability to cause permanent mottling and discoloration of teeth, and therefore it is not good for children under 8 years of age.

A client has been prescribed ciprofloxacin (Cipro). Important information that the nurse must know includes: a. This medicine must be taken on an empty stomach to increase absorption. b. This medicine is classified as an aminoglycoside and is given for systemic bacterial infections. c. This medicine should be given with an antacid to increase the absorption and effectiveness of the medicine. d. This medicine should not be given with the ordered multivitamin.

Answer: D. This medicine should not be given with the ordered multivitamin. Objective: For each of the drug classes listed in Drugs at a Glance, list representative drug examples, and explain their mechanisms of action, primary actions, and important adverse effects. Rationale: Fluoroquinolones should not be given with multivitamins or mineral supplements, because they can reduce absorption of the antibiotic by as much as 90%.

A client has been prescribed thiabendazole (Mintezol) for a helminth infestation. The nurse should include in her teaching: a. To chew the medicine and take it with a fatty meal for the greatest effect. b. To be on a low-fat diet. c. To take stool softeners while on the medicine. d. To increase fruit juices so that there will be an acidic environment to increase absorption.

Answer: a. To chew the medicine and take it with a fatty meal for the greatest effect. Objective: Use the nursing process to care for clients receiving drug therapy for fungal, protozoal, and helminth infections. Rationale: Chewing thiabendazole and taking it with fatty foods increase absorption.

The client asks if metronidazole (Flagyl) can cause any other health problems. The best response by the nurse would be: a. "It can cause only minor abdominal cramping." b. "Report symptoms of rash, pruritus, and numbness throughout the regimen." c. "There have not been any known illnesses as a result of this medication." d. "Not that I am aware of, but you should ask your doctor."

Answer: b. "Report symptoms of rash, pruritus, and numbness throughout the regimen." Objective: For each of the classes shown in Drugs at a Glance, know representative examples, and explain their mechanisms of drug action, primary actions, and important adverse effects. Rationale: Central nervous system toxicity and allergic responses can occur.

A client has been diagnosed with a systemic fungal infection. The physician has prescribed amphotericin B (Fungizone). The nurse will include which of the following in her client education? a. "Drug therapy will be for a very short time, probably 2-4 weeks." b. "Carefully inspect all intramuscular injection sites for bruising." c. "Notify the physician should you come down with symptoms of increased weight gain or vertigo." d. "Stop the medication if you're feeling better."

Answer: c. "Notify the physician should you come down with symptoms of increased weight gain or vertigo." Objective: Use the nursing process to care for clients receiving drug therapy for fungal, protozoal, and helminth infections. Rationale: Amphotericin B (Fungizone) causes some degree of kidney damage in 80% of patients who take it; therefore, the nurse should teach the client to monitor weight and to report hearing loss, vertigo, or unsteady gait.

When administering fluconazole (Diflucan) to a client, the nurse should: a. Have the client chew the tablet. b. Instruct the client to double the dose the next day if one dose is missed. c. Encourage the client to increase fluid intake to a minimum of eight glasses of water a day to prevent crystals in the urine. d. Assess for nausea, vomiting, abdominal pain, or diarrhea.

Answer: d. Assess for nausea, vomiting, abdominal pain, or diarrhea. Objective: For each of the classes shown in Drugs at a Glance, know representative examples, and explain their mechanisms of drug action, primary actions, and important adverse effects. Rationale: GI side effects can occur, and they need careful assessment. The medication should not be chewed. The client should not double the dose if a dose is missed. It does not cause crystals in the urine.

A client has returned from South America but has contracted malaria. The drug the nurse expects to see used is ________________. a. Proguanil (Paludrine) b. Penicillin (Ampicillin) c. Rizatriptan (Maxalt) d. Chlorogrine (Aralin)

Answer: d. Chlorogrine (Aralin) Objective: Categorize drugs used in the treatment of fungal, protozoal, and helminth infections based on their classifications and mechanisms of action. Rationale: Chlorogrine (Aralin) is the classic antimalarial for treating the acute stage.

An adverse reaction to metronidazole (Flagyl) would be assessed by the nurse if: a. The blood pressure is elevated. b. The client complains of hearing loss. c. The client becomes hypothermic. d. The client tells her that he has no appetite.

Answer: d. The client tells her that he has no appetite. Objective: Describe the nurse's role in the pharmacologic management of fungal, protozoal, and helminth infections. Rationale: Anorexia, nausea, diarrhea, dizziness, and headache are common reactions.

A client has been prescribed albendazole (Albenza). While taking this medicine: a. The client should avoid direct sunlight. b. The client will be more susceptible to illnesses and should avoid contact with anyone but the immediate family. c. The client should be weighed daily. d. The client's close contacts will require concurrent treatment.

Answer: d. The client's close contacts will require concurrent treatment. Objective: Identify protozoal and helminth infections that might benefit from pharmacotherapy. Rationale: All others that might be exposed to infestation need treatment to prevent reinfestation.

Medications used to treat HIV infections are more specifically classified as what type of drugs? Antiparasitic Antiviral Antifungal Antiretroviral

Antiretroviral HIV is a member of the retrovirus family; therefore, drugs used to treat this virus are classified as antiretroviral drugs. Although antiretroviral drugs also fall under the broader category of antiviral drugs in general, their mechanisms of action are unique to the acquired immune deficiency syndrome virus. So, they are more commonly referred to by their subclassification as antiretroviral drugs.

The mother of a 4-year-old boy states, "I can't believe my son has type 1 diabetes. We eat well and I was so careful during the pregnancy. What could have caused this?" How should the nurse respond?

Are there others in your family that have type 1 diabetes?" We are not certain what causes type 1 diabetes." "It is thought to be a combination of factors."

Rosemary has been taking Glargine (Lantus) to treat her condition. One of the benefits of Glargine (Lantus) insulin is its ability to: A Release insulin rapidly throughout the day to help control basal glucose. B Release insulin evenly throughout the day and control basal glucose levels. C Simplify the dosing and better control blood glucose levels during the day. D Cause hypoglycemia with other manifestation of other adverse reactions

B Question 20 Explanation: Glargine (Lantus) insulin is designed to release insulin evenly throughout the day and control basal glucose levels.

A client diagnosed with type 1 diabetes receives insulin. He asks the nurse why he can't just take pills instead. What is the best response by the nurse? A "Insulin must be injected because it needs to work quickly." B "Insulin can't be in a pill because it is destroyed in stomach acid." C "Have you talked to your doctor about taking pills instead?" D "I know it is tough, but you will get used to the shots soon."

B Question 9 Explanation: Insulin must be injected because it is destroyed in the stomach acid if taken orally. Telling he will get used to shots does not answer his question and is condescending. Insulin must be injected because it is destroyed in stomach acid if taken orally; the onset of action is not the issue here. The nurse should answer the client's question, not refer him back to the physician.

Which statement made to the nurse by a health care worker assigned to care for a client with human immune deficiency virus (HIV) indicates a breach of confidentiality and requires further education by the nurse? A. "I told family members they need to wash their hands when they enter and leave the room." B. "The other health care worker and I were out in the hallway discussing our concern about getting HIV from our client." C. "Yes, I understand the reasons why I have to wear gloves when I bathe the client." D. "The client's spouse told me she got HIV from a blood transfusion."

B "The other health care worker and I were out in the hallway discussing our concern about getting HIV from our client." Rationale: Discussing this client's illness outside of the client's room is a breach of confidentiality and requires further education by the nurse. Instruction on handwashing to family members or friends is not a breach of confidentiality. Understanding the reasons for wearing gloves recognizes Standard Precautions in direct care and is not a breach of confidentiality. Relaying a direct conversation to the nurse is not a breach of confidentiality.

When preparing a client newly diagnosed with human immune deficiency virus (HIV) and the significant other for discharge, which explanation by the nurse accurately describes proper condom use? A. "Condoms should be used when lesions are present on the penis." B. "Always position the condom with a space at the tip of an erect penis." C. "Make sure it fits loosely to allow for penile erection." D. "Use adequate lubrication, such as petroleum jelly."

B. "Always position the condom with a space at the tip of an erect penis." Rationale: Positioning the condom with a space at the tip of the erect penis allows for the collection of semen at the tip of the condom. Condoms must be used by HIV-infected people at all times for sexual activity, with or without the presence of lesions. Condoms should be applied on an erect penis and should fit snugly, leaving space without air at the tip. Lubricants should be water-based only.

The home health nurse is making an initial home visit to a client currently living with family members after being hospitalized with pneumonia and newly diagnosed with acquired immune deficiency syndrome (AIDS). Which statement by the nurse best acknowledges the client's fear of discovery by his family? A."Do you think that I could post a sign on your bedroom door for everyone about the need to wash their hands?" B. "Is there somewhere private in the home where we can go and talk?" C. "I hope that all of your family members know about your disease and how you need to be protected, because you have been so sick." D. "It is your duty to protect your family members from getting AIDS.

B. "Is there somewhere private in the home where we can go and talk?" Rationale: A nonthreatening approach used initially to find out whether the client has informed family members or desires privacy is very important. The client has a right to privacy and can make the decision whether to post handwashing signs; caution signs invade the client's right to privacy. Protection from infection is important, but stating that the family members should know about the disease is not respectful of the client's right to privacy. The nurse suggesting that it is the client's responsibility to protect his or her family from getting AIDS is an intimidating statement. It is the client's right to make the decision whether to inform family members about his or her illness. However, this "nonaction" could be grounds for a lawsuit if the client were to infect someone inadvertently.

The nursing instructor asks the student nurse to explain a type IV hypersensitivity reaction. Which statement by the student best describes type IV hypersensitivity? A. "It is a reaction of immunoglobulin G (IgG) with the host cell membrane or antigen." B. "The reaction of sensitized T cells with antigen and release of lymphokines activate macrophages and induce inflammation." C. "It results in release of mediators, especially histamine, because of the reaction of immunoglobulin E (IgE) antibody on mast cells." D. "An immune complex of antigen and antibodies is formed and deposited in the walls of blood vessels."

B. "The reaction of sensitized T cells with antigen and release of lymphokines activate macrophages and induce inflammation." Rationale: The reaction of sensitized T cells with antigen and release of lymphokines is a delayed hypersensitivity reaction, as is seen with poison ivy (type IV hypersensitivity). A reaction of IgG with the host cell membrane or antigen describes a type II hypersensitivity reaction. A release of mediators, especially histamine, because of the reaction of IgE antibody on mast cells describes a type I hypersensitivity reaction. An immune complex of antigen and antibodies deposited in the walls of blood vessels describes a type III hypersensitivity reaction.

A male client has just been diagnosed with type 1 diabetes mellitus. When teaching the client and family how diet and exercise affect insulin requirements, Nurse Joy should include which guideline? A. "You'll need more insulin when you exercise or increase your food intake." B. "You'll need less insulin when you exercise or reduce your food intake." C. "You'll need less insulin when you increase your food intake." D. "You'll need more insulin when you exercise or decrease your food intake."

B. "You'll need less insulin when you exercise or reduce your food intake." Exercise, reduced food intake, hypothyroidism, and certain medications decrease the insulin requirements. Growth, pregnancy, greater food intake, stress, surgery, infection, illness, increased insulin antibodies, and certain medications increase the insulin requirements.

An agitated, confused female client arrives in the emergency department. Her history includes type 1 diabetes mellitus, hypertension, and angina pectoris. Assessment reveals pallor, diaphoresis, headache, and intense hunger. A stat blood glucose sample measures 42 mg/dl, and the client is treated for an acute hypoglycemic reaction. After recovery, nurse Lily teaches the client to treat hypoglycemia by ingesting: A. 2 to 5 g of a simple carbohydrate. B. 10 to 15 g of a simple carbohydrate. C. 18 to 20 g of a simple carbohydrate. D. 25 to 30 g of a simple carbohydrate.

B. 10 to 15 g of a simple carbohydrate. To reverse hypoglycemia, the American Diabetes Association recommends ingesting 10 to 15 g of a simple carbohydrate, such as three to five pieces of hard candy, two to three packets of sugar (4 to 6 tsp), or 4 oz of fruit juice. If necessary, this treatment can be repeated in 15 minutes. Ingesting only 2 to 5 g of a simple carbohydrate may not raise the blood glucose level sufficiently. Ingesting more than 15 g may raise it above normal, causing hyperglycemia.

Tonometry is performed on the client with a suspected diagnosis of glaucoma. The nurse analyzes the test results as documented in the client's chart and understands that normal intraocular pressure is: A. 2-7 mmHg B. 10-21 mmHg C. 22-30 mmHg D. 31-35 mmHg

B. 10-21 mmHg Tonometry is the method of measuring intraocular fluid pressure using a calibrated instrument that indents or flattens the corneal apex. Pressures between 10 and 21 mmHg are considered within normal range.

Albert. a 35-year-old insulin dependent diabetic. is admitted to the hospital with a diagnosis of pneumonia. He has been febrile since admission. His daily insulin requirement is 24 units of NPH. Every morning Albert is given NPH insulin at 0730. Meals are served at 0830. 1230. and 1830. The nurse expects that the NPH insulin will reach its maximum effect (peak) between the hours of: A. 1130 and 1330 B. 1330 and 1930 C. 1530 and 2130 D. 1730 and 2330

B. 1330 and 1930

Which antibiotic class is usually reserved for serious UTIs because of the potential for ototoxicity and nephrotoxicity? A. Erythromycins B. Aminoglycosides C. Tetracyclines D. Sulfonamides

B. Aminoglycosides

Which nursing activity can the nurse delegate to a home health aide? A. Changing the dressing for a client with a low absolute neutrophil count B. Assisting with bathing for a client with chronic rejection of a liver transplant C Teaching a client with bacterial pneumonia how to take the prescribed antibiotic D. Assessing incisional tenderness for a client who had a recent kidney transplant

B. Assisting with bathing for a client with chronic rejection of a liver transplant Rationale: Assisting with bathing for a client with chronic rejection of a liver transplant can be delegated to the home health aide. Changing the dressing for a client with a low absolute neutrophil count requires a licensed RN and should not be delegated because of the high risk for infection. Teaching about medications and assessments are within the scope of practice of the RN.

During a class on exercise for diabetic clients, a female client asks the nurse educator how often to exercise. The nurse educator advises the clients to exercise how often to meet the goals of planned exercise? A. At least once a week B. At least three times a week C. At least five times a week D. Every day

B. At least three times a week Diabetic clients must exercise at least three times a week to meet the goals of planned exercise — lowering the blood glucose level, reducing or maintaining the proper weight, increasing the serum high-density lipoprotein level, decreasing serum triglyceride levels, reducing blood pressure, and minimizing stress. Exercising once a week wouldn't achieve these goals. Exercising more than three times a week, although beneficial, would exceed the minimum requirement.

A client with Meniere's disease is experiencing severe vertigo. Which instruction would the nurse give to the client to assist in controlling the vertigo? A. Increase fluid intake to 3000 ml a day B. Avoid sudden head movements C. Lie still and watch the television D. Increase sodium in the diet

B. Avoid sudden head movements The nurse instructs the client to make slow head movements to prevent worsening of the vertigo. Dietary changes such as salt and fluid restrictions that reduce the amount of endolymphatic fluid sometimes are prescribed. Lying still and watching television will not control vertigo.

A client diagnosed with human immune deficiency virus is concerned about getting opportunistic infections and asks the nurse how to prevent them. Which interventions does the nurse recommend to the client? A. Clean toothbrushes once a week. B. Bathe daily using an antimicrobial soap. C. Eat salad at least once a day. D Wash dishes in cool water.

B. Bathe daily using an antimicrobial soap. Rationale: Bathing daily and using an antimicrobial soap will help decrease the risk for opportunistic infections by reducing the number of bacteria found on the skin. Toothbrushes should be cleaned daily through the dishwasher or by rinsing in liquid laundry bleach. Salads and raw fruits and vegetables could be contaminated and should be avoided. Dishes should be washed in hot, soapy water or in a dishwasher.

Which of the following types of antibiotics is more likely to cause superinfections? A. Narrow-spectrum antibiotics B. Broad-spectrum antibiotics C. The original penicillin D. Bacteriostatic drugs

B. Broad-spectrum antibiotics

A nurse is planning care for a client who has Cushing's Syndrome due to chronic corticosteroid use. Which of the following actions should the nurse include in the plan of care? A. Check the client's blood glucose for hypoglycemia B. Check the client's urine specific gravity C. Weigh the client weekly D. Insert an indwelling urinary catheter for the client

B. Check the client's urine specific gravity to assess for FVE

The nurse is assigned to care for four clients. Which client does the nurse assess first? A. Client with human immune deficiency virus (HIV) and Kaposi's sarcoma who has increased swelling of a sarcoma lesion on the right arm B. Client with a history of liver transplantation who is currently taking cyclosporine (Sandimmune) and has an elevated temperature C. Client who has been admitted to receive a monthly dose of serum immune globulin to treat Bruton's agammaglobulinemia D. Client who has been receiving radiation to the abdomen and has a decreased total lymphocyte count

B. Client with a history of liver transplantation who is currently taking cyclosporine (Sandimmune) and has an elevated temperature Rationale: The temperature elevation of the client with a history of liver transplantation indicates that infection may be occurring; the client is at risk for overwhelming infection because of cyclosporine-induced immune suppression. Immediate assessment by the nurse is indicated. Information regarding the HIV-positive client with Kaposi's sarcoma and the client with Bruton's agammaglobulinemia indicates that these clients' physiologic statuses are relatively stable. It is not unusual for a client who is undergoing radiation to have a decreased total lymphocyte count.

The healthcare provider is teaching a group of students about the characteristics of type 1 diabetes mellitus. Which of the following describe the underlying cause of the disease? Please choose from one of the following options. Atrophy of pancreatic alpha cells Destruction of pancreatic beta cells Cellular resistance to insulin Increased hepatic glycogenesis

B. Destruction of pancreatic beta cells Rationale: 1 / 3 The major feature of type 1 diabetes mellitus is an absence of endogenous insulin. Hint #22 / 3 Insulin is produced by beta cells in the pancreas. Hint #33 / 3 The beta cells in a patient with type 1 diabetes mellitus are destroyed, so the patient has no endogenous insulin.

The purpose of culture and sensitivity (C&S) testing is to: A. Prevent an infection, a practice called chemoprophylaxis B. Determine which antibiotic is most effective against the infecting microorganism C. Identify bacteria what have acquired resistance D. Promote the development of drug-resistant bacterial strains by killing the bacteria sensitive to a drug

B. Determine which antibiotics is most effective against the infecting microorganism

Following oral administration, chlorpromazine (Thorazine) is rapidly inactivated by the liver. This inactivation is referred to as: A. Enterohepatic recirculation B. First-pass effect C. Gastric-hepatic barrier D. Enzyme induction

B. First-pass effect

Which factor relates most directly to a diagnosis of primary immune deficiency? A. History of viral infection B. Full-term infant surfactant deficiency C. Contact with anthrax toxin D. Corticosteroid therapy

B. Full-term infant surfactant deficiency Rationale: Genetic mutation causes surfactant deficiency; this is a primary immune deficiency. Viral infection can cause a secondary immune deficiency. Anthrax and medical therapy are examples of a secondary immune deficiency.

Which postoperative kidney transplantation client does the nurse assess first for signs and symptoms of hyperacute rejection? A. Older adult with Parkinson disease receiving a donation from an identical twin B. Grand multipara female with a history of subsequent blood transfusions C. Middle-aged man with a 20-pack-year history D. Young adult with type 1 diabetes

B. Grand multipara female with a history of subsequent blood transfusions Rationale: The grand multipara female with a history of subsequent blood transfusions should be assessed first because multiple pregnancies and blood transfusions greatly increase the risk of a hyperacute rejection. The older adult with Parkinson disease receiving a donation from an identical twin has less chance of hyperacute rejection because his donor is an identical twin. Smoking places the middle-aged man with a 20-pack-year history at higher risk for postoperative respiratory difficulties, but not for hyperacute rejection. Type 1 diabetes requires close postoperative monitoring of blood sugar, but does not predispose the client to a hyperacute rejection.

A client has been ordered norepinephrine (Levophed) for treatment of severe hypotension. The nurse plans to monitor the client for which adverse effect? A. Bradycardia B. Headache C.Infection D. Metabolic alkalosis

B. Headache Rationale: Norepinephrine is a vasopressor and can cause headache; it does not suppress the immune system. Tachycardia and metabolic acidosis are adverse effects of norepinephrine.

The healthcare provider is assessing the glucose level of a patient with a diagnosis of diabetes. Which of these is most helpful in evaluating this patient's long-term glucose management? Please choose from one of the following options. Fasting blood glucose level Hemoglobin A1c Urine specific gravity The patient's food diary

B. Hemoglobin A1c Rationale: 1 / 3 All of these may be part of assessing disease management, but one of these gives the best information. Hint #22 / 3 The healthcare provider will want to measure the patient's "glucose footprint." Hint #33 / 3 Glucose in the blood adheres to hemoglobin, a process called glycosylation. The amount of glycosylation is directly correlated to the amount of glucose in the blood, so the hemoglobin A1c can assess the patient's glucose control over the past 3 months.

The client receives hydrocortisone therapy. The nurse will primarily assess for which electrolyte disturbance? Select one: A. Hypernatremia and hyperkalemia B. Hypernatremia and hyperglycemia C. Hypoglycemia and hyponatremia D. Hypercalcemia and hyperkalemia

B. Hypernatremia and hyperglycemia

A nurse is caring for a client who has type 2 diabetes mellitus and is displaying manifestations of hyperglycemia. Which of the following findings should indicate to the nurse that the client has hyperglycemia? A. Hunger B. Increased urination C. Cold, clammy skin D. Tremors

B. Increased urination Increased urination, or polyuria is a manifestation of hyperglycemia due to a deficiency of insulin which can lead to osmotic diuresis.

Which couple has the highest risk for sexual transmission of HIV without the use of a condom or dental dam? A. Uninfected male performing vaginal intercourse with an infected female B. Infected male performing vaginal intercourse with an uninfected female C. Uninfected male performing anal intercourse with an infected male D. Infected male performing oral sex on an uninfected male

B. Infected male performing vaginal intercourse with an uninfected female Rationale: Sexual acts or practices that permit infected seminal fluid to come into contact with mucous membranes or nonintact skin are the most risky for sexual transmission of HIV. HIV is more easily transmitted from an infected man to an uninfected woman than vice versa. This is because HIV is most easily transmitted when infected body fluids come into contact with mucous membranes or nonintact skin. The vagina has much more mucous membrane than does the penis.

Because of a flu epidemic, the respiratory floor of a hospital does not have any open beds. Which client does the nurse determine is ready for discharge at the request of the discharge planner? A. Older adult client with a history of congestive heart failure, oxygen saturation of 91%, and on O2 at 2 L, with white blood cell count (WBC) 15.5, segmented neutrophils (segs) 8.0, bands 5, lungs with slight crackles in bases, able to assist with activities of daily living, and afebrile B. Middle-aged client with history of multiple sclerosis, decreased ability to ambulate since hospitalization, lungs clear, WBC count 9.5, segs 6.0, bands 1.0, oxygen saturation of 93% on room air, and afebrile C. Young adult client with crackles in all lung lobes, with productive cough of copious amounts of thick yellow sputum, WBC count 20.0, segs 7.0, bands 10.0, oxygen saturation of 95% on O2 at 2 L, and temperature of 100.4° F (38° C) D. Older adult client with recent history of right hip replacement, with productive cough, WBC count 3.4, segs 6.2, bands 5, lungs with crackles right mid-lobe posterior chest wall, oxygen saturation of 89% with O2 at 2 L, and afebrile

B. Middle-aged client with history of multiple sclerosis, decreased ability to ambulate since hospitalization, lungs clear, WBC count 9.5, segs 6.0, bands 1.0, oxygen saturation of 93% on room air, and afebrile Rationale: The middle-aged client with history of multiple sclerosis is ready for discharge; the complete blood count (CBC) is within normal limits. The older adult client with a history of congestive heart failure has elevated WBC and segs and is not ready for discharge. The young adult client with crackles in all lung lobes is not ready for discharge because of elevated WBCs, left shift, and febrile status. The older adult client with recent history of right hip replacement is not ready for discharge because the WBC is below normal (but all other parts of the differential are within normal limits), which could indicate a viral infection, but crackles and low oxygen saturation are still present in the lungs.

Drug therapy of tuberculosis differs from that of most other infections because: A. Patients with tuberculosis have no symptoms B. Mycobacteria have a cell wall that is resistant to penetration by anti-infective drugs C. Patients usually require therapy for a shorter period D. Antituberculosis drugs are used extensively for treating active disease, not preventing it

B. Mycobacteria have a cell wall that is resistant to penetration by anti-infective drugs

Which of these signs suggests that a male client with the syndrome of inappropriate antidiuretic hormone (SIADH) secretion is experiencing complications? A. Tetanic contractions B. Neck vein distention C. Weight loss D. Polyuria

B. Neck vein distention SIADH secretion causes antidiuretic hormone overproduction, which leads to fluid retention. Severe SIADH can cause such complications as vascular fluid overload, signaled by neck vein distention. This syndrome isn't associated with tetanic contractions. It may cause weight gain and fluid retention (secondary to oliguria).

The nurse is preparing to discharge a client on an antiretroviral agent. Which instruction should the nurse include? Select one: A. Take your blood pressure daily. B. Practice good handwashing. C. Weigh yourself daily. D. Take your pulse daily.

B. Practice good handwashing

A client who is human immune deficiency virus positive is experiencing anorexia and diarrhea. Which nursing actions does the nurse delegate to a nursing assistant? A, Collaborate with the client to select foods that are high in calories. B. Provide oral care to the client before meals to enhance appetite. C. Assess the perianal area every 8 hours for signs of skin breakdown. D. Discuss the need to avoid foods that are spicy or irritating.

B. Provide oral care to the client before meals to enhance appetite. Rationale: Providing oral care is within the scope of practice of unlicensed personnel such as nursing assistants. Diet planning, assessment, and client teaching are higher-level actions that require more broad education and scope of practice; these actions should be done by licensed staff.

A client who is exposed to invading organisms recovers rapidly after the invasion without damage to healthy body cells. How has the immune response protected the client? A. Intact skin and mucous membranes B. Self-tolerance C. Inflammatory response against invading foreign proteins D. Antibody-antigen interaction

B. Self-tolerance Rationale : The ability to recognize self versus non-self is necessary to prevent healthy body cells from being destroyed along with the invading organisms. This meets the client's protection needs. The body has some defenses to prevent organisms from gaining access to the internal environment, such as intact skin and mucous membranes; however, they are not perfect—invasion of the body's internal environment by organisms often occurs. Inflammation provides immediate protection against the effects of tissue injury and invading foreign proteins. The inflammatory response is immediate but short-term against injury or invading organisms; it does not provide true immunity. Seven steps (phagocytosis) are needed to produce a specific antibody directed against a specific antigen whenever the person is exposed to that antigen.

The client having an intravenous injection of radiocontrast material (dye) for an angiogram, starts to have skin wheals at the injection site and difficulty breathing. What is the nurse's best first action? A. Administer oxygen by mask or nasal cannula. B. Stop the infusion of the contrast material. C. Prepare an injection of epinephrine. D. Notify the rapid response team.

B. Stop the infusion of the contrast material. Rationale: All actions listed are important. Stopping the infusion of the radiocontrast material while maintaining IV access is critical in limiting the reaction.

When using a Snellen alphabet chart. the nurse records the client's vision as 20/40. Which of the following statements best describes 20/40 vision? A. The client has alterations in near vision and is legally blind. B. The client can see at 20 feet what the person with normal vision can see at 40 feet. C. The client can see at 40 feet what the person with normal vision sees at 20 feet. D. The client has a 20% decrease in acuity in one eye. and a 40% decrease in the other eye

B. The client can see at 20 feet what the person with normal vision can see at 40 feet. The numerator refers to the client's vision while comparing the normal vision in the denominator.

The nurse is assigned to care for a female client with complete right-sided hemiparesis. The nurse plans care knowing that this condition: A. The client has complete bilateral paralysis of the arms and legs. B. The client has weakness on the right side of the body, including the face and tongue C. The client has lost the ability to move the right arm but is able to walk independently. D. The client has lost the ability to move the right arm but is able to walk independently.

B. The client has weakness on the right side of the body, including the face and tongue Hemiparesis is a weakness of one side of the body that may occur after a stroke. Complete hemiparesis is weakness of the face and tongue, arm, and leg on one side. Complete bilateral paralysis does not occur in this condition. The client with right-sided hemiparesis has weakness of the right arm and leg and needs assistance with feeding, bathing, and ambulating.

The client injects his insulin as prescribed but then gets busy and forgets to eat. What is the nurse's most likely assessment finding? Select one: A. The client will be very thirsty. B. The client will have moist skin. C. The client will complain of nausea. D. The client will need to urinate.

B. The client will have moist skin.

The nurse teaches clients with acquired immune deficiency syndrome (AIDS) about the importance of taking their medications as prescribed. What does the nurse recognize as the primary factor for medication noncompliance in the acquired immune deficiency syndrome (AIDS) population? Select one: A. The difficulty with availability of medications to treat acquired immune deficiency syndrome (AIDS) B. The necessity of having to take multiple medications throughout the day C. The weight gain associated with the medications D. A lack of understanding for the reason to take the medications

B. The necessity of having to take multiple medications throughout the day

The nurse is providing care to a client with impaired oxygenation related to anemia. Which nursing intervention has the highest priority? A. Administer antibiotics as prescribed. B. Transfuse ordered packed red blood cells. C.Teach pursed-lip breathing. D. Encourage increased fluid intake.

B. Transfuse ordered packed red blood cells. Rationale: Packed red blood cells increase hemoglobin molecules; this increases sites at which oxygen can attach and improves gas exchange. Antibiotics treat infection; they do not improve oxygenation. Mouth breathing does not improve oxygenation related to anemia. Fluid intake does not have an effect on improving oxygenation.

The nurse is teaching a nursing student about the mechanism by which antimicrobial agents achieve selective toxicity. Which statement by the student indicates a need for further teaching? a. "Some agents disrupt the bacterial cell wall." b. "Some agents act to block the conversion of para-aminobenzoic acid (PABA) to folic acid." c. "Some agents cause phagocytosis of bacterial cells." d. "Some agents weaken the cell wall, causing cell wall lysis."

C

Genevieve has diabetes type 1 and receives insulin for glycemic control. She tells the nurse that she likes to have a glass of wine with dinner. What will the best plan of the nurse for client education include? A The alcohol could cause pancreatic disease. B The alcohol could cause serious liver disease. C The alcohol could predispose you to hypoglycemia. D The alcohol could predispose you to hyperglycemia

C Question 11 Explanation: Alcohol can potentiate hypoglycemic, not hypoglycemic, effects in the client. Alcohol can cause pancreatic disease, but the client's pancreas is not producing any insulin currently. Alcohol can cause liver disease, but the more immediate concern is hypoglycemia.

The nurse is performing an assessment in a client with a suspected diagnosis of cataract. The chief clinical manifestation that the nurse would expect to note in the early stages of cataract formation is: A. Eye pain B. Floating spots C. Blurred vision D. Diplopia

C. Blurred vision A gradual. painless blurring of central vision is the chief clinical manifestation of a cataract. Early symptoms include slightly blurred vision and a decrease in color perception.

Nurse Matt makes a home visit to the client with diabetes mellitus. During the visit, Nurse Matt notes the client's additional insulin vials are not refrigerated. What is the best action by the nurse at this time? A Instruct the client to label each vial with the date when opened. B Tell the client there is no need to keep additional vials. C Have the client place the insulin vials in the refrigerator. D Have the client discard the vials

C Question 13 Explanation: Vials not in use should be refrigerated to preserve drug potency. There is no need to discard the vials. The client should always have additional vials of insulin available. Writing the date of opening on the vial is good practice, but does not address the need to refrigerate additional vials.

Dr. Hugo has prescribed sulfonylureas for Rebecca in the management of diabetes mellitus type 2. As a nurse, you know that the primary purpose of sulfonylureas, such as long-acting glyburide (Micronase), is to: A Induce hypoglycemia by decreasing insulin sensitivity. B Improve insulin sensitivity and decrease hyperglycemia. C Stimulate the beta cells of the pancreas to secrete insulin. D Decrease insulin sensitivity by enhancing glucose uptake

C Question 19 Explanation: Sulfonylureas such as glyburide are used only with patients who have some remaining pancreatic-beta cell function. These drugs stimulate insulin secretion, which reduces liver glucose output and increases cell uptake of glucose, enhancing the number of and sensitivity of cell receptor sites for interaction with insulin.

During a visit in the hospital, the student nurses are asked which of the following persons would most likely be diagnosed with diabetes mellitus. They are correct if they answered a 44-year-old: A Caucasian woman. B Asian woman. C African-American woman. D Hispanic male

C Question 25 Explanation: Age-specific prevalence of diagnosed diabetes mellitus (DM) is higher for African-Americans and Hispanics than for Caucasians. Among those younger than 75, black women had the highest incidence.

After visiting the physician, Angela found out that she has a thyroid problem. In line with her condition, which of the following diagnostic studies is done to determine the size and composition of the thyroid gland? A Thyroid scan with RAI 123I B Electrocardiography C Ultrasonography D Venous duplex Doppler study

C Question 29 Explanation: Although thyroid scans frequently are done to evaluate the thyroid gland, I 123 is used to destroy overactive thyroid cells such as are seen in thyroid cancer. Ultrasonography can be used early in the evaluation process to rule out Graves' disease, nodular goiter, or other thyroid dysfunction.

Nurse Gil is caring for a patient with a diagnosis of hypothyroidism. Which nursing diagnosis should the nurse most seriously consider when analyzing the needs of the patient? A High risk for aspiration related to severe vomiting B Diarrhea related to increased peristalsis C Hypothermia related to slowed metabolic rate D Oral mucous membrane, altered related to disease process

C Question 30 Explanation: Thyroid hormone deficiency results in reduction in the metabolic rate, resulting in hypothermia, and does predispose the older adult to a host of other health-related issues. One-quarter of affected elderly experience constipation.

Serge who has diabetes mellitus is taking oral agents, and is scheduled for a diagnostic test that requires him to be NPO. What is the best plan of the nurse with regard to giving the client his oral medications? A Administer the oral agents immediately after the test. B Notify the the diagnostic department and request orders. C Notify the physician and request orders. D Administer the oral agents with a sip of water before the test.

C Question 8 Explanation: It is best to notify the client's physician and request orders. The client should not receive the medication during NPO status unless directed by the physician. The medications should not be given upon return unless the physician orders this; the client may still need to be NPO. The radiologist in the diagnostic department might give orders, but it would be best to check with the client's physician first.

Marlisa has been diagnosed with diabetes mellitus type 1. She asks Nurse Errol what this means. What is the best response by the nurse? Select all that apply. A "Your alpha cells should be able to secrete insulin, but cannot." B "The exocrine function of your pancreas is to secrete insulin." C "Without insulin, you will develop ketoacidosis (DKA)." D "The endocrine function of your pancreas is to secrete insulin." E "It means your pancreas cannot secrete insulin." Question 2

C, D, E: Question 1 Explanation: One function of your pancreas is to secrete insulin. The endocrine function of the pancreas is to secrete insulin. The endocrine, not the exocrine, function of the pancreas is to secrete insulin. Insulin is secreted by the beta, not the alpha, cells of the pancreas. A consequence of diabetes mellitus type 1 is that without insulin, severe metabolic disturbances, such as ketoacidosis (DKA) will result.

The nurse is conducting a health assessment interview with a client diagnosed with human immune deficiency virus (HIV). Which statement by the client does the nurse immediately address? A. "When I injected heroin, I was exposed to HIV." B."I don't understand how the antiretroviral drugs work." C. "I remember to take my antiretroviral drugs almost every day." D. "My sex drive is weaker than it used to be since I started taking my antiretroviral medications."

C. "I remember to take my antiretroviral drugs almost every day." Rationale: Because inconsistent use of antiretroviral medications can lead to unsuccessful therapy and the development of drug-resistant HIV strains, it is important that clients take these drugs consistently. The nurse should immediately assess the reasons why the client does not take the medications daily and then should implement a plan to improve adherence. The nurse should assess whether the client is still injecting drugs and should make certain the client understands the risks for infection with another strain of HIV or other bloodborne pathogens and the risk for spreading HIV, but this does not need to be addressed immediately. The nurse must provide further education about how the medications work and assess how the lack of knowledge or decreased libido influences compliance, but this does not need to be addressed immediately.

The client is being discharged from the ambulatory care unit following cataract removal. The nurse provides instructions regarding home care. Which of the following. if stated by the client. indicates an understanding of the instructions? A. "I will take Aspirin if I have any discomfort." B. "I will sleep on the side that I was operated on." C. "I will wear my eye shield at night and my glasses during the day." D. "I will not lift anything if it weighs more that 10 pounds."

C. "I will wear my eye shield at night and my glasses during the day." The client is instructed to wear a metal or plastic shield to protect the eye from accidental and is instructed not to rub the eye. Glasses may be worn during the day. Aspirin or medications containing aspirin are not to be administered or taken by the client and the client is instructed to take acetaminophen as needed for pain. The client is instructed not to sleep on the side of the body on which the operation occurred. The client is not to lift more than 5 pounds.

A diabetic patient has been prescribed an alpha-1 glucosidase inhibitor. When teaching the patient about the medication, which of the following information will the healthcare provider include? Please choose from one of the following options. "Call us immediately if you experience tremors or palpitations." "Take one tablet daily first thing in the morning." "Take this medication with the first bite of each meal." "You should select foods low in protein when taking this medication." Scratchpad

C. "Take this medication with the first bite of each meal." Rationale: 1 / 3 Glucosidase is a hormone that breaks down glucose so it can be absorbed. Hint #22 / 3 Glucosidase inhibitors act in the gut and have a rapid onset. Hint #33 / 3 The patient should be instructed to take the medication with the first bite of food at each main meal. Glucosidase inhibitors do not cause hypoglycemia on their own.

The client tells the nurse that the doctor told him his antibiotic did not kill his infection but just slowed its growth. The client is anxious. What is the best response by the nurse to decrease the client's anxiety? Select one: A. "This is okay because your infection is not really that serious." B. "This is okay because your blood work is being monitored daily." C. "This is okay because your body will help kill the infection too." D. "This is okay because your doctor is an infectious disease specialist."

C. "This is okay because your body will help kill the infection too."

The nurse is instructing an unlicensed health care worker on the care of a client with human immune deficiency virus (HIV) who also has active genital herpes. Which statement by the health care worker indicates effective teaching of Standard Precautions? A "I need to know my HIV status, so I must get tested before caring for any clients." B "Putting on a gown and gloves will cover up the itchy sores on my elbows." C. "Washing my hands and putting on a gown and gloves is what I must do before starting care." D. "I will wash my hands before going into the room, and then will put on a gown and gloves only for direct contact with the client's genitals."

C. "Washing my hands and putting on a gown and gloves is what I must do before starting care." Rationale: Standard Precautions include whatever personal protective equipment (PPE) is necessary for the prevention of transmission of HIV and genital herpes. Knowing HIV status is important for preventing transmission of HIV, but is not a Standard Precaution. Health care workers with weeping dermatitis should not provide direct client care regardless of the use of a gown and gloves. Unlicensed health care workers cannot make the determination of what is required for PPE or Standard Precautions.

A male client with primary diabetes insipidus is ready for discharge on desmopressin (DDAVP). Which instruction should nurse Lina provide? A. "Administer desmopressin while the suspension is cold." B. "Your condition isn't chronic, so you won't need to wear a medical identification bracelet." C. "You may not be able to use desmopressin nasally if you have nasal discharge or blockage." D. "You won't need to monitor your fluid intake and output after you start taking desmopressin."

C. "You may not be able to use desmopressin nasally if you have nasal discharge or blockage." Desmopressin may not be absorbed if the intranasal route is compromised. Although diabetes insipidus is treatable, the client should wear medical identification and carry medication at all times to alert medical personnel in an emergency and ensure proper treatment. The client must continue to monitor fluid intake and output and receive adequate fluid replacement.

Nurse Perry is caring for a female client with type 1 diabetes mellitus who exhibits confusion, light-headedness, and aberrant behavior. The client is still conscious. The nurse should first administer: A. I.M. or subcutaneous glucagon. B. I.V. bolus of dextrose 50%. C. 15 to 20 g of a fast-acting carbohydrate such as orange juice. D. 10 U of fast-acting insulin

C. 15 to 20 g of a fast-acting carbohydrate such as orange juice. This client is having a hypoglycemic episode. Because the client is conscious, the nurse should first administer a fast-acting carbohydrate, such as orange juice, hard candy, or honey. If the client has lost consciousness, the nurse should administer either I.M. or subcutaneous glucagon or an I.V. bolus of dextrose 50%. The nurse shouldn't administer insulin to a client who's hypoglycemic; this action will further compromise the client's condition.

A bedtime snack is provided for Albert. This is based on the knowledge that intermediate-acting insulins are effective for an approximate duration of: A. 6-8 hours B. 10-14 hours C. 16-20 hours D. 24-28 hours

C. 16-20 hours

Rotation sites for insulin injection should be separated from one another by 2.5 cm (1 inch) and should be used only every: A. Third day B. Week C. 2-3 weeks D. 2-4 weeks

C. 2-3 weeks

The nurse expects that a type 1 diabetic may receive ____ of his or her morning dose of insulin preoperatively: A. 10-20% B. 25-40% C. 50-60% D. 85-90%

C. 50-60%

In preparation for cataract surgery. the nurse is to administer prescribed eye drops. The nurse reviews the physicians orders. expecting which type of eye drops to be instilled? A. An osmotic diuretic B. A miotic agent C. A mydriatic medication D. A thiazide diuretic

C. A mydriatic medication A mydriatic medication produces mydriasis or dilation of the pupil. Mydriatic medications are used preoperatively in the cataract client. These medication act by dilating the pupils. They also constrict blood vessels. An osmotic diuretic may be used to decrease intraocular pressure. A miotic medication constricts the pupil. A thiazide diuretic is not likely to be prescribed for a client with a cataract.

The nurse is performing an admission assessment on a client with a diagnosis of detached retina. Which of the following is associated with this eye disorder? A. Pain in the affected eye B. Total loss of vision C. A sense of a curtain falling across the field of vision D. A yellow discoloration of the sclera.

C. A sense of a curtain falling across the field of vision A characteristic manifestation of retinal detachment described by the client is the feeling that a shadow or curtain is falling across the field of vision. No pain is associated with detachment of the retina. Options B and D are not characteristics of this disorder. A retinal detachment is an ophthalmic emergency and even more so if visual acuity is still normal

The nurse plans to assess a client with type I hypersensitivity for which clinical manifestation? A. Poison ivy B.Autoimmune hemolytic anemia C. Allergic asthma D. Rheumatoid arthritis

C. Allergic asthma Rationale: Allergic asthma is a manifestation of type I hypersensitivity. Poison ivy is a type IV delayed mechanism of hypersensitivity. Autoimmune hemolytic anemia is a type II cytotoxic mechanism of hypersensitivity. Rheumatoid arthritis is a type III immune complex-mediated mechanism of hypersensitivity. Awarded 0.0 point

The client sustains a contusion of the eyeball following a traumatic injury with a blunt object. Which intervention is initiated immediately? A. Notify the physician B. Irrigate the eye with cold water C. Apply ice to the affected eye D. Accompany the client to the emergency room

C. Apply ice to the affected eye Treatment for contusion begins at the time of injury. Ice is applied immediately. The client then should be seen by a physician and receive a thorough eye examination to rule out the presence of other eye injuries.

Assessment findings reveal that a client admitted to the hospital has a contact type I hypersensitivity to latex. Which preventive nursing intervention is best in planning care for this client? A. Report the need for desensitization therapy. B. Convey the need for pharmacologic therapy to the health care provider. C. Communicate the need for avoidance therapy to the health care team. D. Discuss symptomatic therapy with the health care provider.

C. Communicate the need for avoidance therapy to the health care team. Rationale: Contact hypersensitivities can occur with latex, pollens, foods, and environmental proteins. Avoidance therapy is the recommended nursing intervention. Desensitization therapy is administered via allergy shots when allergens have been identified and cannot easily be avoided. Medications might be indicated if signs of type I or type IV hypersensitivity exist, but this is not a preventive measure. Symptomatic therapy interventions such as an epinephrine pen, antihistamines, and corticosteroids are effective only after the hypersensitivity reaction has already occurred.

Nurse Ronn is assessing a client with possible Cushing's syndrome. In a client with Cushing's syndrome, the nurse would expect to find: A. Hypotension. B. Thick, coarse skin. C. Deposits of adipose tissue in the trunk and dorsocervical area. D. Weight gain in arms and legs

C. Deposits of adipose tissue in the trunk and dorsocervical area. Because of changes in fat distribution, adipose tissue accumulates in the trunk, face (moonface), and dorsocervical areas (buffalo hump). Hypertension is caused by fluid retention. Skin becomes thin and bruises easily because of a loss of collagen. Muscle wasting causes muscle atrophy and thin extremities.

A client who is receiving an intravenous antibiotic begins to cough and states, "My throat feels like it is swelling." Which action does the nurse take next? A. Infuse normal saline at 200 mL/hr. B. Administer epinephrine (Adrenalin) 1:1000, 0.3 mL subcutaneously. C. Discontinue infusing the antibiotic. D. Give diphenhydramine (Benadryl) 100 mg IV

C. Discontinue infusing the antibiotic. Rationale: Because the antibiotic is the most likely cause of the client's apparent anaphylactic reaction, the nurse's first action should be to discontinue the antibiotic. The nurse must first assess the client, and although infusing normal saline and administering epinephrine and diphenhydramine may be indicated, these are not the nurse's first action.

The nurse is developing a plan of care for the client scheduled for cataract surgery. The nurse documents which more appropriate nursing diagnosis in the plan of care? A. Self-care deficit B. Imbalanced nutrition C. Disturbed sensory perception D. Anxiety

C. Disturbed sensory perception The most appropriate nursing diagnosis for the client scheduled for cataract surgery is Disturbed sensory perception (visual) related to lens extraction and replacement. Although the other options identify nursing diagnoses that may be appropriate. they are not related specifically to cataract surgery.

When instructing the female client diagnosed with hyperparathyroidism about diet, nurse Gina should stress the importance of which of the following? A. Restricting fluids B. Restricting sodium C. Forcing fluids D. Restricting potassium

C. Forcing fluids The client should be encouraged to force fluids to prevent renal calculi formation. Sodium should be encouraged to replace losses in urine. Restricting potassium isn't necessary in hyperparathyroidism.

When teaching the client about Meniere's disease. which of the following instructions would a nurse give about vertigo? A. Report dizziness at once B. Drive in daylight hours only C. Get up slowly. turning the entire body D. Change your position using the logroll method

C. Get up slowly. turning the entire body Turning the entire body. not the head. will prevent vertigo. Dizziness is expected but can be prevented. The client shouldn't drive as he may reflexively turn the wheel to correct vertigo. Turning the client in bed slowly and smoothly will be helpful; logrolling isn't needed.

A male client with type 1 diabetes mellitus asks the nurse about taking an oral antidiabetic agent. Nurse Jack explains that these medications are only effective if the client: A. Prefers to take insulin orally.has type 2 diabetes. B. Has type 2 diabetes C. Has type 1 diabetes. D. Is pregnant and has type 2 diabetes

C. Has type 1 diabetes. Oral antidiabetic agents are only effective in adult clients with type 2 diabetes. Oral antidiabetic agents aren't effective in type 1 diabetes. Pregnant and lactating women aren't prescribed oral antidiabetic agents because the effect on the fetus is uncertain.

A drug that is effective against a large number of different species of bacteria is said to: A. Be bacteriocidal B. Be bacteriostatic C. Have a wide spectrum of activity D. Have a narrow spectrum of activity

C. Have a wide spectrum of activity

A nurse is monitoring a client who has syndrome of inappropriate antidiuretic secretion (SIADH). Which of the following findings should the nurse expect? A. Polyuria B. Dehydration C. Hyponatremia D. Hyperthermia

C. Hyponatremia The client who has SIADH will have hyponatremia caused by the excessive release of an antidiuretic hormone (ADH). As a result of excessive ADH, the client retains water that causes dilutional hyponatremia.

The nurse understands that which of the following drugs falls under the classification of biguanides? 1. Metformin HCI (Glucophage) 2. Repaglinide (Prandin) 3. Tolbutamide (Orinase) 4. Acarbose (Precose)

Correct Answer: 1 Rationale 1: Metformin HCI is the only drug that falls within the classification of biguanides. Rationale 2: Repaglinide falls within the classification of meglitinides. Rationale 3: Tolbutamide falls within the classification of sulfonylureas. Rationale 4: Acarbose falls within the classification of alpha-glucosidase inhibitors.

During a health assessment, a 22-year-old college student tells the nurse that she is sexually active and protects herself from HIV and other sexually transmitted diseases (STDs) by using oral contraceptives. What is the nurse's best action? A. Remind the student that only abstinence prevents STDs. B. Ask the health care provider to order an HIV test for this student. C. Inform the student that oral contraceptives protect against pregnancy but not against any STD. D. Reinforce the student's preferred use of oral contraceptives and refrain from commenting on her sexual activity.

C. Inform the student that oral contraceptives protect against pregnancy but not against any STD. Rationale: Oral contraceptives only change the hormonal environment to prevent conception and have no activity against any microorganism. Sexual encounters that result in semen deposition into the vagina can result in HIV or other pathogenic microorganisms coming into contact with vaginal mucous membranes, penetrating those membranes, and infecting the woman. The nurse has an obligation to ensure the student understands this concept. The nurse cannot request an HIV test unless the student asks for one. Although reminding the student that only abstinence prevents STDs and reinforcing the student's preferred use of oral contraceptives and refraining from commenting on her sexual activity are reasonable, they are not the best response in this situation.

A client who is human immune deficiency virus (HIV) positive and has a CD4+ count of 15 has just been admitted with a fever and abdominal pain. Which health care provider request does the nurse implement first? A. Obtain a 12-lead electrocardiogram (ECG). B.Call for a portable chest x-ray. C. Obtain blood cultures from two sites. D. Give cefazolin (Kefzol) 500 mg IV.

C. Obtain blood cultures from two sites. Rationale: Antibiotics should be given as soon as possible to immunocompromised clients, but blood cultures must be obtained first so that culture results will not be affected by the antibiotic. A 12-lead ECG can be obtained and calling for a portable chest x-ray can be done after other priority requests have been carried out.

The client arrives in the emergency room with a penetrating eye injury from wood chips while cutting wood. The nurse assesses the eye and notes a piece of wood protruding from the eye. what is the initial nursing action? A. Remove the piece of wood using a sterile eye clamp B. Apply an eye patch C. Perform visual acuity tests D. Irrigate the eye with sterile saline

C. Perform visual acuity tests If the laceration is the result of a penetrating injury. an object may be noted protruding from the eye. This object must never be removed except by the ophthalmologist because it may be holding ocular structures in place. Application of an eye patch or irrigation of the eye may disrupt the foreign body and cause further tearing of the sclera. (The only option that will prevent further disruption is to assess visual acuity.)

A nurse is caring for a client who has diabetes insipidus For which of the following findings should the nurse monitor? A. Proteinuria B. Oliguria C. Polyuria D. Glycosuria

C. Polyuria Diabetes insipidus is characterized by increased thirst (polydipsia) and increased urine (polyuria) The client who has diabetes insipidus will excrete large quantities of urine with very low specific gravity.

Treatment of tuberculosis usually involves 1. the use of two or more drugs at the same time. 2. surgical removal of tubercular lesions. 3. keeping the client hospitalized. 4. the use of a single drug.

Correct Answer: 1 Rationale 1: Multidrug therapy for 6-12 months is the usual pharmacotherapy for tuberculosis. Rationale 2: Surgery is not the treatment. Rationale 3: It is not necessary to keep the client in the hospital. Rationale 4: Use of a single drug is not usual.

In planning care for a client with an acquired secondary immune deficiency with Candida albicans, which problem has the highest priority? A. Loss of social contact related to misunderstanding of transmission of acquired secondary immune deficiency and the social stigma B. Mouth sores related to Candida albicans secondary to acquired secondary immune deficiency C. Potential for infection transmission related to recurring opportunistic infections D. High risk for inadequate nutrition related to acquired secondary immune deficiency and Candida albicans

C. Potential for infection transmission related to recurring opportunistic infections Rationale: Protecting the client from further opportunistic infection such as Candida albicans is a priority. Loss of social contact is not a priority problem with an opportunistic infection. Mouth sores would be the secondary concern because Candida albicans causes the mouth sores. Nutrition will be affected because of Candida albicans; however, it is not a priority.

The blood glucose of a patient who is newly diagnosed with type 1 diabetes mellitus has a blood glucose level of 340 mg/dL. Which type of insulin prescribed for the patient is appropriate to administer at this time? Please choose from one of the following options. NPH + regular (70/30) NPH Regular Glargine

C. Regular Rationale: 1 / 3 The healthcare provider will want to correct the patient's hyperglycemia quickly. Hint #22 / 3 Eliminate the insulins that have a slow onset. Hint #33 / 3 Regular insulin has the fastest onset (30 - 60 min) in this group of insulins.

A client is being discharged from the hospital after an allergic reaction to environmental airborne allergens. Which instruction is most important for the nurse to include in this client's discharge teaching plan? A.Wash fruits and vegetables with mild soap and water before eating. B. Intermittent exposure to known allergens will produce immunity. C. Remove cloth drapes, carpeting, and upholstered furniture. D. Be cautious when eating unprocessed honey.

C. Remove cloth drapes, carpeting, and upholstered furniture. Rationale: Removing cloth drapes, carpet, and upholstery will reduce airborne pollen, dust mites, and mold. Washing fruits and vegetables pertains to food allergies. Clients do not develop immunity to known allergens by direct exposure; common interventions include avoidance therapy, desensitization therapy, and symptomatic therapy. Honey is said to help some people with allergies to pollen only; it does not have an impact on airborne allergens.

The physician orders cefepime (Maxipime) for a client. What is a priority question for the nurse to ask the client prior to administration of this drug? 1. "Are you breastfeeding?" 2. "Are you pregnant?" 3. "Are you allergic to penicillin?" 4. "Are you allergic to tetracycline?"

Correct Answer: 3 Rationale 1: Cefepime (Maxipime) is a Pregnancy Category B drug and is safe to use while breastfeeding. Rationale 2: Cefepime (Maxipime) is a Pregnancy Category B drug and is safe for use during pregnancy. Rationale 3: Cephalosporins are contraindicated in clients who have experienced a severe allergic reaction to penicillin. Rationale 4: Cephalosporins are not contraindicated in clients who have experienced an allergic reaction to tetracycline.

A female client with hypothyroidism (myxedema) is receiving levothyroxine (Synthroid), 25 mcg P.O. daily. Which finding should nurse Hans recognize as an adverse drug effect? A. Dysuria B. Leg cramps C. Tachycardia D. Blurred vision

C. Tachycardia Levothyroxine, a synthetic thyroid hormone, is given to a client with hypothyroidism to simulate the effects of thyroxine. Adverse effects of this agent include tachycardia. The other options aren't associated with levothyroxine.

Which sign or symptom is most typical of an untreated client with type 1 diabetes? 1. Increased energy 2. Weight gain 3. Fatigue 4. Decreased hunger

Correct Answer: 3 Rationale 1: Clients with type 1 DM do not experience increased energy; a typical sign/symptom is fatigue. Rationale 2: Clients with type 1 DM typically experience weight loss as opposed to weight gain. Rationale 3: Fatigue is a typical sign/symptom of type 1 DM due to sustained hyperglycemia. Rationale 4: Clients with type 1 DM typically experience polyphagiaincreased hungeras opposed to decreased hunger.

A nurse is providing teaching to a client who has Addison's disease about healthy snack foods. Which of the following food choices by the client indicates an understanding of the teaching? A. Sliced bananas B. Baked potato C. Turkey and cheese sandwich D. Plain yogurt with peaches

C. Turkey and cheese sandwich A turkey and cheese sandwich is high in protein, carbs, and sodium. The client who has addison's disease requires a diet low in potassium and high in sodium, carbs and proteins.

When teaching a patient about carbidopa-levodopa (Sinemet), what information should the nurse include about this medication? Carbidopa crosses the blood-brain barrier to increase the metabolism of levodopa to dopamine in the brain. Giving both drugs together minimizes adverse effects and decreases the risk of kidney or liver disease. Carbidopa increases levodopa's conversion in the periphery, enhancing the amount of dopamine available to the brain. Carbidopa decreases levodopa's conversion in the periphery, increasing the levodopa available to cross the blood-brain barrier.

Carbidopa decreases levodopa's conversion in the periphery, increasing the levodopa available to cross the blood-brain barrier. Adding carbidopa to levodopa inhibits the breakdown of levodopa in the periphery, increasing the amount available to cross the blood-brain barrier and decreasing the extrapyramidal adverse effects caused by dopamine in the periphery, resulting in fewer unwanted adverse effects.

The client receives hydrocortisone therapy. The nurse will primarily assess for which electrolyte disturbance? 1. Hypernatremia and hyperglycemia 2. Hypernatremia and hyperkalemia 3. Hypocalecmia and hyperkalemia 4. Hypoglycemia and hyponatremia

Correct Answer: 1 Rationale 1: Hypernatremia and hyperglycemia are seen due to the aldosterone effects (mineralcorticoid activity) causing sodium and fluid retention, and elevations of blood glucose due to promotion of gluconeogenesis. Rationale 2: Hypernatremia would be seen, but hypokalemia would be seen, not hyperkalemia. Rationale 3: Hypercalcemia and hypokalemia would be seen with this therapy. Rationale 4: Hypoglycemia and hyponatremia would not be seen with hydrocortisone therapy.

The nurse teaches the client about glucocorticoid therapy. The nurse evaluates that additional teaching is required when the client makes which statement? 1. I can take the medication at any time as long as I don't forget it. 2. I will monitor my blood sugar on a regular basis. 3. I will eat a diet that is high in protein. 4. I should take my medication after I have eaten.

Correct Answer: 1 Rationale 1: The medication must be taken at the same time of day to maintain serum levels. Rationale 2: It is important for the client to monitor blood glucose levels with glucocorticoid medications. Rationale 3: A high-protein diet is necessary with glucocorticoid medications. Rationale 4: Glucocorticoid medications should be taken after eating.

Vasopressin is used in the treatment of 1. diabetes insipidus. 2. dehydration. 3. electrolyte imbalances. 4. diabetes mellitus.

Correct Answer: 1 Rationale 1: Vasopressin and lypressin are used to treat diabetes insipidus. Rationale 2: Vasopressin is not used to treat dehydration. Rationale 3: Vasopressin is not used to treat electrolyte imbalances. Rationale 4: Vasopressin is not used to treat diabetes mellitus.

The client receives gentamicin (Garamycin) intravenously (IV) in the clinical setting. What is a priority nursing action? 1. Monitor the client for hearing loss. 2. Draw daily blood chemistries. 3. Decrease the fluids for the client during therapy. 4. Place the client on isolation precautions.

Correct Answer: 1 Rationale 1: Aminoglycosides are ototoxic drugs, and the client should be monitored for hearing loss. Rationale 2: Serum levels of the drug are indicated, but not blood chemistries. Rationale 3: Decreasing fluids during therapy is not indicated. Rationale 4: Isolation is determined by the causative organism, not the drug used for treatment.

The client with acquired immune deficiency syndrome (AIDS) asks the nurse why he must take so many medications. What is the best response by the nurse? 1. "To decrease the possibility of the virus developing resistance to the medications." 2. "Research has shown single medications to be ineffective." 3. "Because the earlier we start multiple medications, the better for you." 4. "To provide you with the most effective treatment for your illness."

Correct Answer: 1 Rationale 1: Decreasing the possibility of resistance is the key; single drugs can be effective, but viral resistance is a problem. Rationale 2: Multiple medications will provide the best treatment, but this is not as good an answer as avoiding drug resistance. Rationale 3: It is debatable if multiple drugs should be used early in the course of the disease. Rationale 4: Single medications can be effective, but drug resistance is more likely the reason.

The client is prescribed amoxicillin (Amoxil) for 10 days to treat strep throat. After 5 days, the client tells the nurse he plans to stop the medication because he feels better. What is the best response by the nurse? 1. "If you stop the medicine early, you have not effectively killed out the bacteria making you sick." 2. "You should get another throat culture if your symptoms return." 3. "If you stop the medicine early, this could result in resistance to the antibiotic." 4. "You should get another throat culture to see if the infection is gone."

Correct Answer: 1 Rationale 1: If all the medication is not taken, there is a strong possibility that not all bacteria have been eliminated. Rationale 2: Another throat culture is inappropriate; the client must finish the medication. Rationale 3: Stopping the medicine early can result in resistance to the antibiotic, but the client may not care about this unless he can see how it directly affects him. Rationale 4: Another throat culture is inappropriate; the client must finish the medication.

The client has type 1 diabetes mellitus and receives insulin. Which laboratory test will the nurse assess? 1. Potassium 2. Serum amylase 3. AST (aspartate aminotransferase) 4. Sodium

Correct Answer: 1 Rationale 1: Insulin causes potassium to move into the cell and may cause hypokalemia. Rationale 2: There is no need to monitor the serum amylase level. Rationale 3: There is no need to monitor the AST (aspartate aminotransferase) level. Rationale 4: There is no need to monitor the sodium level.

The physician writes orders for the client with diabetes mellitus. Which order would the nurse validate with the physician? 1. Lantus insulin 20U BID 2. Administering regular insulin 30 minutes prior to meals 3. 5 units of Humalog/10 units NPH daily 4. Metformin (Glucophage) 1000 mg per day in divided doses

Correct Answer: 1 Rationale 1: Lantus insulin is usually prescribed in once-a-day dosing so an order for BID dosing should be validated with the physician. Rationale 2: Regular insulin is administered 30 minutes before meals. Rationale 3: Humalog and NPH insulin can be mixed. Rationale 4: Metformin (Glucophage) is often prescribed in divided doses of 1000 mg per day.

The drug that would most likely be used in the treatment of tuberculosis is 1. Erythromycin (E-mycin). 2. Vancomycin (Vancocin). 3. Isoniazid (INH). 4. Gentamicin (Garamycin).

Correct Answer: 3 Rationale 1: Erythromycin is most effective against gram-positive bacteria. Rationale 2: Vancomycin is used for bactericidal reasons. Rationale 3: Isoniazid (INH) is the drug of choice for anti-tuberculosis therapy. Rationale 4: Gentamicin is used for bactericidal reasons.

The nurse teaches clients with acquired immune deficiency syndrome (AIDS) about the importance of taking their medications as prescribed. What does the nurse recognize as the primary factor for medication noncompliance in the acquired immune deficiency syndrome (AIDS) population? 1. The necessity of having to take multiple medications throughout the day 2. A lack of understanding for the reason to take the medications 3. The weight gain associated with the medications 4. The difficulty with availability of medications to treat acquired immune deficiency syndrome (AIDS)

Correct Answer: 1 Rationale 1: Multiple medications are required throughout the day. The higher the number of medications taken daily equates with a higher noncompliance rate. Rationale 2: A lack of understanding could be a factor, but it is the schedule of taking multiple drugs that leads to noncompliance. Rationale 3: Some medications may cause weight gain, but this is not a primary reason for noncompliance. Rationale 4: Medications are generally available for clients with acquired immune deficiency syndrome (AIDS) today.

Pathogenicity is different than virulence in that pathogenicity can 1. lead to the ability of organisms to cause infection. 2. kill pathogens. 3. cause a disease when pathogens are present. 4. disrupt cell lining.

Correct Answer: 1 Rationale 1: The ability of an organism to cause infection is its pathogenicity. Rationale 2: Medications that can kill bacteria are called bacteriocidal. Rationale 3: A highly virulent microbe is one that can produce disease when present in minute numbers. Rationale 4: Cell lining is not disrupted in this process.

The nurse has finished teaching a client with diabetes mellitus how to administer insulin. The nurse evaluates that learning has occurred when the client makes which statement? 1. I should only use a calibrated insulin syringe for the injections. 2. I should check my blood sugar immediately prior to the administration. 3. I should use the abdominal area only for insulin injections. 4. I should provide direct pressure over the site following the injection

Correct Answer: 1 Rationale 1: To ensure the correct insulin dose, a calibrated insulin syringe must be used. Rationale 2: There is no need to check blood glucose immediately prior to the injection. Rationale 3: Insulin injections should also be rotated to the arm and thigh, not just the abdominal area. Rationale 4: There is no need to apply direct pressure over the site following an insulin injection.

The client has MRSA and receives vancomycin (Vancocin) intravenously (IV). The nurse assesses an upper body rash and decreased urine output. What is the nurse's priority action? 1. Hold the next dose of vancomycin (Vancocin) and notify the physician. 2. Obtain a stat X-ray and notify the physician. 3. Administer an antihistamine and notify the physician. 4. Obtain a sterile urine specimen and notify the physician.

Correct Answer: 1 Rationale 1: Upper body rash and decreased urine output are most likely symptoms of vancomycin (Vancocin) toxicity, so the medication should be held and the physician notified. Rationale 2: There is no reason to obtain a chest x-ray. Rationale 3: The nurse should collaborate with the physician regarding medications for treatment of this situation. Rationale 4: The client's symptoms are most likely not due to a urinary tract infection, so a sterile urine specimen is not indicated.

The nurse makes a home visit to a client with diabetes mellitus. During the visit, the nurse notes that the clients 3-month supply of insulin vials that were delivered a week ago are not refrigerated. What is the best action by the nurse at this time? 1. Have the client place the insulin vials in the refrigerator. 2. Have the client discard the vials. 3. Instruct the client to label each vial with the date when opened. 4. Tell the client this is too much insulin to have on hand.

Correct Answer: 1 Rationale 1: Vials can be stored at room temperature up to one month. For longer storage, they should be refrigerated. Rationale 2: There is no need to discard the vials. Rationale 3: Writing the date of opening on the vial is good practice, but does not address the need to refrigerate additional vials. Rationale 4: There is no indication that this is too much insulin to have on hand.

The nurse plans to teach a client with human immunodeficiency virus (HIV) infection about zidovudine (Retrovir). What will the best plan by the nurse include? Select all that apply. 1. Zidovudine (Retrovir) will need to be stopped if bone marrow depression occurs. 2. Zidovudine (Retrovir) will slow the disease, but not cure it. 3. Zidovudine (Retrovir) prevents spread of the virus through sexual contact. 4. Zidovudine (Retrovir) is only administered to male patients. 5. Zidovudine (Retrovir) therapy frequently results in the development of anemia.

Correct Answer: 1,2, ,5 Rationale 1: Stopping zidovudine (Retrovir) if bone marrow depression occurs allows the bone marrow time to recover. Rationale 2: The drug only slows the disease; it will not cure it. Rationale 3: Zidovudine (Retrovir) will slow the progression of the disease; it will not prevent its transmission. Rationale 4: Zidovudine (Retrovir) can be given to patients of either gender. Rationale 5: Treatment with zidovudine (Retrovir) frequently results in anemia.

A client has been prescribed propylthiouracil (PTU). What medication education should the nurse provide? Select all that apply. 1. It may take several days or weeks for you to see effects of this drug. 2. Take this drug with meals. 3. Do not become pregnant while taking this medication. 4. You may experience a rash while taking this drug. 5. You may continue to breastfeed while taking this drug.

Correct Answer: 1,2,3,4 Rationale 1: The action of this drug may be delayed from several days to as long as 6-12 weeks. Rationale 2: This drug may cause gastric upset, which is decreased if the drug is taken with food. Rationale 3: This is a pregnancy class D drug. Rationale 4: Rash is one of the most frequent adverse effects. Rationale 5: This drug should not be taken while breastfeeding.

The nurse is providing community education regarding ways to reduce development of antibiotic resistance. Which information should be included? Select all that apply. 1. The best way to prevent antibiotic resistance is to prevent infections from occurring. 2. Do not expect to receive an antibiotic prescription for colds and influenza. 3. Take the full amount of any prescribed medication. 4. Go to the doctor as soon as you feel ill. 5. Use good infection control measures.

Correct Answer: 1,2,3,5 Rationale 1: It is much easier to prevent infection than it is to treat an infection. Rationale 2: It is not necessary to treat every respiratory illness with antibiotics. Most are viral and do not respond to antibiotics. Rationale 3: Stopping antibiotic therapy prematurely allows some pathogens to survive. The ones that survive are the strongest pathogens. Rationale 4: Not all illnesses require physician intervention. Rationale 5: Preventing transmission of illnesses is essential.

The nursing instructor teaches the student nurses about the endocrine system. The nursing instructor evaluates that learning has occurred when the student nurses make which statements? Select all that apply. 1. The hypothalamus secretes releasing hormones. 2. Hormones released by the endocrine system influence every organ in the body. 3. The hypothalamus is considered the master gland. 4. The pituitary gland secretes TSH (thyroid stimulating hormone). 5. The endocrine system is a major controller of homeostasis.

Correct Answer: 1,2,4,5 Rationale 1: The hypothalamus secretes releasing hormones. Rationale 2: Hormones released by the endocrine system influence every organ in the body. Rationale 3: The pituitary, not the hypothalamus, is often called the master gland; however, the pituitary and hypothalamus are best visualized as an integrated unit. Rationale 4: The pituitary gland secretes TSH (thyroid stimulating hormone). Rationale 5: The endocrine system is a major controller of homeostasis.

A client has been prescribed levothyroxine (Levothroid). What medication information should the nurse provide? Select all that apply. 1. It may take a few weeks for you to see the full benefits from this drug. 2. Be sure to keep all of your follow-up appointments. 3. Take this medication at whatever time you eat your evening meal. 4. Do not start a fiber laxative without first discussing it with your health care team. 5. Take your calcium supplement at least 4 hours after taking this drug.

Correct Answer: 1,2,4,5 Rationale 1: When given orally, it may take up to 3 weeks for the full effect of the drug to be realized. Rationale 2: Serum TSH levels will be drawn frequently as the client begins this therapy and to monitor its effectiveness as therapy continues. Rationale 3: The medication should be taken at the same time every day, not at variable meal times. It should be taken in the morning to decrease insomnia. Rationale 4: Dietary fiber may bind to and decrease the absorption of levothyroxine. Rationale 5: Calcium and iron supplements should be taken at least 4 hours after taking levothyroxine to prevent interference with drug absorption.

A patient has been prescribed ciprofloxacin (Cipro) for a severe sinus infection. The nurse evaluates that medication education has been effective when the patient makes which statements? Select all that apply. 1. "I should avoid milk while taking this medication." 2. "I should avoid coffee while taking this medication." 3. "If this medication upsets my stomach, I can take it with an antacid." 4. "I may have some diarrhea while taking this medication." 5. "If my stomach gets upset, I should take this medication with food."

Correct Answer: 1,2,4,5 Rationale 1: Dairy products can decrease the absorption of ciprofloxacin. Rationale 2: Ciprofloxacin can increase serum levels of caffeine. Rationale 3: Antacids can diminish drug absorption. Rationale 4: Diarrhea can occur while taking ciprofloxacin. Rationale 5: Ciprofloxacin may be administered with food to diminish adverse GI effects.

A nurse has provided education regarding type 2 diabetes to a newly diagnosed client. Which statements would the nurse interpret as indicating need for additional education? Note: Credit will be given only if all correct choices and no incorrect choices are selected. Standard Text: Select all that apply. 1. Well, at least the medications I will be on will help me lose weight. 2. I can take an oral medication and will never have to inject myself. 3. I can increase my bodys ability to use the insulin I make by exercising regularly. 4. I have several lifestyle changes to make. 5. I dont run the risk of blindness and kidney disease like type 1 diabetics.

Correct Answer: 1,2,5 Rationale 1: Some of the medications used for type 2 diabetes cause weight gain. Rationale 2: As type 2 diabetes progresses, the cells that produce insulin may fail, causing the need for insulin. Some medications especially designed to treat type 2 diabetes are injected. Rationale 3: The activity of insulin receptors can be increased by physical exercise. Rationale 4: Lifestyle changes can help the type 2 diabetic avoid complications. Rationale 5: If type 2 diabetes is poorly managed these complications can occur.

A client has been taking propylthiouracil (PTU) for several weeks. Which client statements would the nurse interpret as indicating that the drug is having its desired effects? Select all that apply. 1. I am sleeping so much better. 2. I can't seem to gain any weight no matter how much I eat. 3. I don't feel as anxious as I once did. 4. My heart doesn't seem to be racing anymore. 5. My skin in not bruising as much.

Correct Answer: 1,3,4 Rationale 1: A reduction in insomnia is a desired effect of this drug. Rationale 2: A desired effect of this drug is that the client will gain weight. Rationale 3: Reduction of anxiety is a desired effect of this drug. Rationale 4: A slower pulse rate is a desired effect of this drug. Rationale 5: Skin bruising is not an effect of hyperthyroidism, so this effect would not be expected.

A client, newly diagnosed with type 1 diabetes, says, I have heard this is a bad disease. What complications could I have? How should the nurse respond? Note: Credit will be given only if all correct choices and no incorrect choices are selected. Standard Text: Select all that apply. 1. Problems with arteries can occur that may cause such problems as heart disease, stroke, kidney disease, or blindness. 2. Lets not talk about that now, but rather focus on keeping you healthy. 3. You could have nerve problems that lead to numbness or tingling in your feet or hands. 4. One of the most serious complications is diabetic ketoacidosis. 5. You may experience inability to think and difficulty with memory.

Correct Answer: 1,3,4 Rationale 1: Arterial damage can lead to the problems listed. Rationale 2: The client is interested in this topic today and the topic should be addressed. Rationale 3: Neuropathy may occur, causing numbness, tingling, or loss of sensation in the limbs. Rationale 4: DKA is one of the most serious complications of type 1 diabetes. Rationale 5: While these symptoms may occur related to CVA, they are not primary complications of type 1 diabetes.

16-year-old patient is admitted to the emergency room after attempting to commit suicide by overdosing on the isoniazid (INH) prescribed for newly diagnosed tuberculosis. What information does the nurse provide to the family? Select all that apply. 1. INH overdose is very serious. 2. Large amounts of intravenous lipids will be administered. 3. Treatment will include infusion of vitamin B6. 4. Liver damage may occur. 5. Oral aspirin will be given as an antidote.

Correct Answer: 1,3,4 Rationale 1: INH is very serious and may be fatal. Rationale 2: There is no indication for large amounts of intravenous lipids. Rationale 3: Treatment includes infusion of vitamin B6. Rationale 4: INH may damage the liver, even in recommended dosages. Rationale 5: Oral aspirin is not the antidote for INH overdose.

The nurse educates clients with acquired immune deficiency syndrome (AIDS) about the nature of viruses. The nurse evaluates that learning has occurred when the clients make which responses? Select all that apply. 1. "Viruses are nonliving particles." 2. "The structure of viruses is complex." 3. "Viruses can infect plants as well as animals." 4. "A virion is a mature virus." 5. "Viruses are intracellular parasites."

Correct Answer: 1,3,4,5 Rationale 1: Viruses are nonliving agents that infect bacteria, plants, and animals. Rationale 2: The structure of viruses is quite primitive compared to the simplest cell. Rationale 3: Viruses can infect plants as well as animals. Rationale 4: A mature infective particle is called a virion. Rationale 5: Viruses must use intracellular machinery to replicate, so they are called intracellular parasites.

The client receives metformin (Glucophage). What will the best plan by the nurse include with regard to patient education with this drug? Note: Credit will be given only if all correct choices and no incorrect choices are selected. Standard Text: Select all that apply. 1. It decreases sugar production in the liver. 2. It inhibits absorption of carbohydrates. 3. It stimulates the pancreas to produce more insulin. 4. It reduces insulin resistance. 5. It increases energy use.

Correct Answer: 1,4 Rationale 1: Metformin (Glucophage) decreases sugar production (gluconeogenesis) in the liver. Rationale 2: Metformin (Glucophage) does not inhibit the absorption of carbohydrates. Rationale 3: Metformin (Glucophage) reduces insulin resistance. Rationale 4: Metformin (Glucophage) reduces insulin resistance. Rationale 5: Metformin (Glucophage) does not increase energy use.

The nurse plans to teach the client with acquired immune deficiency syndrome (AIDS) about bacterial infections. Which information should the nurse include in this teaching? Select all that apply. 1. "If just a few bacteria make you sick, this is virulence." 2. "Most bacteria have developed antibiotic resistance." 3. "Pathogens are divided into two classes, bacteria and viruses." 4. "Pathogenicity means the bacteria can cause an infection." 5. "Actually, most bacteria will not harm us."

Correct Answer: 1,4,5 Rationale 1: A highly virulent microbe is one that can produce disease when present in minute numbers. Rationale 2: Antibiotic resistance is a problem; however, only a few, not most, bacteria have developed it. Rationale 3: Human pathogens include viruses, bacteria, fungi, unicellular organisms, and multicellular animals. Rationale 4: The ability of an organism to cause infection is called pathogenicity. Rationale 5: Only a few dozen pathogens commonly cause disease in humans; most are harmless.

Which of the following is a sign or symptom of hypothyroidism? 1. Anxiety 2. Bradycardia 3. Tachycardia 4. Weight loss

Correct Answer: 2 Rationale 1: Anxiety is a symptom of hyperthyroidism. Rationale 2: Bradycardia can be a more severe symptom of hypothyroidism. Rationale 3: Tachycardia is symptom of hyperthyroidism. Rationale 4: Weight loss is a sign of hyperthyroidism.

The client has diabetes type 1 and receives insulin for glycemic control. The client tells the nurse that she likes to have a glass of wine with dinner. What will the best plan by the nurse for client education include? 1. The alcohol could cause pancreatic disease and decrease insulin production. 2. The alcohol could predispose you to hypoglycemia. 3. The alcohol could cause serious liver disease. 4. The alcohol could predispose you to hyperglycemia.

Correct Answer: 2 Rationale 1: Alcohol can cause pancreatic disease, but the clients pancreas is not producing any insulin currently. Rationale 2: Alcohol can potentiate hypoglycemic effects in the client. Rationale 3: Alcohol can cause liver disease, but the more immediate concern is hypoglycemia. Rationale 4: Alcohol can potentiate hypoglycemic, not hyperglycemic, effects in the client.

The client receives multiple antibiotics to treat a serious infection. What will the priority assessment of the client by the nurse include? 1. Assessing blood cultures for the presence of bacteria 2. Assessing changes in stool, white patches in the mouth, and urogenital itching or rash 3. Assessing renal and liver function tests 4. Assessing whether or not the client has adequate food and fluid intake

Correct Answer: 2 Rationale 1: Assessing blood cultures is important, but not as important as assessing for superinfections. Rationale 2: A superinfection occurs when microorganisms normally present in the body, host flora, are destroyed by antibiotic therapy. A superinfection can be lethal and should be suspected if a new infection appears while the client is receiving antibiotics. Signs of superinfection commonly include diarrhea, white patches in the mouth, urogenital itching, and presence of a blistering itchy rash. Rationale 3: Assessing renal and liver function tests is very important but not as important as assessing for superinfections. Rationale 4: Assessing food and fluid intake is very important but not as important as assessing for superinfections.

A client with diabetes mellitus type 1 is found unresponsive in the clinical setting. Which nursing action is a priority? 1. Call a code. 2. Treat the client for hypoglycemia. 3. Call the physician STAT. 4. Assess the clients vital signs.

Correct Answer: 2 Rationale 1: Assessment for ABCs should precede calling a code; there is no information that the client is not breathing. Rationale 2: When a client with diabetes mellitus type 1 is found unresponsive, the nurse should focus on and treat for hypoglycemia, as this is more likely than hyperglycemia. Rationale 3: .This is an emergency situation where the nurse must act before calling the physician. Rationale 4: Vital signs should be taken after the client is treated for hypoglycemia.

The client receives acyclovir (Zovirax) for treatment of genital herpes. What is a priority assessment by the nurse? 1. Auditory and visual hallucinations 2. Increased serum creatinine 3. Respiratory distress 4. Thrombocytopenia

Correct Answer: 2 Rationale 1: Auditory and visual hallucinations are not adverse effects of acyclovir (Zovirax). Rationale 2: Acyclovir (Zovirax) is nephrotoxic, so serum creatinine should be monitored. Rationale 3: Respiratory distress is not an adverse effect of acyclovir (Zovirax). Rationale 4: Bone marrow suppression is not an adverse effect of acyclovir (Zovirax).

The client receives efavirenz (Sustiva) as treatment for acquired immune deficiency syndrome (AIDS). The nurse assesses the client for which serious adverse effect? 1. Cardiac arrest 2. Rash 3. Bone marrow suppression 4. Seizures

Correct Answer: 2 Rationale 1: Cardiac arrest is not an expected adverse effect of efavirenz (Sustiva). Rationale 2: A serious adverse effect of efavirenz (Sustiva) is Stevens-Johnson rash, which can be life threatening. Rationale 3: Bone marrow suppression is not an adverse effect of efavirenz (Sustiva). Rationale 4: Seizures are not an adverse effect of efavirenz (Sustiva).

The client receives multiple drugs for treatment of tuberculosis. The nurse teaches the client the rationale for multiple drug treatment and evaluates learning as effective when the client makes which statement? 1. "Current research indicates that the most effective way to treat tuberculosis is with multiple drugs." 2. "Multiple drugs are necessary because the bacteria are likely to develop resistance to just one drug." 3. "Treatment for tuberculosis is complex, and multiple drugs must be continued for as long as I am contagious." 4. "Multiple drug treatment is necessary for me to be able to develop immunity to tuberculosis."

Correct Answer: 2 Rationale 1: Current research does support multiple drug treatment, but this does not explain the rationale for this to the client. Rationale 2: Tuberculosis bacilli are likely to develop resistance to one drug, so multiple drugs must be used. Rationale 3: Treatment must be continued long after the client is no longer contagious. Rationale 4: Clients cannot develop immunity to bacterial infections.

The mechanism of action of regular insulin is to 1. stimulate the pancreas to produce insulin. 2. promote entry of glucose into the cells. 3. promote the entry of glucose into the bloodstream. 4. stimulate the pancreas to secrete more insulin.

Correct Answer: 2 Rationale 1: Oral hypoglycemic drugs, such as glipizide, stimulate the pancreas to produce insulin. Rationale 2: The action of regular insulin is to promote entry of glucose into the cells, thereby lowering glucose. Rationale 3: Insulin would not promote glucose into the bloodstream. Rationale 4: Oral hypoglycemic drugs, such as glipizide, stimulate the pancreas to secrete insulin.

The client asks the nurse why there aren't better drugs for human immunodeficiency virus (HIV) infection when so much money is spent on research. What is the best response by the nurse? 1. "Antiviral drugs are almost impossible to create." 2. "Developing new drugs is difficult because the virus mutates so readily." 3. "Developing new drugs is difficult because people think acquired immune deficiency syndrome (AIDS) is a rare disease." 4. "Developing new drugs is difficult because we still do not understand the virus."

Correct Answer: 2 Rationale 1: Over 20 new antiretroviral drugs have been developed. Rationale 2: Antiviral pharmacotherapy can be extremely challenging because of the rapid mutation rate of viruses, which can quickly render drugs ineffective. Rationale 3: Most people recognize that human immunodeficiency virus (HIV) infection is not rare. Rationale 4: After more than 30 years of research, the virus is mostly understood.

The nurse teaches a class for the public about diabetes mellitus. Which individual does the nurse assess as being at highest risk for developing diabetes? 1. The 38-year-old client who smokes one pack of cigarettes per day 2. The 42-year-old client who is 50 pounds overweight 3. The 50-year-old client who does not get any physical exercise 4. The 56-year-old client who drinks three glasses of wine each evening

Correct Answer: 2 Rationale 1: Smoking is a serious health concern, but is not a specific risk factor for diabetes. Rationale 2: Obesity increases the likelihood of developing diabetes mellitus due to overstimulation of the endocrine system. Rationale 3: Exercise is important, but a lack of exercise is not as big of a risk factor as obesity. Rationale 4: Consuming alcohol is associated with liver disease, but is not as high a risk factor for diabetes as obesity.

The physician orders insulin lispro (Humalog), 10 units for the client. When will the nurse administer this medication? 1. Thirty minutes before meals 2. Five minutes before a meal 3. When the meal trays arrive on the floor 4. Fifteen minutes after meals

Correct Answer: 2 Rationale 1: The onset of action for insulin lispro (Humalog) is 10 to 15 minutes so it must be given when the client is eating, not 30 minutes before meals, to prevent hypoglycemia. Rationale 2: The onset of action for insulin lispro (Humalog) is 10 to 15 minutes so it must be given 5 to 10 minutes before the client eats. Rationale 3: The onset of action for insulin lispro (Humalog) is 10 to 15 minutes so it must be given just before the client eats, not when meal trays arrive on the floor, to prevent hypoglycemia. Rationale 4: The onset of action for insulin lispro (Humalog) is 10 to 15 minutes so it must be given when the client is eating, not 15 minutes after meals, to prevent hypoglycemia.

A client with diabetes mellitus is taking oral agents, and is scheduled for a diagnostic test that requires him to be NPO (nothing by mouth) and to have contrast dye. What is the best plan by the nurse with regard to giving the client his oral medications? 1. Notify the diagnostic department and request orders. 2. Notify the physician and request orders. 3. Administer the oral agents immediately after the test. 4. Administer the oral agents with a sip of water before the test.

Correct Answer: 2 Rationale 1: The radiologist in the diagnostic department might give orders, but it would be best to check with the clients physician first. Rationale 2: It is best to notify the clients physician and request orders. Rationale 3: Some oral medications should not be given for up to 2 days after receiving IV contrast. Rationale 4: The client should not receive the medication during NPO (nothing by mouth) status unless directed by the physician. Some oral medications should not be given for 2 days before receiving IV contrast.

The nurse works in infection control and teaches a class to staff nurses about the ways that resistance to antibiotics can occur. The nurse evaluates that learning has occurred when the nurses make which statement? 1. "Resistance to antibiotics most often occurs when physicians prescribe too many of them for elderly clients." 2. "Resistance to antibiotics can occur by the common use of them for nosocomial infections." 3. "Resistance to antibiotics most often occurs when physicians prescribe too many of them for children with ear infections." 4. "Resistance to antibiotics can occur by the prophylactic use of them for preoperative clients."

Correct Answer: 2 Rationale 1: The use of antibiotics by physicians with elderly clients is not the major cause of antibiotic resistance. Rationale 2: The organisms that cause nosocomial infections have most likely been treated with antibiotics and are the most likely organisms to develop resistance to antibiotics. Rationale 3: The use of antibiotics by physicians for children with ear infections is not the primary cause of antibiotic resistance. Rationale 4: The prophylactic use of antibiotics does not promote antibiotic resistance.

The nurse is aware that the major characteristic of viruses is that they are 1. extracellular parasites. 2. intracellular parasites. 3. extracellular bacteria. 4. intracellular bacteria.

Correct Answer: 2 Rationale 1: Viruses are not extracellular parasites. Rationale 2: Viruses are intracellular parasites. They must be inside a host cell to cause infection. Rationale 3: Viruses are not bacteria. Rationale 4: Viruses are not bacteria.

A client has been prescribed trimethoprim-sulfamethoxazole (Septra) for treatment of a urinary tract infection. Which comments, made by the client, would the nurse discuss with the prescriber before allowing the client to leave the clinic? Select all that apply. 1. "My husband and I plan to start a family as soon as possible." 2. "I forgot to take my potassium supplement today." 3. "Is it okay to take this with my warfarin?" 4. "It is so cloudy today." 5. "My 80-year-old mother is coming to visit today."

Correct Answer: 2,3 Rationale 1: Sulfa drugs are contraindicated for use by women at term. Rationale 2: Potassium supplements should not be taken during therapy unless directed by the health care provider. Rationale 3: Sulfa drugs may enhance the effects of oral anticoagulants. Rationale 4: Sulfa drugs can result in photosensitivity. Rationale 5: There is no reason this visit should be of concern regarding the medication prescribed.

A client who has diabetes mellitus is diagnosed with tuberculosis and has been prescribed multiple-drug therapy. What instruction should the nurse provide to this client? Select all that apply. 1. These medications can cause hypoglycemia, so you should always carry a sugar source. 2. Test your blood glucose more frequently while on these medications. 3. If your blood glucose levels elevate consistently, contact us. 4. Take your medication for diabetes at least 6 hours after taking these medications. 5. While on these medications, you will be more prone to infections in your feet.

Correct Answer: 2,3 Rationale 1: The medication regimen is more likely to cause hyperglycemia. Rationale 2: These medications may cause hyperglycemia. The client should monitor blood glucose levels more closely. Rationale 3: Constant elevation of blood glucose levels may warrant alteration in medication therapy for diabetes. Rationale 4: There is no reason to separate the administration of these medications by 6 hours. Rationale 5: There is no evidence that medications used to treat tuberculosis will increase the likelihood of infections in the feet.

The client is to receive an injection of penicillin G benzathine (Bicillin LA) in the outpatient clinic. What are the priority nursing actions prior to administering this injection? Select all that apply. 1. Have the client lie down and assess vital signs before she leaves. 2. Ask the client if she has ever had an allergy to penicillin before. 3. Inform the client that she will need to wait 30 minutes before leaving the clinic. 4. Inform the client that she must have someone drive her home. 5. Advise the client to rest for the remainder of the day.

Correct Answer: 2,3 Rationale 1: There is no reason to have the client lie down. It is not important to assess vital signs unless a problem has developed. Rationale 2: It is always important to ask about allergies. The nurse must be aware, however, that no history of allergy does not guarantee there will not be an allergic response with this administration. Rationale 3: It is important that the client be reassessed for development of allergic reaction before leaving the clinic. Rationale 4: There is no indication that the client will require someone else to drive her home. Rationale 5: There is no specific reason the client should rest related to administration of penicillin.

The client has been diagnosed with diabetes mellitus type 1. He asks the nurse what this means. What is the best response by the nurse? Note: Credit will be given only if all correct choices and no incorrect choices are selected. Standard Text: Select all that apply. 1. The exocrine function of your pancreas is to secrete insulin and it is not working. 2. Without insulin you will develop ketoacidosis (DKA). 3. The endocrine function of your pancreas is to secrete insulin, but it isnt working. 4. Your alpha cells should be able to secrete insulin, but cannot. 5. It means your pancreas cannot secrete insulin.

Correct Answer: 2,3,5 Rationale 1: The endocrine, not the exocrine, function of the pancreas is to secrete insulin. Rationale 2: A consequence of diabetes mellitus type 1 is that without insulin, severe metabolic disturbances, such as diabetic ketoacidosis (DKA) will result. Rationale 3: The endocrine function of the pancreas is to secrete insulin. Rationale 4: Insulin is secreted by the beta, not the alpha, cells of the pancreas. Rationale 5: One function of the pancreas is to secrete insulin.

A nurse is reviewing the blood work of a client who has recently begun treatment for type 2 diabetes. Which results would indicate that the client is on target with therapy? Note: Credit will be given only if all correct choices and no incorrect choices are selected. Standard Text: Select all that apply. 1. HBA1C level is 8.4%. 2. HBA1C level is 6.3%. 3. Fasting blood glucose is 130 g/dL. 4. Fasting blood glucose is 100 g/dL. 5. Fasting blood glucose is 68 g/dL.

Correct Answer: 2,4 Rationale 1: This level is too high. Rationale 2: Target level is 6.5% or less. Rationale 3: This is too high. Rationale 4: Goal is 110 g/dL. Rationale 5: A FBG of 68 g/dL is too low.

A client has been prescribed exenatide (Byetta). What medication education should the nurse provide? Note: Credit will be given only if all correct choices and no incorrect choices are selected. Standard Text: Select all that apply. 1. Drink an 8 ounce glass of water with this pill. 2. You should take this medication twice each day. 3. You may experience dryness of the mouth while taking this drug. 4. You may develop diarrhea while taking this drug. 5. This drug will help you secrete more insulin.A client has been prescribed exenatide (Byetta). What medication education should the nurse provide? Note: Credit will be given only if all correct choices and no incorrect choices are selected. Standard Text: Select all that apply. 1. Drink an 8 ounce glass of water with this pill. 2. You should take this medication twice each day. 3. You may experience dryness of the mouth while taking this drug. 4. You may develop diarrhea while taking this drug. 5. This drug will help you secrete more insulin.

Correct Answer: 2,4,5 Rationale 1: Exenatide is an injectable drug. Rationale 2: Exenatide is often injected twice daily. Rationale 3: This is not an expected adverse reaction. Rationale 4: Diarrhea is an expected adverse effect of this drug. Rationale 5: One of the actions of this drug is to increase secretion of insulin.

The client receives acyclovir (Zovirax) as treatment for herpes simplex type 1 virus (genital herpes). What is the best expected outcome for this client? 1. The client will prevent a reoccurrence of infection in the affected area. 2. The client will identify the names of sexual contacts in the past month. 3. The client will report decreased pain using the approved pain scale. 4. The client will report a decrease in the number of lesions in the affected area.

Correct Answer: 3 Rationale 1: It is not possible to prevent reoccurrence. Rationale 2: It is not necessary to provide the names of sexual contacts, and this is not related to acyclovir (Zovirax). Rationale 3: Pain is a major problem associated with this infection; the best outcome is decreased pain. Rationale 4: Decrease in lesions is not an outcome goal of treatment with acyclovir (Zovirax).

The mother of a 4-year-old boy states, I cant believe my son has type 1 diabetes. We eat well and I was so careful during the pregnancy. What could have caused this? How should the nurse respond? Note: Credit will be given only if all correct choices and no incorrect choices are selected. Standard Text: Select all that apply. 1. There must have been something you were exposed to during your pregnancy that caused it. 2. Are there others in your family that have type 1 diabetes? 3. He must have been allowed to eat too much sugar. 4. We are not certain what causes type 1 diabetes. 5. It is thought to be a combination of factors.

Correct Answer: 2,4,5 Rationale 1: There is no known factor that would cause type 1 diabetes if the mother was exposed. Rationale 2: There is a genetic factor associated with type 1 diabetes. Rationale 3: This is not a therapeutic statement and supports the urban myth that diabetes is caused by sugar ingestion. Rationale 4: The specific factors that cause Type 1 diabetes are still undiscovered. Rationale 5: It is thought that type 1 diabetes is caused by a number of interrelated factors.

A client receiving chemotherapy has a very low white blood cell count. Antibiotic therapy is initiated. What rationales should the nurse provide for the addition of this drug? Select all that apply. 1. You have an infection. 2. We would like to prevent you from developing any infections. 3. Chemotherapy often causes infections. 4. This antibiotic will help your chemotherapy fight off your cancer. 5. If you are developing an infection, this medication will help kill it out early.

Correct Answer: 2,5 Rationale 1: There is no evidence of infection being present. Rationale 2: Antibiotics are given to those with low WBCs to help prevent or lessen infections. Rationale 3: The chemotherapy does not cause an infection, but does decrease immunity, allowing infection to occur. Rationale 4: This antibiotic is not being given to kill cancer cells. Rationale 5: Prophylactic antibiotics are given to kill bacteria while their numbers are small.

A client has been diagnosed with multidrug-resistant tuberculosis, and drug therapy has been initiated. The nurse evaluates that medication education has been effective when the patient says, "I can expect to take this medication for up to _____ months." Record your answer rounding to the nearest whole number

Correct Answer: 24 Rationale: Tuberculosis treatment requires long-term antibiotic therapy for up to 12 months. When the tuberculosis is multidrug-resistant, treatment may extend up to 24 months.

The client is very distraught that her 12-year-old son is of very short stature. What is the best plan by the nurse? 1. Tell the client that treatment with growth hormone might result in acromegaly. 2. Tell the client that treatment for short stature is too expensive for many to afford. 3. Tell the client that treatment with growth hormone might help, and refer her to an endocrinologist. 4. Tell the client that treatment with growth hormone can add 6 inches of height to her son.

Correct Answer: 3 Rationale 1: Acromegaly is the result of too much growth hormone in adults. Rationale 2: Treatment with growth hormone is very expensive, but it is up to the client to decide whether or not to pursue treatment. Rationale 3: Clients should be referred to endocrinologists to discuss the uses of this hormone. Rationale 4: In order to achieve this much additional growth, treatment would have to be started very early in life, not at age 12.

A client has diabetes insipidus and receives desmopressin (DDAVP). The nurse completes medication education and evaluates that learning has occurred when the client makes which statement? 1. This medication is a potent vasodilator; my blood pressure can fall. 2. This medication promotes diuresis in my body; my blood pressure can fall. 3. This medication increases water reabsorption in my kidneys. 4. This medication suppresses hormone secretion from my posterior pituitary gland.

Correct Answer: 3 Rationale 1: Desmopressin is a potent vasoconstrictor, not a vasodilator, and can cause blood pressure to rise, not fall. Rationale 2: Desmopressin promotes water retention; blood pressure can rise, not fall. Rationale 3: Desmopressin is the same as ADH (antidiuretic hormone), and acts on the collecting ducts in the kidney to increase water reabsorption. Rationale 4: Desmopressin does not suppress hormone secretion from the posterior pituitary gland.

The client has been diagnosed with Cushings syndrome. What will the nurses assessment likely reveal? 1. Low blood pressure and hypoglycemia 2. Well-healed scars on the upper body 3. Upper body obesity 4. Thin, gaunt appearance of the face

Correct Answer: 3 Rationale 1: Hypertension and hyperglycemia are commonly seen. Rationale 2: Delayed wound healing is a sign of Cushings disease. Rationale 3: Primary symptoms of Cushings syndrome include upper body obesity. Rationale 4: A redistribution of fat around the face results in a moon face appearance.

The client receives glucocorticoid therapy. The nurse would prioritize assessment for which finding? 1. Hypothermia 2. Hypotension 3. Hypertension 4. Weight loss

Correct Answer: 3 Rationale 1: Hypothermia would not be seen; temperature regulation is not related to glucocorticoid therapy. Rationale 2: Hypertension would be expected related to the increased production of angiotensin II. Rationale 3: Hypertension would be expected related to the increased production of angiotensin II. Rationale 4: Weight loss would not be seen; weight gain is more likely with glucocorticoid therapy.

The client receives treatment with radioactive iodine (Iodine-131) therapy. What will the best evaluation by the nurse reveal? 1. The client will only temporarily accomplish the euthyroid state. 2. The client could safety become pregnant while receiving this treatment. 3. The client will most likely require thyroid replacement therapy. 4. The client does not have to distance herself from others.

Correct Answer: 3 Rationale 1: Treatment with radioactive iodine (Iodine-131) usually results in a permanent euthyroid state. Rationale 2: This therapy is contraindicated in pregnant clients. Rationale 3: Clients treated with radioactive iodine (Iodine-131) therapy often end up with hypothyroidism and require replacement therapy. Rationale 4: Physical distancing is necessary for others to prevent exposure to radiation.

Discharge planning for the client prescribed tetracycline will include which instruction? 1. Take the medication with antacids. 2. Take the medication with iron supplements. 3. Do not take the medication with milk. 4. Decrease the amount of vitamins.

Correct Answer: 3 Rationale 1: Antacids can decrease the effectiveness of tetracycline. Rationale 2: Iron can decrease the effectiveness of tetracycline. Rationale 3: Tetracycline effectiveness can be decreased by using milk products. Rationale 4: It is not necessary to decrease vitamins.

The client has just begun highly active antiretroviral therapy (HAART) for the treatment of acquired immune deficiency syndrome (AIDS). Which teaching point is a priority for this client? 1. Medications must be taken for 3 years after viral load is not measurable. 2. The goal of highly active antiretroviral therapy (HAART) is to reduce plasma human immunodeficiency virus (HIV) ribonucleic acid (RNA) to the lowest possible level. 3. Taking medications as scheduled is vital to successful treatment. 4. Know which medications target which phases of the human immunodeficiency virus (HIV) replication cycle.

Correct Answer: 3 Rationale 1: Medications must be continued for the lifetime of the client. Rationale 2: The goal of highly active antiretroviral therapy (HAART) is to reduce plasma human immunodeficiency virus (HIV), but this is not as important as medication compliance. Rationale 3: Taking medicines, as scheduled, is vital to maintain adequate treatment and prevent resistance to the medication. Rationale 4: The client may be interested in knowing which medications target which phases of the human immunodeficiency virus (HIV) ribonucleic acid (RNA) reproduction cycle, but this is not as important as medication compliance.

What is the primary function of the islets of Langerhans in the pancreas? 1. Secretion of enzymes 2. Acting as exocrine 3. Secretion of glucagon and insulin 4. Absorption of insulin

Correct Answer: 3 Rationale 1: Secretion of enzymes relates to the exocrine function, which is to release enzymes for chemical digestion of nutrients. Rationale 2: Other cells of the pancreas are responsible for exocrine function. Rationale 3: The cluster of cells within the pancreas (islets of Langerhans) is responsible for the endocrine function, which is to release insulin and glucagon. Rationale 4: These are secretory cells, not absorptive cells.

The client has type 1 diabetes and receives insulin. He asks the nurse why he cant just take pills instead. What is the best response by the nurse? 1. I know it is tough, but you will get used to the shots soon. 2. Have you talked to your doctor about taking pills instead? 3. Insulin cant be in a pill because it is destroyed in stomach acid. 4. Insulin must be injected because it needs to work quickly.

Correct Answer: 3 Rationale 1: Telling the client he will get used to the shots does not answer his question and is condescending. Rationale 2: The nurse should answer the clients question, not refer him back to the physician. Rationale 3: Insulin must be injected because it is destroyed in stomach acid if taken orally. Rationale 4: Insulin must be injected because it is destroyed in stomach acid if taken orally; the onset of action is not the issue here.

The client has acquired immune deficiency syndrome (AIDS) and has just learned she is pregnant. She tearfully asks the nurse if her baby will die of acquired immune deficiency syndrome (AIDS). What is the nurse's best response? 1. "There are special tests that we can do to see if your baby is positive before birth." 2. "I know that is a great worry for all of us." 3. "Drug therapy can help prevent your child from contracting the virus." 4. "There are drugs that can be given after birth to reverse AIDS."

Correct Answer: 3 Rationale 1: The patient's concern is about her baby dying from AIDS. Testing options is not the best answer. Rationale 2: Worrying about mother-to-baby transmission is a concern for all involved in this case, but this statement is not therapeutic. Rationale 3: A regime of oral zidovudine (Retrovir) will decrease the possibility of mother-to-baby transmission by 70%. Rationale 4: There are no drugs to "reverse" AIDS.

The nurse does health teaching with a client who has acquired immune deficiency syndrome (AIDS) and who has been started on antiviral medications. The nurse recognizes that teaching has been effective when the client makes which statement? 1. "I will need to limit my travel to avoid people with other infections." 2. "I will need to be on a high-calorie diet to enhance the effect of the medication." 3. "I will still need to take precautions to avoid spreading the virus to others." 4. "I will not be able to continue working at my high-stress job anymore."

Correct Answer: 3 Rationale 1: There is no need to limit travel at this point. Rationale 2: A well-balanced diet is the best diet for a client with acquired immune deficiency syndrome (AIDS). Rationale 3: Precautions are still necessary to avoid spreading the virus to others. Rationale 4: A high-stress job is not ideal, but at this point the client doesn't need to stop working.

The client injects his insulin as prescribed, but then gets busy and forgets to eat. What is the nurses most likely assessment finding? 1. The client will be very thirsty. 2. The client will need to urinate. 3. The client will have moist skin. 4. The client will complain of nausea.

Correct Answer: 3 Rationale 1: Thirst is a sign of hyperglycemia; the client would experience hypoglycemia if he did not eat. Rationale 2: Increased urination is a sign of hyperglycemia; the client would experience hypoglycemia if he did not eat. Rationale 3: Moist skin is a sign of hypoglycemia, which the client would experience if he injected insulin and did not eat. Rationale 4: Nausea is a sign of hyperglycemia; the client would experience hypoglycemia if he did not eat.

The student nurse asks the nursing instructor for help with her microbiology class. The student is studying bacteria. What does the best instruction by the nursing instructor include? Select all that apply. 1. Bacteria are either aerobic or anaerobic. 2. Bacteria are multicellular organisms. 3. E. coli are gram-negative bacteria. 4. Gram-staining is one way to identify bacteria. 5. Spherical-shaped bacteria are called cocci.

Correct Answer: 3,4,5 Rationale 1: Some organisms have the ability to change their metabolism and survive in either aerobic or anaerobic conditions. Rationale 2: Bacteria are single-celled organisms. Rationale 3: E. coli are gram-negative bacteria. Rationale 4: Gram-staining is one way to identify bacteria. Rationale 5: Spherical-shaped bacteria are called cocci.

An elderly client has hypothyroidism. Which assessment finding would the nurse report to the physician immediately? 1. Dry skin 2. Generalized weakness 3. Muscle cramps 4. Slurred speech

Correct Answer: 4 Rationale 1: Dry skin is an early sign of hypothyroidism and is not an emergent situation. Rationale 2: Generalized weakness is an early sign of hypothyroidism and is likely the reason the client sought treatment. It is not emergent. Rationale 3: Muscle cramps are an early sign of hypothyroidism and are not emergent. Rationale 4: Slurring of the speech in an elderly client may be related to one of the more severe symptoms of hypothyroidism or may be related to a neurological condition such a CVA. This finding should be reported.

The nurse plans to administer intranasal desmopressin (DDAVP) to the client. What will the best plan by the nurse include? 1. Instruct the client to blow his nose following administration. 2. Be sure to have fresh water at the bedside. 3. Withhold other medications so absorption of desmopressin (DDAVP) will not be affected. 4. Direct the spray high into the nasal cavity.

Correct Answer: 4 Rationale 1: Intranasal sprays of desmopressin (DDAVP) should be alternately rotated between nares. There is no need for the client to blow his nose following administration of intranasal desmopressin (DDAVP). Rationale 2: Fresh water is a good idea, but not necessary with intranasal desmopressin (DDAVP). Rationale 3: There is no need to withhold other medications when a client receives intranasal desmopressin (DDAVP). Rationale 4: The spray should be directed high into the nasal cavity, not into the nasopharynx.

Negative feedback loop ensures endocrine homeostasis by 1. stimulating secretion from the first hormone. 2. inhibiting release of the second hormone. 3. preventing under the responses of the endocrine system. 4. inhibiting secretion from the first hormone.

Correct Answer: 4 Rationale 1: Negative feedback does not stimulate secretion from the first hormone. Rationale 2: Negative feedback does not stimulate secretion from the second hormone. Rationale 3: Negative feedback does not prevent a response from the endocrine system. Rationale 4: Negative feedback loop inhibits secretion of the first hormone.

A client comes to the emergency department complaining of a sore throat. He has white patches on his tonsils, and he has swollen cervical lymph nodes. What will the best plan by the nurse include? 1. Plan to administer a narrow-spectrum antibiotic. 2. Plan to administer a broad-spectrum antibiotic. 3. Plan to obtain blood cultures. 4. Plan to obtain a throat culture

Correct Answer: 4 Rationale 1: Initial therapy with a narrow-spectrum antibiotic is too specific without knowing the causative organism. Rationale 2: A broad-spectrum antibiotic is commonly ordered, but a throat culture should be obtained first. Rationale 3: Blood cultures are not necessary at this point because the infection is in the throat; it is not systemic. Rationale 4: A throat culture is necessary to identify the causative organism and initiate the best antibiotic treatment.

A client comes to the emergency department with a fever of 104°F. The nurse anticipates which actions to help identify the correct antibiotic? 1. Obtaining liver and renal function tests. 2. Obtaining a complete blood count (CBC) test. 3. Obtaining a sterile urine specimen. 4. Obtaining blood for culture and sensitivity.

Correct Answer: 4 Rationale 1: Liver and renal function tests will not identify the causative organism. Rationale 2: The CBC will reveal the impact of infective agents on the immune system but will not identify the agent. Rationale 3: Obtaining a urine specimen is not the best method of determining this client's infective agent. Rationale 4: A high fever is usually indicative of a systemic infection. Blood cultures are the best way of identifying the causative organism.

The physician orders penicillin for a female client who has a sinus infection. What is a priority question to ask the client prior to administering the medication? 1. "Are you pregnant?" 2. "Do you plan to become pregnant?" 3. "Are you breastfeeding?" 4. "Are you taking birth control pills?"

Correct Answer: 4 Rationale 1: Penicillin is a Pregnancy Category B drug and is safe to take if a client is pregnant. Rationale 2: Penicillin is a Pregnancy Category B drug and is safe to take if a client plans to become pregnant. Rationale 3: Penicillin is a Pregnancy Category B drug and is safe to take if a client is breastfeeding. Rationale 4: Penicillin can cause birth control pills to lose their effectiveness.

The nurse accidentally sticks herself with a needle after starting an intravenous (IV) line on a client with acquired immune deficiency syndrome (AIDS). The nurse tells the supervisor about the accident. What is the best initial response by the supervisor to decrease anxiety in the nurse? 1. "If you are started on medications soon, it will decrease the severity of the disease." 2. "Workers compensation will cover the cost of your illness and medications." 3. "Did you use the hospital protocols for starting intravenous (IV) lines on a client with acquired immune deficiency syndrome (AIDS)?" 4. "Fortunately, the chances of you contracting human immunodeficiency virus (HIV) after the stick are very small."

Correct Answer: 4 Rationale 1: Telling the nurse to start on medication will not reduce anxiety. Rationale 2: If the nurse did contract human immunodeficiency virus (HIV), workers compensation would cover the cost, but this response will not reduce anxiety. Rationale 3: Asking the nurse if she used protocols is very non-therapeutic at this point and will most likely increase anxiety. Rationale 4: There have been very few cases of client-to-health care worker transmission; the risk is very small.

The principal goals of therapy for older patients who have poor glycemic control are: A Enhancing quality of life. B Decreasing the chance of complications. C Improving self-care through education. D All of the above

D Question 16 Explanation: The principal goals of therapy for older persons with diabetes mellitus and poor glycemic control are enhancing quality of life, decreasing the chance of complications, improving self-care through education, and maintaining or improving general health status.

A 50-year-old widower is admitted to the hospital with a diagnosis of diabetes mellitus and complaints of rapid-onset weight loss, elevated blood glucose levels, and polyphagia, the gerontology nurse should anticipate which of the following secondary medical diagnoses? A Impaired glucose tolerance B Gestational diabetes mellitus C Pituitary tumor D Pancreatic tumor

D Question 21 Explanation: The onset of hyperglycemia in the older adult can occur more slowly. When the older adult reports rapid-onset weight loss, elevated blood glucose levels, and polyphagia, the healthcare provider should consider pancreatic tumor.

An older woman with diabetes mellitus visits the clinic concerning her condition. Of which of the following symptoms might an older woman with diabetes mellitus complain? A Anorexia B Pain intolerance C Weight loss D Perineal itching

D Question 22 Explanation: Older women might complain of perineal itching due to vaginal candidiasis.

An ailing 70-year-old woman with a diagnosis of type 2 diabetes mellitus has been ill with pneumonia. The client's intake has been very poor, and she is admitted to the hospital for observation and management as needed. What is the most likely problem with this patient? A Insulin resistance has developed. B Diabetic ketoacidosis is occurring. C Hypoglycemia unawareness is developing. D Hyperglycemic hyperosmolar non-ketotic coma

D Question 26 Explanation: Illness, especially with the frail elderly patient whose appetite is poor, can result in dehydration and HHNC. Insulin resistance usually is indicated by a daily insulin requirement of 200 units or more. Diabetic ketoacidosis, an acute metabolic condition, usually is caused by absent or markedly decreased amounts of insulin.

The nurse is reviewing discharge teaching with a client who suffered an anaphylactic reaction to a bee sting. Which statement by the client indicates the need for further teaching? A. "I must wear a medical alert bracelet stating that I am allergic to bee stings." B. "I need to carry epinephrine with me." C. "My spouse must learn how to give me an injection." D. "I am immune to bee stings now that I have had a reaction."

D. "I am immune to bee stings now that I have had a reaction." Rationale: No immunity develops after an anaphylactic reaction. In fact, the next reaction could be more severe. The client should carry epinephrine (EpiPen) at all times and always wear a medical alert bracelet that states all allergies. Someone (spouse, neighbor, or family member) must learn how to give the client an injection in case the client is unable self-administer the injection

The nurse has finished teaching a client with diabetes mellitus how to administer insulin. The nurse evaluates that learning has occurred when the client makes which statement? Select one: A. "I should provide direct pressure over the site following the injection." B. "I should check my blood sugar immediately prior to the administration." C. "I should use the abdominal area only for insulin injections." D. "I should only use a calibrated insulin syringe for the injections."

D. "I should only use a calibrated insulin syringe for the injections."

After the nurse instills atropine drops into both eyes for a client undergoing ophthalmic examination. which of the following instructions would be given to the client? A. "Be careful because the blink reflex is paralyzed." B. "Avoid wearing your regular glasses when driving." C. "Be aware that the pupils may be unusually small." D. "Wear dark glasses in bright light because the pupils are dilated."

D. "Wear dark glasses in bright light because the pupils are dilated." Atropine. an anticholinergic drug. has mydriatic effects causing pupil dilation. This allows more light onto the retina and may cause photophobia and blurred vision. Atropine doesn't paralyze the blink reflex or cause miosis (pupil constriction). Driving may be contraindicated to blurred vision.

A middle-aged client, who is alert, is admitted to the emergency department with wheezing, difficulty breathing, angioedema, blood pressure of 70/52 mm Hg, and apical pulse of 122 beats/min and irregular. The nurse makes an immediate assessment using the "ABCs" for any client experiencing anaphylaxis. What nursing intervention is the immediate priority? A. Raise the lower extremities. B Start intravenous (IV) administration of normal saline. C. Reassure the client that appropriate interventions are being instituted. D. Apply oxygen using a high-flow non-rebreather mask at 40% to 60%.

D. Apply oxygen using a high-flow non-rebreather mask at 40% to 60%. Rationale: Oxygen application helps provide adequate oxygenation for the client who is in respiratory distress. Assessing respiratory status is an immediate priority; raising the lower extremities, starting an IV infusion, and reassuring the client are not the first priority because the client is in respiratory distress.

Which statement best exemplifies a client's protection from cancer provided by cell-mediated immunity (CMI) after exposure to asbestos? A Cytotoxic and cytolytic T cells destroy cells that contain the major histocompatibility complex of a processed antigen. B Helper and inducer T cells recognize self cells versus non-self cells and secrete lymphokines that can enhance the activity of white blood cells. C Suppressor T cells prevent hypersensitivity when a client is exposed to non-self cells or to proteins. D. Balance elicits protection when helper or inducer T cells outnumber suppressor T cells by a ratio of 2:1

D. Balance elicits protection when helper or inducer T cells outnumber suppressor T cells by a ratio of 2:1 Rationale: Optimal function of CMI requires a balance between helper and inducer T cells and suppressor T cells. This balance occurs when helper and inducer T cells outnumber suppressor T cells by a ratio of 2:1. The activity of cytotoxic and cytolytic T cells is most effective against self cells infected by parasites. Overreactions can cause tissue damage if an imbalance exists between helper and inducer T cells. When suppressor T cells are increased, immune function is suppressed and the risk for infection increases.

The healthcare provider is caring for a patient who has diabetes and is also diagnosed with hypertension. Which of the following medications on the patient's medication administration record will cause the most concern? Please choose from one of the following options. Angiotensin receptor blocker ACE inhibitor Calcium channel clocker Beta-blocker

D. Beta-blocker Rationale: 1 / 3 Recall the mechanism of action of each of these medications and relate this to common complications of diabetes. Hint #22 / 3 Hypoglycemia is an acute complication that may be experienced by patients who are diabetic. Hint #33 / 3 Beta-blockers reduce blood pressure by blocking actions of the sympathetic nervous system, which is activated when a patient becomes hypoglycemic. Blocking of adrenergic beta receptors will also block many of the symptoms of hypoglycemia, hindering the patient's ability to recognize early signs of hypoglycemia.

The nurse is assessing the adaptation of the female client to changes in functional status after a brain attack (stroke). The nurse assesses that the client is adapting most successfully if the client: A. Gets angry with family if they interrupt a task B. Experiences bouts of depression and irritability C. Has difficulty with using modified feeding utensils D. Consistently uses adaptive equipment in dressing self

D. Consistently uses adaptive equipment in dressing self Clients are evaluated as coping successfully with lifestyle changes after a brain attack (stroke) if they make appropriate lifestyle alterations, use the assistance of others, and have appropriate social interactions. Options A, B, and C are not adaptive behaviors.

Which of the following would most likely be used for a patient allergic to penicillin? A. Clindamycin (Cleocin) B. Amoxicillin (Amoxil) C. Sulfisoxazole (Gantrisin) D. Erythromycin (E-mycin)

D. Erythromycin (E-mycin)

The nurse is developing a teaching plan for the client with glaucoma. Which of the following instructions would the nurse include in the plan of care? A. Decrease fluid intake to control the intraocular pressure B. Avoid overuse of the eyes C. Decrease the amount of salt in the diet D. Eye medications will need to be administered lifelong.

D. Eye medications will need to be administered lifelong. The administration of eye drops is a critical component of the treatment plan for the client with glaucoma. The client needs to be instructed that medications will need to be taken for the rest of his or her life.

A client with an acquired immune deficiency is seen in the clinic for re-evaluation of the immune system's response to prescribed medication. Which test result does the nurse convey to the health care provider? A. Therapeutic highly active antiretroviral therapy (HAART) level B. Positive human immune deficiency virus (HIV), enzyme-linked immunosorbent assay (ELISA), Western blot C. Positive Papanicolaou (Pap) test D. Improved CD4+ T-cell count and reduced viral load

D. Improved CD4+ T-cell count and reduced viral load Rationale: Improved CD4+ T-cell count and reduced viral load reflect the response to prescribed HAART medication. Therapeutic HAART level is the recommended medication combination given to clients with HIV to cause an increase in the CD4+ T-cell count. ELISA and Western blot, if positive, indicate that the client is HIV positive (a fact already known for this client) and do not indicate response to prescribed medication. Pap smears can be precancerous in an HIV-positive client, but the test does not indicate the immune system's response to prescribed medication.

A first-line drug for the treatment of M. tuberculosis is: A. Erythromycin (E-mycin, Erythrocin) B. Gentamicin (Garamycin) C. Vancomycin (Vancocin) D. Isoniazid (INH)

D. Isonizaid (INH)

What advantage does amoxicillin (Amoxil, Trimox) have over penicillin G? A. It is less expensive B. It has greater absorption C. It has fewer side effects D. It has penicillinase resistance

D. It has penicillinase resistance

An unresponsive patient who has diabetes is brought to the emergency department with slow, deep respirations. Additional findings include: blood glucose 450 mg/dL (24.9 mmol/L), arterial pH 7.2, and urinalysis showing presence of ketones and glucose. Which of the following statements best describes the the underlying cause of this patient's presentation? Please choose from one of the following options. Hypoglycemia causes release of glucagon resulting in glycogenolysis and hyperglycemia Nocturnal elevation of growth hormone results in hyperglycemia in the morning Hyperglycemia causes oxidative stress, renal dysfunction, and acidosis Lack of insulin causes increased counterregulatory hormones and fatty acid release

D. Lack of insulin causes increased counterregulatory hormones and fatty acid release Rationale: 1 / 3 The cells of this patient are in a state of "starvation." Hint #22 / 3 Think of why the cells of diabetic patients are not able to utilize glucose. Hint #33 / 3 This patient is suffering from diabetic ketoacidosis, caused by insulin deficiency. The body responds by releasing an counterregulatory hormones and undergoing lipolysis to increase glucose, resulting in hyperglycemia, ketogenesis, and acidosis.

Clinical nursing assessment for a patient with microangiopathy who has manifested impaired peripheral arterial circulation includes all of the following except: A. Integumentary inspection for the presence of brown spots on the lower extremities B. Observation for paleness of the lower extremities C. Observation for blanching of the feet after the legs are elevated for 60 seconds D. Palpation for increased pulse volume in the arteries of the lower extremities

D. Palpation for increased pulse volume in the arteries of the lower extremities

Nurse John is assigned to care for a postoperative male client who has diabetes mellitus. During the assessment interview, the client reports that he's impotent and says he's concerned about its effect on his marriage. In planning this client's care, the most appropriate intervention would be to: A. Encourage the client to ask questions about personal sexuality. B. Provide time for privacy. C. Provide support for the spouse or significant other. D. Suggest referral to a sex counselor or other appropriate professional

D. Suggest referral to a sex counselor or other appropriate professional. The nurse should refer this client to a sex counselor or other professional. Making appropriate referrals is a valid part of planning the client's care. The nurse doesn't normally provide sex counseling.

A nurse is caring for a client with diabetic ketoacidosis and documents that the client is experiencing Kussmaul's respirations. Based on this documentation. which of the following did the nurse most likely observe? A.Respirations that cease for several seconds B.Respirations that are regular but abnormally slow C. Respirations that are labored and increased in depth and rate D. Respirations that are abnormally deep. regular. and increased in rate

D.Respirations that are abnormally deep. regular. and increased in rate Kussmaul's respirations are abnormally deep. regular. and increased in rate. In apnea. respirations cease for several seconds. In bradypnea. respirations are regular but abnormally slow. In hyperpnea. respirations are labored and increased in depth and rate.

The nurse is caring for a postoperative craniotomy patient. Which prescribed drug does the nurse make sure is readily available to treat acute seizure activity? Flumazenil (Romazicon) Gabapentin (Neurontin) Diazepam (Valium) Ethosuximide (Zarontin)

Diazepam (Valium) Drug therapy for acute seizure activity is typically diazepam (Valium), which is considered by many to be the drug of choice. Other drugs used for acute therapy include lorazepam, fosphenytoin, phenytoin, and phenobarbital.

Ergot alkaloids exert their therapeutic effect by which action? Simulation of the alpha receptors. Vasodilation. Vasoconstriction. Blockade of the beta2 receptors.

Vasoconstriction. Ergot alkaloids narrow or constrict blood vessels in the brain. They are useful in the treatment of migraine headache caused by vasodilation of vessels in the brain. Ergot alkaloids were the mainstay of treatment of migraine headaches but have been replaced by the triptans for first-line therapy.

The nurse makes a home visit to a client with diabetes mellitus. During the visit, the nurse notes that the client's 3-month supply of insulin vials that were delivered a week ago are not refrigerated. What is the best action by the nurse at this time?

Vials can be stored at room temperature up to one month. For longer storage, they should be refrigerated.

How is the effectiveness of antiviral drugs administered to treat HIV infection assessed and evaluated? Viral load Red blood cell counts Lymphocyte counts Megakaryocytes Submit

Viral load All antiretroviral drugs work to reduce the viral load, which is the number of viral RNA copies per milliliter of blood.

Quinolones are a class of antibiotics known for several significant complications. Which are possible adverse effects with these drugs? (Select all that apply.) a. Prolongation of the QT interval b. Ototoxicity c. Nephrotoxicity d. Tendon rupture e. Abnormal cartilage development in children

a, d, e Quinolones are not used in prepubescent children because of the risk of cartilage development issues. Quinolones may also cause a cardiac effect that involves prolongation of the QT interval on the electrocardiogram. The use of these medications can result in tendonitis or ruptured tendons in adults. Nephrotoxicity and ototoxicity are not associated with quinolones.

Nurse Gina is taking care of a client that was diagnosed with Addison Disease. The physician ordered a mineralocorticoid and a glucocorticoid to treat his condition. The client asks the nurse why these medications are necessary. The following response by the nurse can best describe why he needs to take these medications: a. The medications the physician ordered will replace the hormone that your body is not producing adequately. b. Your parathyroid gland is not secreting the cortisol that your body needs. c. Your body is producing too much hormone and this medication will suppress your immune response to prevent the secretion of too much cortisol. d. The steroids that the physician ordered will help your muscles grow since your condition will attack your muscles.

a

The nurse understands that which of the following drugs falls under the classification of biguanides? a. Metformin HCI (Glucophage) b. Repaglinide (Prandin) c. Tolbutamide (Orinase) d. Acarbose (Precose)

a

When administering niacin, the nurse needs to monitor for which adverse effect? a. cutaneous flushing b. muscle pain c. headache d. constipation

a

What are important for the nurse to monitor in a client receiving an antifungal medication? (Select all that apply.) a. Blood urea nitrogen b. Daily weights c. Creatinine d. Mental status e. Intake and output

a, b, c, e Nursing interventions appropriate to clients receiving antifungal drugs vary depending on the particular drug. However, it is important for the nurse to monitor all clients for indications of possible medication-induced renal damage so that prompt interventions can occur to prevent further dysfunction. Monitoring intake and output amounts, daily weights, and renal function tests will help prevent such damage.

When providing instructions to clients on use of antibiotics, which instructions would the nurse include in the teaching? (Select all that apply.) a. Wash your hands before and after preparing food. b. Complete the entire course of therapy. c. Increase fluid intake up to 3000 mL/day. d. Save unused medication in a cool dry place for later use. e. Notify the provider of any possible reactions that occur.

a, b, c, e There should not be any leftover medication, but if there is, it needs to be discarded in the appropriate method. The health care provider typically only writes a prescription for the exact amount of medication needed by the client.

Which types of antiviral drugs are used to treat HIV infection? (Select all that apply.) a. Nonnucleoside reverse transcriptase inhibitors b. Protease inhibitors c. Neuraminidase inhibitors d. Fusion inhibitors e. Reverse transcriptase inhibitors

a, b, d, e Neuraminidase inhibitors are used in the treatment of the influenza virus.

Which are known contraindications for use of erythropoiesis-stimulating agents (ESAs) in clients? (Select all that apply.) a. History of thrombosis b. Cancer of the neck c. End-stage renal disease d. Uncontrolled hypertension e. Hemoglobin level of 15 g/dL

a, b, d, e Contraindications for ESAs include known drug allergy. Use of epoetin and darbepoetin is contraindicated in cases of uncontrolled hypertension and when hemoglobin levels are above 10 g/dL for clients with cancer and 11 g/dL for clients with renal disease. Use in clients with head or neck cancers or clients at risk for thrombosis is controversial because these medications increase tumor growth and the risk for thrombosis.

Which groups of individuals are at highest risk for development of iron deficiency anemia? (Select all that apply.) a. Women ages 12 to 40 years b. Men ages 20 to 40 years c. Children d. Men older than age 50 years e. Pregnant women

a, c, e, Individuals who require the highest amount of iron are women (especially pregnant women) and children, and they are the groups most likely to develop iron-deficiency anemia. For women, this is partly because of ongoing menstrual blood losses. Most vitamin supplements for men contain little or no iron because men are much less likely to develop iron-deficiency anemia.

The nurse is teaching a patient about the inhaler advair (salmeterol/fluticasone). Which statements by the patient indicate a correct understanding of this medication? Select all that apply a. "I will rinse my mouth with water after each dose" b. " I need to use this inhaler whenever I feel short of breath" c. "This medication is taken twice a day, every 12 hours." d. "I can take this inhaler if I get short of breath while exercising." e. "I will call my doctor if I notice white patches inside my mouth."

a,c,e

Which statement by the patient indicates a need for further instruction about colestipol (Colestid) from the nurse? a. "I will mix and stir the powder thoroughly with at least 1 to 2 oz of fluid." b. "I should take this medication 1 hour after or 4 hours before my other medications." c. "The potential adverse effects of this drug are rash and itching." d. "I might need to take fat-soluble vitamins to supplement my diet."

a. "I will mix and stir the powder thoroughly with at least 1 to 2 oz of fluid." Colestipol is available in powder form that must be mixed thoroughly with food or fluids (at least 4 to 6 oz of fluid) before administration to avoid esophageal irritation or obstruction and intestinal obstruction.

When developing a nursing care plan for a client receiving epoetin alfa (Epogen), the nurse will include monitoring for which adverse effect? a. Change in level of consciousness b. Impaired liver function c. Severe hypotension d. Chronic diarrhea

a. Change in level of consciousness Epoetin alfa has an increased risk of thrombolic events. If the client has a change in the level of consciousness, this might be a sign of a stroke or myocardial infarction. Clients taking this medication need assessment and monitoring for this possibility.

Hydroxymethylglutaryl-coenzyme A (HMG-CoA) reductase inhibitors (statins) are generally administered at which time of day? a. Evening b. 12:00 noon c. afternoon d. morning

a. Evening The liver produces the majority of cholesterol during the night. Thus, statin drugs, which decrease the cholesterol synthesis, are generally administered in the evening or bedtime so that the peak drug levels coincide with cholesterol production.

When assessing a patient who is to receive a decongestant, the nurse will recognize that a potential contraindication to this drug would be which condition? a. Glaucoma b. Fever c. peptic ulcer disease d. Allergic rhinitis

a. Glaucoma

The nurse would question a prescription for simvastatin (Zocor) in a patient with which condition? a. hepatic disease b. diabetes c. luekemia d. heart failure

a. Hepatic disease Simvastatin (Zocor) can cause an increase in liver enzymes and thus should not be used in patients with preexisting liver disease.

By which action does atorvastatin (Lipitor) decrease lipid levels? a. Inhibiting HMG-CoA reductase, the enzyme responsible for the biosynthesis of cholesterol in the liver b. Decreasing the amount of triglycerides produced by the liver and increasing the removal of triglycerides by the liver c. Binding to bile in the intestinal tract, forming an insoluble complex that is excreted in the feces d. Stimulating the gallbladder and biliary system to increase excretion of dietary cholesterol

a. Inhibiting HMG-CoA reductase, the enzyme responsible for the biosynthesis of cholesterol in the liver Atorvastatin (Lipitor) is an HMG-CoA reductase inhibitor that inhibits HMG-CoA reductase, the enzyme needed to make cholesterol in the liver.

What advantage does loratadine (Claritin) have compared with traditional antihistamines such as diphenhydramine (Benadryl)? a. less sedative effect b. minimal gastrointestinal effect c. increased bronchodialation d. decreased risk of cardiac dysrhythmias

a. Less sedative effect Loratadine is a nonsedating antihistamine and it does not readily distribute into the central nervous system, which diminishes the sedative effects associated with traditional antihistamines.

The nurse is providing discharge teaching for a patient about potential serious adverse effects to simvastatin (Zocor). Which symptom may indicate the patient is experiencing a serious adverse effect to this medication? a. muscle pain b. itching c. headache d. weight loss

a. Muscle pain Unexplained muscle pain and soreness are symptoms of a relatively uncommon but serious adverse effect of rhabdomyolysis associated with statin drugs and must be immediately reported to the health care provider.

When administering ferrous sulfate (iron) to a client, the nurse plans to give this medication with which fluid to increase absorption of the iron? a. orange juice b. black tea c. 8 oz of water d. 4 oz of milk

a. Orange juice The absorption of iron can be enhanced when it is given with ascorbic acid (vitamin C), which is present in orange juice.

The nurse is caring for a client who is prescribed epoetin alfa (Epogen). What does the nurse expect as the reason for use of this medication in this client? a. Treatment of anemia due to renal failure b. To replace blood loss from surgery c. To cure or reverse kidney failure d. Management of pregnancy-associated anemia

a. Treatment of anemia due to renal failure Epoetin alfa (Epogen) is the biosynthetic form of erythropoietin and is a hormone produced by the kidneys in response to a decrease in erythrocytes. The client in renal failure has impaired kidney function and therefore may have this medication prescribed to treat anemia that is associated with end-stage renal disease that results from the lack of natural production of this hormone.

The nurse notes in a patient's medication history that the patient is taking benzonatate (Tessalon Perles) as needed. Based on this finding, the nurse interprets that the patient has which problem? a. Cough b. seasonal allergies c. chronic rhinitis d. motion sickness

a. cough

The nurse makes a home visit to a client with diabetes mellitus. During the visit the nurse notes that the client's 3 month supply of insulin vials were delivered a week ago are not refrigerated. What is the best action by the nurse at this time? a. have the client place the insulin in vials in the refrigerator b. have the client discard the vials c. instruct the client to label each vial with the date when opened d. tell the client this is too much insulin to have on hand

a. have the client place the insulin in vials in the refrigerator

The nurse has finished teaching a client with diabetes mellitus how to administer insulin. The nurse evaluates that learning as occurred when the client makes which statement? a. i should only use a calibrated insulin syringe for insulin injections b. i should check my blood sugar immediately prior to the administration c. i should use the abdominal area only for insulin injections d. i should provide direct pressure over the site following the injection

a. i should only use a calibrated insulin syringe for insulin injections

The nurse is providing education to a client with a history of chronic nasal congestion secondary to allergic rhinitis. Which class of medications should the nurse anticipate the provider would recommend for the client to use on a long-term basis? a. intranasal steroids b. expectorants c. antihistamines d. antitussives

a. intranasal steroids (Inhaled intranasal steroids and anticholinergic drugs are not associated with rebound congestion and are often used prophylactically to prevent nasal congestion in clients with chronic upper respiratory tract symptoms. Local intranasal steroids would have the least likely possible systemic adverse effects of all the medication classes possible to use for chronic and long term use.)

The client receives metformin (glucophage). What will the best plan by the nurse include with regard to patient education with this drug? (select all that apply) a. it decreases sugar production in the liver b. it inhibits absorption of carbohydrates c. it stimulates the pancreas to produce more insulin d. it reduces insulin resistance e. it increases energy use

a. it decreases sugar production in the liver d. it reduces insulin resistance

A client has been prescribed levothyroxine (Levothyroid). What med information should the nurse provide?(select all) a. it may take a few weeks for you to see the full benefits from this drug b. be sure to keep all of your follow-up appointments c. take this medication at whatever time you eat your evening meal d. do not start a fiber laxative without first discussing it with you r health care team. e. take your calcium supplement at least 4 hours after taking this drug

a. it may take a few weeks for you to see the full benefits from this drug b. be sure to keep all of your follow-up appointments d. do not start a fiber laxative without first discussing it with your health care team. e. take your calcium supplement at least 4 hours after taking this drug

The physician writes orders for the client with diabetes mellitus. Which order would the nurse validate with the physician? a. lantus insulin 20U BID b. administering regular insulin 30 minutes prior to meals c. 5 units of Humalog/10 units NPH daily d. metformin (glucophage) 1000mg per day in divided doses

a. lantus insulin 20U BID

The nurse understands that which of the following drugs falls under the classification of biguanides? a. metformin HCl (glucophage) b. repaglinide (Prandin) c. tolbutamide (orinase) d. acarbose (precose)

a. metformin HCl (glucophage)

The nursing instructor teaches the student nurses about the endocrine system. The nursing instructor evaluates that learning has occurred when the student nurses make which statements? (Select all that apply) a. the hypothalamus secretes the releasing hormones b. the hormones released by the endocrine system influence every organ in the body c. the hypothalamus is considered the master gland d. the pituitary gland secretes TSH (thyroid stimulating hormone) e. the endocrine system is a major controller of homeostasis.

a. the hypothalamus secretes the releasing hormones b. the hormones released by the endocrine system influence every organ in the body d. the pituitary gland secretes TSH (thyroid stimulating hormone) e. the endocrine system is a major controller of homeostasis.

select all that apply for adverse effects of long term corticosteroid therapy: a. peptic ulcers b. myopathy c. behavioral changes d. osteoporosis e. potassium alteration

all a. peptic ulcers b. myopathy c. behavioral changes d. osteoporosis e. potassium alteration

An example of a medication that stimulates the pancreas to secrete more insulin is: a. metformin b. glyburide c. Glucophage. d. pioglitazone

b

Insulin is normally released when: a. the blood glucose stays the same. b. the blood glucose increases. c. the blood glucose decreases. d. glucagon increases.

b

The mechanism of action of regular insulin is to: a. stimulate the pancreas to produce insulin. b. promote entry of glucose into the cells. c. promote the entry of glucose into the bloodstream. d. stimulate the pancreas to secrete more insulin.

b

The nurse is administering cholestyramine (Questran), a bile acid sequestrant. Which nursing intervention(s) is appropriate? select all that apply a. administering the drug on an empty stomach b. administering the drug with meals c. instructing the patient to follow a low-fiber supplement while taking this drug d. instructing the patient to take a fiber supplement while taking this drug e. Increasing fluid intake f. not administering this drug at the same time as other drugs.

b, d, e, f

The nurse caring for a client being treated for Addison's disease enters the patient's room to find him yelling hysterically at his wife. The wife tells the nurse that he just "snapped" and became mad about "nothing". The wife states, "This just isn't like him, I do not know what is going on". Which of the following responses could best explain the most likely reason for the client's outburst: a. Addison's disease commonly causes severe rage and mood swings and it is not his fault b. The medications he is receiving can cause irritability and mood swings, however, I will let the physician know about his symptoms because sometime this can happen if he receives too much of the medication used to treat his condition. c. You must have said something that caused him to get mad at you, the last time I went in there he was extremely pleasant. d. It has to be hard for him to have you in his room all the time, it would be best if you leave and let him have some time alone.

b

The nurse is teaching a patient about oral iron supplements. Which statement is correct? a. "You need to take this medication on an empty stomach or else it wont be absorbed. b. It is better absorbed on an empty stomach, but if that causes your stomach to be upset, you can take it with food." c. "Take this medication with a sip of water, and then lie down to avoid problems with low blood pressure." d. "If you have trouble swallowing the tablet, you may crush it."

b

Which actions describe the beneficial effects produced by sulfonylurea oral hypoglycemics? (Select all that apply.) a. Increase hepatic glucose production b. Stimulate insulin secretion from beta cells c. Enhance action of insulin in various tissues d. Inhibit breakdown of insulin by liver

b, c, d The sulfonylureas stimulate insulin secretion from the beta cells of the pancreas; enhance the actions of insulin in muscle, liver, and adipose tissue; and prevent the liver from breaking insulin down as fast as it ordinarily would (reduced hepatic clearance). Increased hepatic glucose production would serve to increase serum glucose levels, the opposite effect of oral hypoglycemic drugs.

Before administration of any antiviral medication, what nursing responsibilities would be performed? (Select all that apply.) Select all that apply. a. Monitor for medication adverse effects b. History of medication use c. Documentation of known allergies d. Baseline vital signs e. Head-to-toe physical assessment

b, c, d, e Before administering an antiviral drug, perform a thorough head-to-toe physical assessment and take a medical and medication history. Document any known allergies before use of these and any other medications. Also assess the client's nutritional status and baseline vital signs because of the profound effects of viral illnesses on physiologic status, especially if the client is immunocompromised. Assess and document any contraindications, cautions, and drug interactions associated with all of the antiviral drugs. Monitoring for adverse effects would occur after the medication has been administered.

The nurse is aware that viruses can enter the body through various routes. Through which routes can viruses enter the body? (Select all that apply.) a. Localized allergic reaction to medication b. Inhalation through the respiratory tract c. Transplacentally from mother to infant d. Through an animal bite e. Ingestion via the gastrointestinal (GI) tract

b, c, d, e Viruses can enter the body through at least four routes: (1) inhalation through the respiratory tract, (2) ingestion via the GI tract, (3) transplacentally via mother to infant, and (4) inoculation via skin or mucous membranes. The inoculation route can take several forms, including sexual contact, blood transfusions, sharing of syringes or needles, organ transplantation, and animal bites (including human, animal, insect, spider, and others). Viruses cannot enter the body through an allergic reaction caused by medication.

The nurse is discussing with the client the management of symptoms of an upper respiratory tract infection. Which classes of medications are often used in treating the symptoms of upper respiratory tract infections? (Select all that apply.) a. antibiotics b. antihistamines c. antitussives d. expectorants e. nasal decongestants

b, c, d, e, (Treatment of the common symptoms of upper respiratory tract infections involves the combined use of antihistamines, nasal decongestants, antitussives, and expectorants. Upper respiratory tract infections are often viral or allergic in nature and the symptoms would not be managed with antibiotics.)

a patient has been taking simvastatin (Zocor) for 6 months. Today he received a call that he needs to come to the office for a "laboratory check". The nurse expects which laboratory studies to be ordered at this time? select all that apply a. pt/INR b. total cholesterol c. triglyceride d. liver function studies e. complete blood count f. HDL and LDL levels

b, c, d, f

The nurse is discussing use of antihistamines for allergic rhinitis. Which medications would be included in the list of H1 antagonists used in the treatment of allergic rhinitis? (Select all that apply.) a. Nizatidine (Axid) b. Loratadine (Claritin) c. Fexofenadine (Allegra) d. Ranitidine (Zantac) e. Cetirizine (Zyrtec)

b, c, e, Antihistamines are drugs that directly compete with histamine for specific receptor sites. For this reason, they are also called histamine antagonists. H1 antagonists include drugs such as diphenhydramine (Benadryl), chlorpheniramine (generic), fexofenadine (Allegra), loratadine (Claritin), and cetirizine (Zyrtec).

Question 9 of 11 A client who is prescribed metronidazole (Flagyl) for a gynecologic infection provides the nurse with a list of medications that are routinely taken. Which medication would lead the nurse to question the prescription for Flagyl? a. Multivitamin (Thera-Tabs) b. Lithium (Eskalith) c. Ibuprofen (Advil) d. Levothyroxine (Synthroid)

b. Lithium (Eskalith) Concomitant use of lithium and metronidazole may result in lithium toxicity. Thus, a client who reports taking lithium should alert the nurse to notify the health care provider because of the potential significant interaction.

To assess for a potentially serious adverse effect to HMG-CoA reductase inhibitors, the nurse should monitor which laboratory results? a. Serum electrolytes b. urine specific gravity c. Liver function studies d. complete blood count

c. Liver function studies HMG-CoA reductase inhibitors can cause hepatic toxicity; thus, liver function studies are often measured every 6 to 8 weeks for the first 6 months of statin therapy and then every 3 to 6 months, depending on the prescriber and the patient situation.


Conjuntos de estudio relacionados

US Citizenship Questions (Spanish/English)

View Set

eeb 100 practice questions behavior

View Set

Ch. 3 Metabolism and Energy Balance Quiz

View Set

Digital Marketing Chapter 3 and 4

View Set

U40: L4: Voyages of Exploration : Practice quiz

View Set

prep Chapter 14: Clinical Judgment

View Set

Texas Statutes and Rules Pertinent to Life Insurance Only

View Set